In meiner Serie über grundlegende Fragen zur Astronomie habe ich letzte Woche erklärt, wie groß das sichtbare Universum ist. Aber das was wir sehen, ist nicht das gesamte Universum. Und eine häufige Frage, die mir in diesem Zusammenhang immer wieder gestellt wird, lautet: Ist das Universum unendlich groß?

Die Frage lässt sich schnell beantworten: Wir wissen nicht, ob das Universum unendlich groß ist oder nicht. Aber es lohnt sich trotzdem, ein bisschen genauer darüber nachzudenken. Alles hängt mit der Form des Universums zusammen. Hier wird es allerdings ein wenig knifflig. Das Universum in dem wir leben hat drei Raumdimensionen (diverse Hypothesen der Stringtheorie mit mehr Dimensionen ignorieren wir jetzt einfach mal). Und wir können uns zwar sehr einfach alle möglichen Formen vorstellen, die in einem dreidimensionalen Raum existieren. Die Form des dreidimensionalen Raums selbst dagegen kann man sich nicht mehr direkt vorstellen sondern muss mit Analogien arbeiten.

Es gibt im Prinzip nur drei unterschiedliche Formen, die in Frage kommen: Das Universum kann positiv gekrümmt sein, negativ gekrümmt sein oder gar nicht gekrümmt sein. Wie kann man sich das vorstellen? Nehmen wir als Beispiel die Erde: Die Oberfläche unseres Planeten ist positiv gekrümmt. Eine Eigenschaft so einer Oberfläche ist ihre Unbegrenztheit. Man kann über die Erde spazieren so lange man möchte und wir doch nie auf ein “Ende” stoßen. Die Erdoberfläche ist zwar nicht unendlich groß, aber trotzdem unbegrenzt. Die Oberfläche der Erde ist allerdings zweidimensional. Um das mit dem Universum zu vergleichen müssten wir uns eine vierdimensionale Kugel vorstellen, deren Oberfläche dann ein dreidimensionaler Raum mit gekrümmter Oberfläche wäre. Das können wir uns allerdings nicht vorstellen – trotzdem gilt auch hier: Ist das Universum positiv gekrümmt, dann käme man auch hier nie zu einem Ende sondern würde – nach einer sehr, sehr langen Zeit – bei jeder Reise die immer nur in eine Richtung führt am Ende wieder am Ausgangspunkt landen. Genau so wie auch ein Flugzeug problemlos um die Erde fliegen und wieder am Startflughafen landen kann.

Eine Analogie für ein Universum ohne Krümmung ist ein Blatt Papier, dessen Oberfläche völlig eben ist. Besteht das Universum aus einem dreidimensionalen flachen Raum, dann ist es allerdings nicht irgendwann zu Ende, wie das bei einem normalen Blatt Papier der Fall ist. Es wäre dann unendlich groß und man könnte in jede Richtung beliebig lange reisen ohne jemals an den Ausgangspunkt zurück zu kehren (Es gibt Spezialfälle eines flachen Universums das trotzdem nicht unendlich groß ist – zum Beispiel einen “Hypertorus” – auf die ich aber jetzt nicht eingehen will).

Ein Universum mit negativer Krümmung lässt sich ebenso schwer veranschaulichen. Eine Analogie wäre eine Form, die wie ein Pringles-Chip aussieht beziehungsweise ein Reitsattel. Auch in so einem Universum könnte man beliebig lange reisen, ohne jemals zurück zu kehren.

Welche Form das Universum tatsächlich hat, wissen wir noch nicht. Es deuten aber alle bisher gemachten Beobachtungen und Messungen darauf hin, dass es flach ist; also keine Krümmung hat. Und damit auch unendlich groß ist. Das mag jetzt vielleicht seltsam klingen, denn es gab ja vor 13,8 Milliarden Jahren einen Urknall. Wenn als vor einer endlich langen Zeit alles in einem Punkt begonnen hat und sich seitdem ausdehnt, wie kann es dann möglich sein, dass das Universum unendlich weit ausgedehnt ist und flach ist? Muss es nicht eine “Kugel” sein, also eine positive Krümmung haben?

Nein, muss es nicht. Denn die Vorstellung, dass der Urknall an einem einzigen “Punkt” stattgefunden hat, wird zwar häufig verwendet, ist aber nicht die einzige Möglichkeit, sich das Ganze vorzustellen. Stellen wir uns (sofern wir das können) ein unendlich großes Gummituch vor. Wenn sich das nun ausdehnt und seine Größe verdoppelt, ist es danach natürlich immer noch unendlich groß. Das gleiche gilt, wenn wir es schrumpfen lassen: Es bleibt immer unendlich groß. Wenn das Universum also unendlich groß ist, dann war es immer schon unendlich groß und wird immer unendlich groß sein. Es war vor allem auch zum Zeitpunkt des Urknalls unendlich groß!

Beim Urknall begann das Universum zu expandieren und deswegen bewegen sich heute alle Galaxien voneinander weg. Aber der Urknall fand in diesem Fall eben nicht an einem einzigen Punkt statt. Er fand genaugenommen überall im unendlich großen Raum statt! Und seitdem bläht sich der gesamte Raum auf, alles entfernt sich voneinander – aber das Universum wird deswegen nicht größer. Es war und bleibt immer unendlich groß.

Wie gesagt: Wir wissen nicht genau, welche Form das Universum hat. Aber alles deutet derzeit darauf hin, dass es flach ist. Und unendlich groß.

P.S. Ich habe jetzt auch eine Übersichtsseite zu den Fragen erstellt, wo alle Antworten nochmal gesammelt verlinkt sind und ihr selbst auch Fragen stellen könnt.

Kommentare (227)

  1. #1 SonnenKlar
    21. Juli 2014

    Eigentlich wollte ich zuerst eine Frage stellen. Als ich den Kommentar fertig hatte, bemerkte ich aber, daß es keine ist. Daher hier:

    Die Beschreibung mit dem Universum, das “schon immer” unendlich groß war, auch beim Urknall habe ich in Deinem Blog jetzt zum ersten Mal gehört. Vorstellen kann ich mir den “überall im unendlichen Raum” gleichzeitig auftretenden Urknall auch nicht.

    Bisher hatte ich es so verstanden, daß es eine Möglichkeit wäre, daß im Multiversum ewige Inflation herrscht und Universen am laufenden Band entstehen und aufhören zu existieren, wie Wasserbläßchen in einem Aquarium.
    In der Vorstellung gibt es doch für jedes Universum einen Punkt in dem sie entstehen.
    Selbst ohne angenommenes Multiversum, hätte der Urknall doch seinen Ausgangspunkt in einer Singularität gehabt. Wenn gesagt wird, man kann ihn bis auf ca. 5,391·10-44 Sekunden genau beschreiben, dann bezieht sich das doch auf einen Vorgang an einem “Punkt”, oder? o_O

    Bissher hatte ich mir das Universum so vorgestellt, wie rechts auf dem Bild bei Wikipedia zu sehen ist:
    https://en.wikipedia.org/wiki/Universe

  2. #2 André Lampe
    Berlin
    21. Juli 2014

    Ich habe immer mal wieder gelesen, “dass es keinen Sinn macht nach dem Raum oder der Zeit vor dem Urknall zu fragen, weil Zeit und Raum erst beim Urknall entstanden sind” – vereinfacht gesagt. Wenn ich da was vollkomme überholtes im Oberstübchen gespeichert habe, würde ich mich über Aufklärung freuen, ansonsnten wäre es schon spannend wie dieser Aspekt eine Rolle spielen kann bei dem was du hier erklärt hast. Jedenfalls Danke für die Aufklärungsarbeit – vervolge es mit Spannung.

  3. #3 André Lampe
    Berlin
    21. Juli 2014

    …verfolge…. #facepalm

  4. #4 SonnenKlar
    21. Juli 2014

    Ah… hab meinen Denkfehler erkannt. Das Bild zeigt das beobachtbare Universum.

    Trotzdem verstehe ich die Sache mit dem überall gleichzeitig stattfindenden Urknall nicht.
    Würde das nicht gegen ein Multiversum sprechen?

    Bisher hatte ich “unendlichen, geraden Raum” so verstanden, daß er deshalb unendlich ist, weil er sich ja noch ausdehnt und damit wohl auch niemals aufhören wird.

  5. #5 sax
    21. Juli 2014

    Hmm, aber beim Urknall waren aber beliebig weit voneineander getrennte Punkte beliebig nah beisammen.

    Nehmen wir das Gummituch als vorstellung, um es noch einfacher zu machen, nehmen wir einfac ein unendlich großes Gummiband.

    Nun malen wir zu einem beliebigen Zeitpunkt, sagen wir genau JETZT in der Gegenwart Maßstabsmarkierungen auf das Gummiband auf, in Abstand von sagen wie einem Meter.

    Wollen wir nun den Abstand zwischen 2 Objekten A und B auf dem Gummiband wissen, zählen wir einfach die Anzahl der Masßstabsmarkierungen auf dem Band. Wie erhaten einen Abstad, ich nenne ich mas d. Nun dehnen oder stauchen wir das Gummiband so das es überall gleich gedehnt oder gestaucht wird, dann ist der neue Abstand ich nenne ihn D einfach ein vielfaches des alten Abstandedes.
    D=S*d. D nenne ich Maßstabsentfernung (das würde man mit einem Metermaßstab messen), d ist der Abstand der Objekte in der Gegenwart und S nenne ich Skalenfaktor.

    Nun nehmen wir an das Gummiband dehnt sich völlig gleichmäßig immer weiter aus, dann ist der Skalenfaktor zeitbahängig und der Abstand zweier Objekte als Funktion der Zeit ist:
    D(t)= S(t) *d.
    und so ungefähr sieht die Metrik des UJniversums aus, vorrausgesetz das Universum ist homogen und isotrop.

    Den Ort eines jedesn Punktes kann ich durch d angeben, die Angabe in d entspricht einem Koordinatensystem, das sozusagen an das Gumminband angeklebt ist und mit diesem mit expandiert.

    Welche Form die Funktion S(t) hat hängt von der Dichte des Universum ab. Am Anfang, also zum Urknall, watr S(t)=0.

    Damit ist die Entfernung zwischen allen Objekten die heute einen endlichen Abstand voneinander haben zum Zeitpunkt des Urknalls Null gewesen. Bemerkenswerterweise kann es zu einem beleibig kleinem Zeitpunkt S(t>0), wo S zwar winzig klein, aber eben endlich ist, schon beliebig weit entfernte Orte in Universum gegeben haben.

    Damit war der “Ort” des Urknalls aber wirklich “überall”, denn alle Orte die es gibt, hatten singulär (nur exakt zu t
    =0) den Abstand D=0 voneinander. In gewisser Weise war alles an einem Punkt, aber dieser Punkt war nicht an einem bestimmten Ort in Raum und Zeit, er war damals alles was es gab und was vonb ihm blieb ist heute überall. In diesem Sinne kann jetzt auch guten Gewissens sagen, der Urknall vond genau HIER! statt.

  6. #6 SonnenKlar
    21. Juli 2014

    @sax
    So verstehe ich das auch. Aber Florian hat ja gesagt:
    “Aber der Urknall fand in diesem Fall eben nicht an einem einzigen Punkt statt. Er fand genaugenommen überall im unendlich großen Raum statt!”

    Das scheint dem aber zu widersprechen.

  7. #7 Matthias Wolf
    21. Juli 2014

    »Er [der Urknall, Anm.] fand genaugenommen überall im unendlich großen Raum statt!«

    Das wirft allerdings die Frage auf, wie die Synchronisation in einem unendlich großen Raum statt gefunden haben könnte.

    Außerdem ist ein ausgedehnter Raum der Relativität unterworfen – damit könnten sich zwei Beobachter in verschiedenen Inertialsystemen /nicht/ mehr auf einen Zeitpunkt des Urknalls einigen bzw. würden nicht beide einen einzigen Knall ›sehen‹.

    Gibt es zu diesen offensichtlichen Problemen Anworten?

  8. #8 Alderamin
    21. Juli 2014

    @Sonnenklar

    Lies Dir das mal durch, dann sollte klar sein, wie man aus unendlicher Zeit unendlichen Raum machen kann:

    https://edge.org/conversation/next-step-infinity

    tl;dr

    Wenn man sich unser Universum als eine wachsende Blase in einem ein inflationär expandierenden Raum vorstellt, dann findet an der Innenwand dieser Blase permanent ein Phasenübergang von inflationärer Expansion zu unserem normalen Vakuum statt, d.h. der Urknall. Definiert man den Zeitbegriff innerhalb der Blase in Richtung zunehmender Entropie, dann ist die Außenwand der Blase (minimale Entropie) für die Insassen der Zeitpunkt Null, obwohl sie aus externer Sicht über einen unendlich langen Zeitraum anwächst. Der von außen gesehen unendlich lange Zeitraum wird für die Insassen zur unendlichen Größe zum Zeitpunkt Null.

  9. #9 sax
    21. Juli 2014

    @Mathias Wolf
    Das wirft allerdings die Frage auf, wie die Synchronisation in einem unendlich großen Raum statt gefunden haben könnte.
    was für eine Synchronisation meinst du, verstehe die Frage gerade nicht.

    Außerdem ist ein ausgedehnter Raum der Relativität unterworfen – damit könnten sich zwei Beobachter in verschiedenen Inertialsystemen /nicht/ mehr auf einen Zeitpunkt des Urknalls einigen bzw. würden nicht beide einen einzigen Knall ›sehen‹.

    Hier stößt die spezielle Relativitätstheorie an ihre Grenzen, diese funktioniert eigentlich nur, wenn es keine Gravitation gibt, oder diese nicht allzu groß ist.

    Ich schweife erst mal etwas aus um das klar zu machen:
    Stelle dir vor du bist in einem Hochhaus, in einem frei fallendem Fahrstuhl. Dort heben sich Trägheit und Graviation auf, und du bist dort drin praktisch schwerelos, es gelten die Newtonschen Gesetze und damt bist du nach allem was du messen kannst in einem Inertialsystem. Ein kleiner Kasten, der sich in diesem Fahrstuhl geradlinig gleichförmig bewegt, würde ebenfalls ein Inetialsystem darstellen. Zwischen Fahrstuhl und dem Kasten würde die Lorentztransformation gelten und die üblichen Aussagen der SRT. Du könntest den Kasten im Fahrstuhl auch als Inertialsystem vbeschreiben, denn er ist ja bzgl. deines Fahrstuhls nicht beschleunigt.

    Jetzt stell dir vor, jemand befindet sich auf der Gegenüberliegenden Seite der Erde ebenfalls in einem frei fallenden Fahrstuhl. Dort hebt sich enbenfalls Gravitation und Trägheit gegeneinander auf und er befindet sich ebenfalls in einem Inertialsystem.

    Aus deiner sicht betrachtet bewegt sdich dieser Fahrstuhl aber mit doppelter Erdbeschleunigung auf dich zu, du würdest also, wüßtest du nicht um die Gravitation, diesen Fahrstuhl nicht als Inertialsystem wahrnehmen. Zwischen den beiden Fahrstühlen ist die Lorentztransformation nicht anwendbar und die Dinge die ma in der SRT über relativität der Gleichzeitigkeit lernt muss man hier noch einmal neu überdenken.

    Fie Fahrstühle stellen jeweils lokale Inetzialsysteme dar, genaugenommen sind sie nur im Schwerpunkt des Fahrstuhlss Inertialsysteme, ein bisschen davon weg heben sich Gravitationskraft und Trägheitskraft schon nicht mehr exakt gegeneinadner auf.

    Ähnlich ist es mit dem expandierendem Universum. Kehren wir noch ein mal zum Gummibandbild zurück. Jeder Punk ist gegen jeden beschleunigt. In diesem Bild gibt es eigentlich gar keine Inertialsysteme.

    Aber jeder Punkt ist für sich genommen ein lokales Inertialsystem (Da Gravitation vom Rest des Universum und die Trägheit durch beschleunigte Ausdehnung gerade im Gleichgewicht sind). Eine imaginäre Stoppuhr die an einem beliebigen Punkt aufgestellt wäre (die gegenüber dem ‘gummiband” ruht) und die Zeit vom Urknall an misst würde heute irgenwas anzeigen. Da aber jeder Punkt de Universums nach dem kosmologischem Prinzip gleichberechtigt ist, zeigt jede Uhr das alter des an diesem Punkt beobachtbaren Universums an, solche Uhren wären auch synchron.

  10. #10 sax
    21. Juli 2014

    Leider gibts immer noch kein Vorschaufunktion bei den SB, ich hoffe mein Text ist trotz der kaputten Formatierung noch lesbsar.

  11. #11 SonnenKlar
    21. Juli 2014

    @Alderamin
    Danke, super Artikel!
    Hab mir alles durchgelesen und glaube es jetzt ein wenig besser zu verstehen.

    PS:
    Ist mit dem Inflaton-Feld das Multiversum gemeint?

  12. #12 Christian
    Unna
    21. Juli 2014

    Ich versuche mal, meine verwirrten Gedanken zu formulieren. Wenn doch im (beobachtbaren) Universum zwei Objekte einen gewissen Abstand haben und sich das Universum ausdehnt, dann kann man das Ganze doch zurückrechnen und herausbekommen, wann die Entfernung nur noch halb so groß war. Und irgendwann kommt zu einem Zeitpunkt, wo alles nah beieinander war: der Urknall vor ca. 13,75 Milliarden Jahren. Dabei entstanden Raum, Zeit und Energie, also die Materie. Im Text habe ich gelesen, dass der Raum schon immer unendlich war. Also kein Beginn? Ready State-Universum? Aber das widerspricht doch alles, was wir bisher herausbekommen haben. Das ganze Standardmodel der Physik mit der Entstehung der Elementarteilchen basiert doch auf eine Entstehung des Universums. Wenn dieses aber schon vorher da war…
    Ich sehe also den Widerspruch dort wo geschrieben steht, dass das Universum beim Urknall unendlich groß war. Aber beim Urknall entstand doch erst alles, wie kann denn da schon etwas gewesen sein? Oder entstand in einem schon bestehenden unendlichen Universum vor kanpp 13 Milliarden Jahren an einem bestimmten Punkt dieses Universums ein “Urkanll”, der nur hier diese Geschichte, wie wir sie kennen, beginnen ließ? Und das dann das beobachtbare Universum ist? Aber das wäre doch dann das klassische Model des Multiversums oder Blasenuniversums, oder?
    Wenn ich aber den letzten Teil so durchlese: “Beim Urknall begann das Universum zu expandieren und deswegen bewegen sich heute alle Galaxien voneinander weg. Aber der Urknall fand in diesem Fall eben nicht an einem einzigen Punkt statt. Er fand genaugenommen überall im unendlich großen Raum statt! Und seitdem bläht sich der gesamte Raum auf, alles entfernt sich voneinander – aber das Universum wird deswegen nicht größer. Es war und bleibt immer unendlich groß. ”
    Ich verstehe das jetzt so: Das gesamte unendliche Universum existierte so vor sich hin, und irgendwann (vor 13 Milliarden Jahren?) schickte es sich an, sich auszudehnen. Wenn es aber schon immer existiert hat, warum begann es bei der Ausdehnung plötzlich die Hintergrundstrahlung zu entwickeln, die erstem Elemente usw. Bei einem schon immer bestehendem Universum vergrößert sich doch lediglich der Abstand zwischen zwei Orten/Teilchen, die schon immer da waren. Ist denn alles, was wir mit Hilfe von CERN gelernt haben für die Katz?
    Ich hoffe, ich konnte mich so einigermaßen verständlich ausdrücken.

  13. #13 Florian Freistetter
    21. Juli 2014

    @André: “Wenn ich da was vollkomme überholtes im Oberstübchen gespeichert habe, würde ich mich über Aufklärung freuen, “

    Das Problem ist, das man solche Sachen kaum veranschaulichen kann; darum beschränkt man sich meistens auf das klassische Bild der Explosion. Aber es geht im wesentlich “nur” darum, dass früher alles viel dichter und heißer war. Das kann ein einziger, dichter und heißer Punkt sein. Es kann aber auch einfach alles zusammenrücken und wenn das in einem unendlichen Universum passiert, ist das am Ende immer noch unendlich groß.
    Ich empfehle da immer das Buch “Der Stoff aus dem der Kosmos ist” von Brian Greene, wo diese und andere Fragen umfassend erklärt werden.

  14. #14 Florian Freistetter
    21. Juli 2014

    @Christian: “Wenn doch im (beobachtbaren) Universum zwei Objekte einen gewissen Abstand haben und sich das Universum ausdehnt, dann kann man das Ganze doch zurückrechnen und herausbekommen, wann die Entfernung nur noch halb so groß war.”

    Ja. Aber nimm zB die Zahlengerade. Die geht von 1 nach 2 nach 3 bis unendlich. Jetzt teilst du die gesamte Gerade durch 2. Also 1/2 nach 1 nach 3/2 und so weiter. Die ist am Ende aber IMMER noch unendlich lang. Ein unendlich großes flaches Universum war immer schon unendlich groß.

    “Wenn es aber schon immer existiert hat, warum begann es bei der Ausdehnung plötzlich die Hintergrundstrahlung zu entwickeln, die erstem Elemente usw.”

    Es geht darum, dass es früher überall sehr viel heißer und dichter war. Dann kommt der Rest ganz automatisch. Das Universum war nicht immer schon so wie es jetzt ist. Sondern früher halt – aus welchen Gründen auch immer – in dem Zustand, den wir dem “Urknall” zuschreiben. Aber es muss nicht punktförmig gewesen sein.

  15. #15 Alderamin
    21. Juli 2014

    @Sonnenklar

    Ist mit dem Inflaton-Feld das Multiversum gemeint?

    Nicht ganz, das Inflaton-Feld ist (sowas wie? oder genau?) die Vakuumenergie, die das Weltall inflationär aufgebläht hat, bis es auf einen niedrigeren Energiezustand fiel und die überschüssige Energie als Strahlung in den entstandenen Raum warf – der Urknall. Daraus kann, muss aber nicht zwingenderweise, ein Multiversum hervorgehen. Es kommt auf die Variante der Inflation an. Die neulich gemessenen BICEP2-Werte, falls sie bestätigt werden, deuten eher auf ein Multiversum hin, aber auch nicht völlig alternativlos (das Wort klaue ich mal von Frau Merkel 🙂 )

  16. #16 Martin Windischer
    21. Juli 2014

    “Es deuten aber alle bisher gemachten Beobachtungen und Messungen darauf hin, dass es flach ist; also keine Krümmung hat.”

    Wie misst man so was eigentlich? Die einzige mir bekannte Methode wäre, dass man ein großes Dreieck nimmt, und die drei Winkel abmisst. Da wir aber kaum aus unserem Sonnensystem, geschweige denn unserer Galaxie rauskommen, können wir kein wirklich großes Dreieck nehmen. Das wäre dann vergleichbar mit einer Person, der den Boden mit einem Lineal vergleicht und daraus schließt, dass die Erde flach sein müsste.

  17. #17 Matthias Wolf
    21. Juli 2014

    @Sax (#9): Lb. Sax, danke für die einleuchtende Erklärung! (Keine Sorge, war tadellos lesbar.)

    Es bleibt (mir) aber ein Problem übrig: die Homogenität in der Hintergrundstrahlung, die wir beobachten und unter diesem Licht erst einmal erklären müssten (besser als 1/100‰) .

    Das bedeutet doch, dass die Expansion tatsächlich in allen Punkten des beobachtbaren Universums ›gleichzeitig‹ (wenigstens aus unserem lokalen Inertialsystem heraus gesehen) begonnen haben muss, denn sonst müssten wir wärmere und kühlere Regionen erkennen können, was wir aber nicht tun.

    Und auch das bringt mich zum Problem der ›Synchronisierung‹, das ich oben gemeint habe. Da das beobachtbare Universum raumartig ›durchverbunden‹ ist, ist es erst einmal eine Unmöglichkeit, eine Expansion in allen Punkten gleichzeitig (mit Proper distance = räumliche Distanz zwischen allen Expansionsbeginnen in allen Punkten) beginnen zu lassen.

    Könnte das (so aus dem hohlen Bauch) für das ›holographische Universum‹ (https://www.scientificamerican.com/article/universe-really-is-a-holo/ ) sprechen, in dem wir, und das was wir beobachten, lediglich eine Projektion einer höherdimensionalen Universums ist?

  18. #18 SonnenKlar
    21. Juli 2014

    @Alderamin
    Dann habe ich’s doch noch nicht ganz verstanden.
    Ich dachte wir seien mit unserem Universum nur eine (unendlich kleine) blase im Multiversum, das der ewigen Inflation unterliegt und daß das “Inflaton”-Feld genannt wird. Daß unser Universum sozusagen nur ein Teil des Inflatonfeldes ist, dessen Inflationswert unter 1 ist.

    Zum Beispiel wie hier:
    https://www-tc.pbs.org/wgbh/nova/assets/img/johnson-multiverse/image-02-small.jpg
    Die Bläschen wären die Universen, die Flüssigkeit das Inflatonfeld. Die Gesamtheit das Multiversum.

    Ist das falsch? Oder heisst ewige Inflation (außerhalb unseres Universums) nicht automatisch, daß es ein Multiversum gibt?

  19. #19 Alderamin
    21. Juli 2014

    @Sonnenklar

    Doch, ewige Inflation heißt Multiversum, aber die kosmische Inflationstheorie gibt es auch in Varianten, wo die Inflation komplett überall aufhört und nur ein Universum entsteht.

    Dann bekommt man allerdings das Problem zu erklären, warum bei uns die Naturkonstanten so fein abgestimmt sind, dass es Atome, Sterne, Planeten und Leben in dem einen und einzigen Universum gibt. Im Multiversum könnte jedes Blasenuniversum andere Naturgesetze haben, und dann ergibt sich unseres ganz zwanglos aus der Statistik und dem anthropischen Prinzip. Aber das ist kein Beweis für die Richtigkeit der ewigen Inflation.

  20. #20 SonnenKlar
    21. Juli 2014

    Super danke 🙂

  21. #21 Alderamin
    21. Juli 2014

    @Matthias Wolf

    Der kosmische Hintergrund des beobachtbaren Weltalls ist homogen und das erklärt man damit, dass es vor der Inflation im Strahlungsgleichgewicht war und die Inflation Bereiche, die sich vorher noch thermisch durch Strahlung ausgleichen konnten, so weit auseinander gerissen hat, dass sie das heute (und auch schon zur Zeit der Entstehung der Hintergrundstrahlung) nicht mehr können.

    Über den Bereich jenseits des beobachtbaren Universums können wir keine Aussagen machen und wissen auch nicht, ob es mit unserem Bereich ebenfalls vor der Inflation im thermischen Gleichgewicht war.

  22. #22 sax
    21. Juli 2014

    @Mathias Wolf

    Das bedeutet doch, dass die Expansion tatsächlich in allen Punkten des beobachtbaren Universums ›gleichzeitig‹ (wenigstens aus unserem lokalen Inertialsystem heraus gesehen) begonnen haben muss, denn sonst müssten wir wärmere und kühlere Regionen erkennen können, was wir aber nicht tun.

    Ich will mich nicht zu weit aus dem Fenster lehnen, denn ich kenne mich nur mit dem standartmodell der Kosmologie ein bischen aus. (Also das was man in Lehrbüchern findet).

    Nach den Gleichungen die das oben von mir erwähnte S
    (t) beschreiben (Friedmann-Lemaître-Gleichung), wäre es schwer zu verstehen wenn der Skalenfaktor des Universums mit einem anderem Wert als 0 begonnen haben sollte. Wie ich oben schon schrieb ergibt sich der Abstand zwischen (außer der expansion) ruhenden Objekt als Abstand zum heutigen Zeipunkt bei dem S(t_h)=1 gilt. (Man kann aber auch für jeden beliebigen anderen Zeitpunkt als S=1 definieren), multipliziert mit diese Skalenfaktor.
    Wenn es den Zeitpunkt des Beginns S(0)=0 wirklich gab, war damals jedes noch so weit abetr endlich entfernte Objekt an ein und dem selben Punkt. Nur ein Objekt für das man wirklich unendlich als Entfernung angeben muss, wäre dann nicht mehr an diesem Punkt da “0*unendlich” alles mögliche ergeben kann (wenn man es als Grenzwert versteht). In jedem Fall war aber zum Urknall das Gesamte beobachtbare Universum an einem Punkt konzentriert, und es ist dann nicht verwunderlich, das das beobachbare Universum synchron mit der Expansion begann. (und den Rest kann man nicht beobachten, per Definition)

    Eigentlich sehe ich nicht, wie man im Rahmen der klassichen kosmologischen Modelle von einem zum Urknall bei t=0 (bei dem S(0)=0) galt, von einem undendlich ausgedehntem Kosmos sprechen kann.

    NNur wenn wir über beliebig kurze Zeiten nach dem singulären Zeitpunkt t=0 sprechen, kann man sagen, wenn das Universum flach oder hyperbolisch ist, war es für jeden Zeitpunkt nach dem Urknall unendlich groß.

    Betrachten wir mal kurz für zwei beliebige Punkte A und B im Universum und fragen wann diese, sagen wir so weit wie ein Atomdurchmesser, voneinander entfernt waren. Wir können dann immer einen Zeitpunkt angeben, wann dies der Fall war. Es gibt aber dennoch auch zu diesem Zeipunt noch Punkte die sehr sehr weit voneinander entfernt sind. Für jeden beliebig kleinen Zeitraum nach dem Urknall gibt es Punkte die noch “unendlich” weit weg sind, erst bei t=0 ist wirklich alles auf einem Punkt. (btw: Das erinnert mich irgendwie an das Problem von gleichmäßiger und Punktweiser Konvergenz).

  23. #23 Alderamin
    21. Juli 2014

    @sax

    Die Friedmann-Gleichung gilt aber schon nicht mehr während der Inflation. Außerdem wissen wir ja, dass die ART auf beliebig kleinen Skalen unsinnige Werte ergibt. Insofern kann man die Freidmann-Gleichung nicht mit Fug und Recht bis zur Singularität anwenden…

  24. #24 McIng
    21. Juli 2014

    Frage in die Runde: Ein wesentliches Postulat in den beschriebenen Fragestellungen ist doch die Unbeschränktheit, es gibt also keine Grenze des Raumes/Zeit. Ist nicht jedes Schwarze Loch eine solche Grenze, an der das Universum begrenzt ist?

  25. #25 sax
    21. Juli 2014

    @Aldemarin
    Okay, aber dann wissen wir eigentlich gar nicht nicht ob das Universum zum Anfang eine ausdehnung hatte oder nicht.

    Das die Friedmann Gleichungen während der Inflation nicht gelten ist mir neu, ich dachte bisher das aufgrund dieses nicht näher spezifizierten Quantenfeldes eine Zustandsgleichung fürs Universum gilt, in welcher der Druck negativ wird, was laut der Friedmann Gleichung zu einer extremen Inflation führt, aber wie gesagt, ich habe nur Grundlagenbücher zu dem Thema gelesen und da wurde die Inflation maximal kurz angeschnitten. Welche Gleichungen galte denn während der Inflation?

  26. #26 Alderamin
    21. Juli 2014

    @sax

    Welche Gleichungen galte denn während der Inflation?

    Keine Ahnung, hier steht auch, dass die Expansion anschließend an die Inflationdurch die Friedmann-Gleichung beschrieben wird. Mindestens die kosmologische Konstante muss sich aber dramatisch geändert haben. Dass der negative Druck zur positiven Expansion führt, habe ich auch so gelesen, und dass dies aus der ART folgt. Ob das in dieser Form noch die Friedmann-Gleichung ist, weiß ich nicht.

  27. #27 Florian Freistetter
    21. Juli 2014

    @Martin: “Wie misst man so was eigentlich? “

    zB über die Hintergrundstrahlung. Je nach Krümmung ist die Verteilung der HotSpots in der Strahlung unterschiedlich.

  28. #28 Matthias Wolf
    21. Juli 2014

    @Alderamin (#19): ebenfalls danke. Das ist mir alles durchaus bekannt – ich sehe bloß nicht, was es mit dem Problem des ›gleichzeitigen‹ Inflationsereignisses zu tun hat.

    @Sax (#22): »In jedem Fall war aber zum Urknall das Gesamte beobachtbare Universum an einem Punkt konzentriert,« – genau das wird aber hier in Abrede gestellt!?

    »und es ist dann nicht verwunderlich, das das beobachbare Universum synchron mit der Expansion begann.« Absolut. Ich versuche, für mich die Nuss zu knacken, wie ein Urknall in einem unendlich MAL kleineren Universum, das aber selbst immer noch unendlich groß war, GLEICHZEITIG gefeuert worden sein könnte.

    Aber, danke, Leute, lasst gut sein! Ich sehe, ich muss mich vorher nochmal dringend über meinen Kosmologie-Schmöker hermachen! (Stephen Serjeant: Observational Cosmology; Cambridge Univerity Press; 2010)

  29. #29 Timo Bienert
    21. Juli 2014

    Mein erster Kommentar hier, aber das Thema beschäftigt mich schon seit Jahren. Wie schon bei der Physik des schwarzen Loches versagt diese, wenn die Größenordnung “unendlich” ins Spiel kommt. Meines Erachtens ist dies zwar mathematisch möglich, aber die Physik unseres Universums lässt “unendlich” nicht zu. Man kann sich zwar wie eingangs von Florian erwähnt, das Universum in seiner Form nicht vorstellen, aber meiner Meinung nach ist die positive Krümmun aufgrund der Analogie zur Kugel plausibler. In diesem Zusammenhang empfehle ich das Buch “Der Krieg um das schwarze Loch” von Leonard Susskind, dass, sofern man “die Neuverdrahtung” einigermaßen hinbekommt, eine, und das ist maßlos untertrieben, “interessante” Sicht auf den möglichen “Aufbau” des Universums analog zum schwarzen Loch, Stichwort holographisches Prinzip, bekommt. Soll heißen, so wie der Ereignishorizont “unser” Universum vom Inneren des schwarzen Lochs und damit den dort vorhandenen, mglw. “anderen” physikalischen Eigenschaften trennt, könnte ebenfalls ein Ereignishorizont an der “äußeren Grenze” unseres Universums uns von möglichen anderen Universen trennen. Dies wäre in einer Unendlichkeit allerdings nicht möglich.

  30. #30 Florian Freistetter
    21. Juli 2014

    @Timo: “aber meiner Meinung nach ist die positive Krümmun aufgrund der Analogie zur Kugel plausibler.”

    Nur sprechen da halt derzeit die Messungen dagegen. Wenn, dann muss es WIRKLICH groß sein, so groß, damit uns die positive Krümmung nicht auffällt, weil sie so schwach ist.

  31. #31 Rolf Schälike
    Hamburg
    21. Juli 2014

    Das Universum ist mindestens vierdeminsional. Die Zeit als vierte Dimension sollte nie vergessen werden. Das ist unstrittig.

  32. #32 Florian Freistetter
    21. Juli 2014

    @Rolf Schälike: “Das Universum ist mindestens vierdeminsional. Die Zeit als vierte Dimension sollte nie vergessen werden. “

    Da es hier aber um die räumliche Ausdehnung geht, spielt die Zeit keine große Rolle…

  33. #33 Rolf Schälike
    Hamburg
    21. Juli 2014

    Die Zeit ist ebenfalls eine räumliche Koordinate, gleichberechtigt mit den von uns empfundenen drei räumlichen Koordinaten. Der Rauzm ist eben mindestens vierdeminsional.

    Es ist vergleichbar mit dem Empfinden, die Erde sei eine Scheibe (zweideminsional). Auch, dass Luft etwas wiegt, ist für viele – vor allem Kinder – nicht offensichtich.

    Ich empfinde das Weltall vierdimensional und die Erde nicht als eine Scheibe.

    Dieses Empfinden gehört zum Allgemeinwissen. Es ist nicht allzu schwer, das zu begreifen.

  34. #34 noch'n Flo
    Schoggiland
    21. Juli 2014

    @ R. Schälike:

    Ich empfinde das Weltall vierdimensional und die Erde nicht als eine Scheibe.

    Hat das denn hier irgendjemand behauptet? Der Sinn Ihrer Aussage erschliesst sich mir irgendwie nicht. Insbesondere

    Dieses Empfinden gehört zum Allgemeinwissen.

    ist schon einmal unsinnig.

  35. #35 DeLuRo
    21. Juli 2014

    @RS: “Dieses Empfinden gehört zum Allgemeinwissen.”

    Also, ich empfinde nicht so, und dann habe ich wohl zu wenig Allgemeinwissen. Beim Empfinden hört die Naturwissenschaft auf; das ist eher was für Psychologen. Wissenschaft braucht reproduzierbare Werte, und die kann man nicht durch Allgemeinswissen einforden.

    Mag sein, dass man in der Relativitätstheorie von einer 4-dimensionalen Raumzeit spricht, und in Stringtheorien von 11 oder 22 Dimensionen. Eines lässt sich aber sicher sagen, und das reduziert wieder auf 3 Raumdimensionen: nur in diesen Raumdimensionen können wir (zumindest theoretisch, zumindest in handlichen Abständen) eine Distanz zwischen zwei Punkten messen. Können wir die Distanz nur durch die Ausbreitung elektromagnetischer Wellen messen, müssen wir uns schon auf die Relativitätstheorie verlassen.

    Bei der Zeit aber können wir uns als Beobachter (als messender Physiker) immer nur an einem einzigen nicht willentlich festlegbaren Punkt befinden, und wollen wir Zeit( -länge, -dauer) messen, müssen wir mittels eines Vergleichsmaßstabes mitzählen, z.B. Schwingungen von Atomen.

  36. #36 Rolf Schälike
    Hamburg
    21. Juli 2014

    @DeLuRO: “Beim Empfinden hört die Naturwissenschaft auf; das ist eher was für Psychologen. Wissenschaft braucht reproduzierbare Werte, und die kann man nicht durch Allgemeinswissen einforden.”

    Absolut richtig: Durch reprodizierbare Werte, die wahrgenommen werden, entwickelt sich das Wissen und das Empfinden.

    Alledings muss die Bereitschaft bestehen, repruduzierbare Werte wahrzunhemen. Kostet oft Überwindung und viel Arbeit.

    Bestimmte reproduzierbare Werte gehören zum Allgemeinwissen, wie z.B., dass die Erde keine Scheibe ist, dass die Luft erwas wiegt.

  37. #37 DeLuRo
    21. Juli 2014

    @RS: “Reproduzierbare Werte” heißt vor allem: Objektiv zählbar oder sonst messbar, und “objektiv” heißt unabhängig von einer bestimmten Person. Da Empfinden individuell und subjektiv ist, gibt dies meistens einen Konflikt und schließt sich in der Regel gegenseitig aus. Ausnahmen mag es geben.

  38. #38 Rolf Schälike
    Hamburg
    21. Juli 2014

    @DeLuRo: ” “Reproduzierbare Werte” heißt vor allem: Objektiv zählbar oder sonst messbar, und “objektiv” heißt unabhängig von einer bestimmten Person. Da Empfinden individuell und subjektiv ist, gibt dies meistens einen Konflikt und schließt sich in der Regel gegenseitig aus. Ausnahmen mag es geben.”

    Meßwerte gelangen immer an dem Menschen. Die Meßwerte werden empfunden. Wo ist da der Konflikt?

    Richtig ist allerdings auch, dass man z.B. mathematisch durchaus davon ausdgehen kann, dass die Erde der Mittelpunkt der Welt ist und das die Sonne sich um die Erde dreht.

    Ist allerdings mathematisch unschön. Insofern ist der Mensch schon entscheidend, was wahr und was unwahr ist. Das Empfinden ist nichts Mystisches.

  39. #39 DeLuRo
    21. Juli 2014

    @RS: Präziser gesagt, wenn ich bei objektiven Werten bleibe und die Empfindung auch nichts anderes sagen sollte, gibt es keinen Konflikt, andernfalls schon (ein “kann” also). Reproduzierbare Werte sind aber maschinell messbare Werte, und das alleine als einzige Möglichkeit wird oft wegen der “Geistigkeit” des Menschen abgelehnt. Wer allerdings “geistig” über das maschinell Reproduzierbare hinausgeht, ist auch aus dem Bereich der Naturwissenschaften hinaus.

  40. #40 T
    21. Juli 2014

    @ RS

    Meßwerte gelangen immer an dem Menschen. Die Meßwerte werden empfunden. Wo ist da der Konflikt?

    Das erinnert mich an den Text von Peter Bichsel, in dem ein Mann die Namen der Dinge vertauscht und so den Kontakt zu den Menschen verliert. Inzwischen habe ich verstanden und empfinde es deutlich, dass es Ihnen vor allem um den Widerspruch geht. Das mag eine Weile unterhaltsam sein, aber inzwischen geht es mir auf die Nerven. Sie können freilich alles so benennen, wie es Ihnen beliebt, aber allein durch sprachliche Kreativität werden Sie nicht zu besseren Ergebnissen kommen als die Experten. Ich fürchte nur, dass Sie Experte für alles sind. Lassen Sie es doch einfach bleiben und gehen Sie ein bissl spazieren.

  41. #41 DeLuRo
    21. Juli 2014

    Danke @T. Das ist auch der typische Konflikt zwischen Natur- und Geisteswissenschaftlern. Man spricht aneinander vorbei, weil die Begrifflichkeiten auf verschiedenen Erklärungsebenen ansetzen und meistens auch verschiedene Konnotationen haben. Wie bei Physik und Metaphysik.

  42. #42 DeLuRo
    21. Juli 2014

    Es gibt noch eine weitergehende Begründung, warum die Zeitdimension objektiv nicht mit den drei Raumdimensionen gleichwertig ist. Das wird dann aber auch komplizierter. Wenn ich einen Gegenstand habe (der nirgendwo “festgeschraubt” sein sollte), dann kann ich ihn beliebig im Raum verdrehen. Und noch besser: wenn eine Kante in der einen Raumrichtung die Länge k hat, wird sie auch nach Verschieben und Verdrehen die Länge k haben. Das liegt daran, dass man unseren Raum auf kleineren Skalen als Raum mit Euklidischer Metrik annehmen kann, also konstant rechtwinklig (und falls das Universum flach ist, ist es überhaupt Euklidisch). Metrik bedeutet die Messbarkeit von Abständen.

    Eine Kante im Raum kann ich nicht und niemals in die Zeitdimension drehen; das widerspricht auch unseren Vorstellungen. Ich kann vielleicht gerade noch einen flachen Körper genau so konstruieren, passend definiert, dass es so aussieht als ob. Vor allem aber kann ich ihn nicht aus der Zeitdimension in den Raum drehen, denn sonst könnte ich aus dem Nichts Volumen erschaffen. — Derartiges Verdrehen, Verschieben und einige Operationen mehr kann man übrigens gut mathematisch als Transformationen auf Vektorräumen beschreiben (Vektorgeometrie).

    Die Verwendung einer 4-dimensionalen Raumzeit ergibt dabei weder nach physikalischen noch nach mathematischen Regeln einen Widerspruch, so dass man sich da nicht auf “Andersartigkeit” berufen könnte. Ich möchte keine Formeln hinschreiben, aber qualitativ ist die Metrik einer Raumzeit so formuliert, dass die Zeitkoordinate nach Quadrieren umgekehrtes Vorzeichen zu den 3 Raumkoordinaten hat, damit es physikalisch überhaupt funktioniert. Das erlaubt gerade noch, dass bei gleichbleibendem Zeitpunkt weiter beliebig zwischen Raumkoordinaten transformiert werden kann.

    Eine physikalisch korrekt durchführbare “Drehung” in die Raumzeit kann auch hier nicht passieren. Zwar lässt sich rein mathematisch eine Transformation beschreiben, die das tun könnte. Diese Transformation enthält aber imaginäre Komplexzahlen, was zur Folge hat, dass sie physikalisch nicht anwendbar ist. Eine physikalisch mögliche Transformation hat nur Realwerte (und ich spreche von Materie, nicht von EM-Feldern). — Also kann auch eine 4-dimensionale Raumzeit nicht begründen, dass Raumdimensionen und Zeit gleichwertig wären.

  43. #43 DeLuRo
    21. Juli 2014

    letzter Absatz: gemeint ist “…Drehung von einer Raum- in die Zeitkoordinate (und umgekehrt) kann auch hier nicht passieren…”

  44. #44 mr_mad_man
    21. Juli 2014

    Nach dem ersten Teil “Beobachtbares Universum” hatte ich schon ganz sehnsüchtig auf diesen zweiten Teil “Gesamt-Universum” gewartet, gefühlt alle fünf Minuten bei SB reingeguckt, und nun ist der Artikel endlich da. Freu und Danke.

    Die drei Analogien zu den möglichen Formen des Universums, über die man immer wieder in diesem Zusammenhang “stolpert”, finde ich sehr gut erklärt. Für mich ist es am einfachsten sich die positive Krümmung vorzustellen, so wie eine Hohlkugel, deren gegenüberliegende Punkte an den “Innenseiten” jeweils identisch sind. Aber geschenkt, die Beobachtungen deuten ja eher auf ein flaches Universum hin. Sich diese Form vorzustellen ist eigentlich auch nicht so schwierig, aber dummerweise kollidiert ein solches Universum mit der Vorstellung des Urknalls so wie er fast überall beschrieben und vor allem dargestellt wird. Dieses Bild von einem Universum in einem Punkt (oder wie es wissenschaftlich ausgedrückt wird Singularität), welches sich von diesem Punkt in alle Richtungen ausdehnt, hat sich so sehr in die Köpfe eingebrannt (ich will jetzt wirklich nicht von mir auf andere schließen, glaube aber, dass es vielen so geht), dass man es nicht revidieren will oder kann. Denn der Widerspruch zwischen Punkt/Singularität und “Überall” ist offensichtlich. Wenn ich es richtig verstehe fordert ein flaches Universum ein “Überall”-Urknall und eben kein “Punkt”-Urknall. Vielleicht wäre es an der Zeit nach geeigneten Darstellungen für den “Überall”-Urknall zu suchen (sicherlich schwieriger als eine schnöde Explosion) und diese auch in den populärwissenschaftlichen Büchern und Fernsehsendungen zu etablieren, damit sich das “falsche” Bild des “Punkt”-Urknalls nicht weiter verfestigt.

  45. #45 Steffmann
    21. Juli 2014

    @mr_mad_man:

    Wenn ich es richtig verstehe fordert ein flaches Universum ein “Überall”-Urknall und eben kein “Punkt”-Urknall. Vielleicht wäre es an der Zeit nach geeigneten Darstellungen für den “Überall”-Urknall zu suchen (sicherlich schwieriger als eine schnöde Explosion)

    Es braucht keine Suche nach der geeigneten Darstellung, da diese schon existiert. Und vermutlich, aber nicht gesichert, auch nachgewiesen ist => Inflation.
    Und wenn sich die Inflationstheorie bestätigt, brauchen wir auch nicht mehr ständig über das “grosse Wunder” der richtigen Verteilung der Naturkräfte diskutieren. Denn damit wäre klar, dass unser Universum und damit wir nur ein Zufallsprodukt sind.

  46. #46 Steffmann
    21. Juli 2014

    @mr_mad_man & all:

    Sorry, besser “ewige Inflation”.

  47. #47 Alderamin
    21. Juli 2014

    @Matthias Wolf

    @Alderamin (#19): ebenfalls danke. Das ist mir alles durchaus bekannt – ich sehe bloß nicht, was es mit dem Problem des ›gleichzeitigen‹ Inflationsereignisses zu tun hat.

    Hast Du auch die #8 gelesen? Daraus wird doch ganz offensichtlich, wie die Synchronität des Urknalls entsteht.

  48. #48 mr_mad_man
    21. Juli 2014

    @Steffmann: “Es braucht keine Suche nach der geeigneten Darstellung, da diese schon existiert. Und vermutlich, aber nicht gesichert, auch nachgewiesen ist => ewige Inflation.”

    Ja, das habe ich aus den vielen Kommentaren oben auch so herausgelesen. Mit geht es dabei darum das “visuell” darstellbar zu machen. Wie ich ja schon schrieb: eine schnöde Explosion, die sich von einem Punkt in alle Richtungen ausbreitet ist wohl das gänzlich falsche Bild (graphische Darstellung/verbale Beschreibung).

  49. #49 Steffmann
    21. Juli 2014

    mr_mad_man:

    Wie ich ja schon schrieb: eine schnöde Explosion, die sich von einem Punkt in alle Richtungen ausbreitet

    So kannst und darst du dir das nicht vorstellen. Nimm eine Folie und dehne diese bis es nicht mehr geht. Überall entsehen auf einmal Löcher. Das jeweilige Loch entsteht mehr oder weniger spontan. Und die Ausdehnung beschleunigt sich, sobald das Gefüge der Folie mal zerstört ist. Das trifft es aber nicht, trotzdem, nur mal als Alternative zur Vorstellung.
    Das Problem mit der Vorstellung ist, das wir nur 3-dimensional denken können. Die “Folie” ist aber multidimensional.

  50. #50 Matthias Wolf
    21. Juli 2014

    @Alderamin (#47): danke – muss ich im Detail erst verdauen. /Klingt/ grundvernünftig, aber ich weiß noch nicht, wie ich mir die Blase und das Außerhalb vorstellen muss (bzw. wo man das kosmologisch einordnen kann).

  51. #51 MK
    Baerlin
    21. Juli 2014

    Als Erstes: Ich habe absolut keine Ahnung von all dem was da kosmologisch ablaufen soll… nur interessiert mich dieses Thema.

    Bei der Lektuere hier, faellt mir eines spontan ein: “Nehmen wir eine beliebigen Samen, sagen wir den einer Eiche, also eine Eichel die noch, im Vergleich zu einem Senfsamen, relativ gross ist, aber in keiner Beziehung zu der Groesse des daraus entstehenden Baumes, seiner Fruechte die diese baum in seiner Wachstumsperiode erzeugen wird, also unzaehlige Eicheln, die widerum unzaehlige Baeume von schwer abschaetzbarer Groesses hervorbringen koennen, was auch geschieht, allerdings voellig unabhaengig von dem Baum, der Frucht, sondern von Umstaenden die absolut nichts mit dem Baum oder der Frucht zu tun haben! – Na klingelt was?

  52. #52 Steffmann
    21. Juli 2014

    @MK.

    Und angenommen, alles was du weisst, reicht für die richtige Einschätzung des Sachverhaltes nicht aus ?

  53. #53 mr_mad_man
    21. Juli 2014

    @Steffmann: “So kannst und darst du dir das nicht vorstellen. Nimm eine Folie und dehne diese…”

    Danke, ich finde das sehr anschaulich. Was jetzt noch fehlt, ist in Fernsehsenungen etc. die Explosion durch die Folie zu ersetzen.

  54. #55 Peroppi
    21. Juli 2014

    Danke für diese Artikelserie. Man sieht anhand der vielen Kommentare, dass die Fragen tatsächlich passend sind und viele Leute interessieren, mich inklusive 🙂

    Geht man eigentlich davon aus, dass der Raum beim Urknall, also sozusagen beim Zeitpunkt 0, unendlich dicht komprimiert war? Oder ist es auch möglich, dass der Raum nur sehr sehr dicht, aber nicht unendlich dicht, war? Wenn ich den Artikel richtig verstanden habe, macht das für die Größe des Universums beim Urknall (vermutlich) keinen Unterschied – in beiden Fällen war es (möglicherweise) bereits unendlich groß.

    Interessant ist auch, dass das Universum räumlich betrachtet (vermutlich) keinen “Rand” hat, egal ob es nun gekrümmt ist oder nicht. Aber dass es wohl zeitlich gesehen einen Anfang hat, was man doch in der zeitdimension als “Rand” ansehen kann?

  55. #56 Alderamin
    21. Juli 2014

    @Matthias Wolf

    wie ich mir die Blase und das Außerhalb vorstellen muss (bzw. wo man das kosmologisch einordnen kann).

    Von außen besehen ist das Außerhalb einfach der umgebende Raum, eine Zone drumherum mit anderer Vakuumenergie. Man könnte sich vorstellen, dass auch in unserem Universum noch nicht das absolute Energieminimum gegeben ist und sich spontan irgendwo eine Zone darin bildet, die von einem Punkt aus expandierend eine Blase mit noch niedrigerer Energie erzeugt, die sich mit Lichtgeschwindigkeit ausdehnt (google mal “Vakuumzerfall” oder “vacuum decay”).

    Von innen besehen ist es das (zeitliche) Davor. An der Grenze kippt der Zeitbegriff.

    Ähnliches soll auch in Schwarzen Löchern am Ereignishorizont passieren, da werde der Radius zur Zeitrichtung, las ich mal. Es geht von dort an nur noch in eine Richtung: nach innen.

  56. #57 Steffmann
    21. Juli 2014

    @mr_mad_man

    Danke, ich finde das sehr anschaulich. Was jetzt noch fehlt, ist in Fernsehsenungen etc. die Explosion durch die Folie zu ersetzen.

    Nicht dein Ernst oder ? Dir ist schon klar, dass ich hier Zeit in dich investiere ? Und deinen Kommentaren nach zu urteilen, kannst du ne Acht von ner Neun unterscheiden.

  57. #58 PDP10
    21. Juli 2014

    @Steffmann:

    Was is’n dir über die Leber gelaufen?

    @mr_mad_man hat doch schon in in #48 geschrieben, dass eine Explosion, die sich von einem Punkt in alle Richtungen ausbreitet, offenbar das falsche Bild ist (den Teil hast du in deinem Zitat “rausgeschnitten”) … und jetzt versucht er halt eine sinnvolle Visualisierung zu finden.
    Was daran so schrecklich ist, dass du ihn so angehen musst erschliesst sich mir ehrlich gesagt nicht …

    Ach so, und @MK hat offenbar einfach nur kürzlich “Asterix und die Trabantenstadt” gelesen … 😉

  58. #59 Steffmann
    21. Juli 2014

    @PDP10:

    Oh je, ich bin grade vom 80 h- Marathon Arbeit gekommen und dachte, ich hätte die Diskussion geblickt.

    Sorry, das war aber diesesmal auch nciht so einfach zu durchschauen. Nichts für ungut, mangelnde Sorgfalt meinerseits.

  59. #60 mr_mad_man
    21. Juli 2014

    @Steffmann: “Nicht dein Ernst oder ? Dir ist schon klar, dass ich hier Zeit in dich investiere ?”

    Entschuldige bitte, jetzt bin ich echt verwirrt. Was habe ich falsch gemacht? Ja das ist mir klar, und ich bin für jeden Beitrag sehr dankbar, der mein Verständnis von Universum und Urknall erweitert.

    Ich möchte gerne Deinen “Groll” entschärfen (da das wirklich nicht meine Absicht war) und die Ausgangslage meines Gedankenganges noch mal kurz zusammenfassen:

    1. Die Standard-Darstellung des Urknalls (TV etc.) ist eine Explosion die von einem Punkt ausgeht.
    2. Stand der Beobachtungen ist, dass das Universum flach ist
    3. Die Beschreibung eines Urknalls wie in 1. (aus einem Punkt) beschrieben passt nicht zum flachen Universum. Der Urknall war “überall” (überall ungleich Punkt)
    4. Die Standard-Urknall-Darstellung (Punkt) muss durch eine andere ersetzt werden
    5. Du (und andere Kommentare) erklären als Alternative die ewige Inflation. Du präsentierst das anschauliche Bild der Folie.
    6. Wenn der Stand des Wissens von einem flachen Universum ausgeht, muss die Beschreibung des Urknalls dazu passen, also die Folie.

    Ich hoffe diese Zusammenfassung kann helfen, den Punkt des Anstoßes oder evtl. meines Denkfehler ausfindig zu machen. Wenn ich Dich verärgert habe, bitte ich um Entschuldigung, denn ich bin nach wie vor ernsthaft daran interessiert diese Themen und Fragen für mich zu beantworten. Vielleicht stellt man als Laie manchmal so dämliche Fragen oder tätigt Aussagen, die anderen als blanke Provokation vorkommen. Aber wie gesagt, das ist nicht meine Absicht.

  60. #61 Steffmann
    21. Juli 2014

    @mr_mad_man:

    Sorry……

  61. #62 PDP10
    21. Juli 2014

    @Steffmann:

    “Oh je, ich bin grade vom 80 h- Marathon Arbeit gekommen und dachte, ich hätte die Diskussion geblickt. “

    Mut je nich dun!

    Hab mir schon sowas gedacht .. du bist ja sonst nicht so … 🙂

  62. #63 Alderamin
    22. Juli 2014

    @Peroppi

    Geht man eigentlich davon aus, dass der Raum beim Urknall, also sozusagen beim Zeitpunkt 0, unendlich dicht komprimiert war? Oder ist es auch möglich, dass der Raum nur sehr sehr dicht, aber nicht unendlich dicht, war?

    Um die Inflation zur Entstehung eines Universums anzustoßen, braucht es nach Alan Guth, einem der Väter der Inflationstheorie, nur ca. 30 g Energie-Masse-Äquivalent in einem Volumen der Seitenlänge 10^-26 cm. Das bedeutet dann allerdings die phantastische Dichte von 10^80 g/cm³. Das ist so dicht, dass bei dieser Dichte die Masse des gesamten beobachtbaren Universums in ein Atom passen würde. Ist aber deutlich weniger als unendlich und deutlich größer als ein Punkt.

    Möglicherweise könnte ein Schwarzes Loch so etwas leisten.

  63. #64 Steffmann
    22. Juli 2014

    @PDP10:

    *ein Freund, ein guter Freund, das ist das beste was es gibt auf der Welt *……..Danke dir. Und gute Nacht 😉

  64. #65 PDP10
    22. Juli 2014

    @Alderamin:

    “Man könnte sich vorstellen, dass auch in unserem Universum noch nicht das absolute Energieminimum gegeben ist und sich spontan irgendwo eine Zone darin bildet, die von einem Punkt aus expandierend eine Blase mit noch niedrigerer Energie erzeugt, die sich mit Lichtgeschwindigkeit ausdehnt [..]”

    Ich dachte, die Idee hätte sich mittlerweile erledigt?

    Wegen eines Rechenfehlers, oder wie war das?

    Blöderweise finde ich grade die Quelle nicht, aber ich meine es bei der BDW online gelesen zu haben und da verschwinden leider fast alle Artikel nach einiger Zeit hinter einer Paywall …

  65. #66 sax
    22. Juli 2014

    Noch ein Kommentar zu #30

    Nur sprechen da halt derzeit die Messungen dagegen. Wenn, dann muss es WIRKLICH groß sein, so groß, damit uns die positive Krümmung nicht auffällt, weil sie so schwach ist.

    @Rolf Schälike: “Das Universum ist mindestens vierdeminsional. Die Zeit als vierte Dimension sollte nie vergessen werden. “

    Da es hier aber um die räumliche Ausdehnung geht, spielt die Zeit keine große Rolle…

    War nicht gerade genau das ein Grund für die Einführung der Inflationshyphothese, dass das Universum so aufgebläht wurde, dass der beobachtbare Teil davon so winzig im Vergleich zum ganzen ist und deshalb eine evt. vorhandene Krümmung nicht mehr auffällt.?

  66. #67 Matthias Wolf
    22. Juli 2014

    Noch eine Frage: Wenn das Universum aus einem Punkt entsteht, verstehe ich, wieso sich ein ›beobachtbarer Teil‹ heraus bildet, und wieso es Teile gibt, deren Licht uns noch nicht erreichen konnte.

    Unklar ist mir aber, wieso eine (wie auch immer gefeuerte) Inflation eines ›unendlich großen Gummituchs‹ den Beginn der Zeit markieren würde (was ja notwendige Voraussetzung zu sein scheint, dass es diese Grenze geben kann).

    Kann jemand dazu etwas sagen?

  67. #68 Hans
    22. Juli 2014

    Ich kann mir nicht helfen, aber diese Erklärung zum unendlich grossen aber flachen Universum erinnert mich an den Versuch zu erklären, was “Ewigkeit” ist, bzw. sein soll. Da hapert es ja auch am Vorstellungsvermögen und die Analogien sind meisst auch irgendwie Krüken, da man immer irgendwo auf Widersprüche stösst.

    Nebenbei, aber nur just for fun, da “esotherische Quelle“: Das Universum soll Eiförmig sein, also in etwa der Form eines Hühnereis entsprechen. Man hätte also immer noch eine positive Krümmung, wenn ich das mit der Krümmung richtig verstehe, nur eben keine Kugelform, sondern ein mathematisch schwieriger (umständlicher?) zu beschreibendes Oval bzw. Ovoid.

  68. #69 Florian Freistetter
    22. Juli 2014

    @Peroppi: “Oder ist es auch möglich, dass der Raum nur sehr sehr dicht, aber nicht unendlich dicht, war?”

    Naja, da “unendlich dicht” in der realen Welt nicht existieren kann, geht man davon aus, dass das nicht der Fall war. Aber was der Fall war, weiß man leider noch nicht so wirklich…

  69. #70 Alderamin
    22. Juli 2014

    @PDP10

    Eher im Gegenteil. Die vor kurzem erst bestimmte Masse des Higgs-Teilchens deutet darauf hin, dass unser Vakuum metastabil ist. Siehe https://en.wikipedia.org/wiki/False_vacuum#Stability_and_instability_of_the_vacuum

  70. #71 Alderamin
    22. Juli 2014

    @Matthias Wolf

    Noch eine Frage: Wenn das Universum aus einem Punkt entsteht, verstehe ich, wieso sich ein ›beobachtbarer Teil‹ heraus bildet, und wieso es Teile gibt, deren Licht uns noch nicht erreichen konnte.

    Weil die Inflation (nachdem das Universum bereits kein Punkt mehr war; war es wohl auch nie) Orte so schnell und so weit getrennt hat, dass das Licht nicht mithalten konnte. Die Inflation vergrößerte den Raum um den Faktor 10^25 in jeder Raumdimension, und das in nur 10^-33 Sekunden.

    In der Hintergrundstrahlung kann man an den Korrelationen der Feinstruktur über den Winkelabstand erkennen, dass sich Abhängigkeiten nur über einen Winkelabstand ergeben, der einer Lichtlaufzeit von 380000 Jahren entspricht. Das ist der Horizont zur damaligen Zeit. Heute ist unser Horizont auf 13,8 Milliarden Jahre Lichtlaufzeit gewachsen. Die Orte, von denen dieses Licht stammt, sind aber durch die normale Raumausdehnung mittlerweile schon 43 Milliarden Lichtjahre entfernt, d.h. sie entfliehen mit mehr als Lichtgeschwindigkeit (in Wahrheit ruhen sie jedoch an ihren Orten, nur der Raum zwischen ihnen und uns quillt auf). Deswegen wird uns zukünftig kein Licht mehr von ihnen erreichen.

    Unklar ist mir aber, wieso eine (wie auch immer gefeuerte) Inflation eines ›unendlich großen Gummituchs‹ den Beginn der Zeit markieren würde (was ja notwendige Voraussetzung zu sein scheint, dass es diese Grenze geben kann).

    Das kommt auf die Variante des Urknall-Modells an. Ursprünglich sah man keine Chance, jemals etwas über das Davor zu erfahren, und da war die verbreiteteste Ansicht, Zeit und Raum entstanden beide mit dem Urknall. Dann fing das Universum wohl mit endlicher Größe bzw als Punkt an. Diese Variante ist auch immer noch nicht widerlegt. Zeit und Raum könnten auch wie ein starrer Block “in der Gegend herumhängen” und unser Zeitgefühl nur eine Illusion sein, die die Richtung zunehmender Entropie anzeigt (weil das Denken und Fühlen auf Prozessen beruht, die die Entropie erhöhen).

    Bei der ewigen Inflation mit Blasenuniversen gibt es zwar schon außerhalb der Blasen eine Zeit, aber es fängt in einer Blase eine andere Zeitrichtung an, wie ich in #8 verlinkt hatte. Die ewige Inflation könnte ihrerseits einen Anfang gehabt haben und einem spontan entstandenen Planck-Raumzeitvolumen entsprungen sein. Dann gab es eben kein Davor.

  71. #72 swage
    23. Juli 2014

    Ist eine interessante Frage. Rein von den Beobachtungen her kann man die Frage nicht klären und ich wüsste auch keinen Ansatz um das in Zukunft zu klären,

    aber:

    man kann ein logisches Argument anführen. Es wurde in einer der neuen Cosmos Folgen angesprochen. Das Argument lautet wie folgt:

    Wenn ich einen Pfeil an den Rand des Universums schieße (also rein hypothetisch) und er trifft auf eine Barriere (sprich: den Rand), so gibt es auch ein “hinter der Barriere”, sonst würde diese nicht existieren. Existiert sie aber nicht, so gibt es keine Barriere und folglich keine Grenze.

    Wenn wir nun annehmen dass das Universum auf eine Folge von Quantenfluktuationene zurükzuführen ist, die auch dort zu finden sind wo sich “nichts” oder “noch nichts” befindet, so ist dies gleichzusetzen mit unendlicher Raum für potentielle Entstehung von Universen.

    Deswegen würde ich für “unendlich” plädieren. Es geht eigentlich gar nicht anders (das wäre dann die Multiversum Hypothese). Es bedeutet das auch das mehrere BB Events gibt und zwar in Beiden Zeitrichtungen.

    Das würde auch die Problematik der frühen supermassereichen Objekte und das Gravitationsproblem zur Galaxienentstehung lösen. Es könnte sein das die Umgebung gar nicht mal so leer war, wie allgemein angenommen wird.

    Natürlich… sind die Galaxien immer noch zu schwer und wir haben immer noch keine Idee was die Ausdehnung des Raumes verursacht (die könnte aber etwas damit zu tun haben wie sich umliegende Universen gegenseitig beeinflussen, auch das ist nicht unbedingt eine neue Idee).

  72. #73 Kalony
    23. Juli 2014

    Bei der Gelegenheit dieses natürlich ausgesprochen interessantenThemas, möchte ich nur mal Hallo sagen und das ich seit einiger Zeit diesen Blog mitlese und mich sehr über das Engagement und über die verständliche Art und Weise freue…auch wenn ich, wie alle anderen gestehen muss, das zum Beispiel diese 3 Theorien hier nicht wirklich vorstellbar sind. Vor allem Theorie 2 und 3.

    Also Dank an den Schreiberling und all die anderen qualifizierten Diskutanten

  73. #74 swage
    23. Juli 2014

    Ich denke an 3. führt gar kein Weg vorbei. Wäre dem nicht so, also Urknall überall, müsste er von einem Punkt oder im Falle eines Multiversums eben Punkten ausgehen.

    Das bedeutet aber wiederum, das, aufgrund der Rotverschiebung, sich die Erde mehr oder weniger im Zentrum des Urknalls befinden muss.

    Ich mag Modelle nicht die auf einem Zentralen Dreh- und Angelpunkt beruhen, weil… das haben wir immer und immer wieder gemacht und immer und immer wieder hat es sich als falsch erwiesen (Erde im Zentrum, Sonne im Zentrum, Galaxie im Zentrum, etc).

    Und zweitens sind speziell die Modelle extrem suspekt, die gerade uns ins Zentrum rücken. Die Wahrscheinlichkeit dafür das ein derartiges Modell den Tatsachen entsprechen könnte halte ich für extrem niedrig.

    Also würde ich eher die Modelle in Betracht ziehen, die es zulassen, das die selbe Beobachtung von jedem beliebigen Punkt im Universum gemacht werden kann.

    Wenn dem aber so ist, was ich für realistischer halte, dann kann sich das Universum aber nicht von einem Punkt ausdehnen, sondern quasi überall.

    Ich könnte mir das Ganze in etwas so vorstellen: am Anfang haben wir nahezu perfektes Vakuum.

    Zeitdilatation erreicht man einerseits durch Geschwindigkeit (SRT), aber andererseits auch durch Masse (ART), was sich dann gravitative Zeitdilatation nennt. Und das funktioniert so:

    Wenn ich einem schweren Objekt nahe bin vergeht die Zeit langsamer relativ zu Regionen mit niedrigerem Gravitationspotential langsamer.

    Wenn es jetzt aber (fast?) keine Masse gibt… werden all diese Quanteneffekte augenblicklich in einen unendlich kleinen Augenblick “gequetscht”.

    Wie oben erwähnt mag ich keine Modelle die irgendetwas, insbesondere uns selbst in den Mittelpunkt rücken. Und das ist nicht nur räumlich, sondern auch zeitlich zu verstehen.

    Wenn man die kosmische Expansion mal weiter führt, so bedeutet das im Grunde genommen, das sich das Verhältnis von Raum zu Materie (wieder?) immer mehr in Richtung Vakuum verschiebt und sich somit wieder dem Zustand der Ausgangssituation angleicht.

  74. #75 Toddy
    23. Juli 2014

    Da hier öfters die Frage nach der passenden Visualisierung gefragt wird…

    Ich für meinen Teil habe die befriedigendste Darstellung in der, der zwei kollidierenden Branen gefunden.
    Das erklärt für mich sehr einleutend den Sachverhalt des “Überall-Big Bangs”.

    Hier eine von unzähligen Darstellungen:

    https://media.radiosai.org/journals/Vol_06/01APR08/images/04_FeatureArticles/SFI/collision.jpg

  75. #76 Alderamin
    23. Juli 2014

    @Toddy

    Wieso sollte in der Realität die andrer Brane so eben (ist sie im Bild auch gar nicht, das würde man in der Hintergrundstrahlung nach der Kollision auch sehen) und genau parallel zu unserer sein?

    Und siehe #8…

  76. #77 sax
    23. Juli 2014

    @Swage:
    “Das bedeutet aber wiederum, das, aufgrund der Rotverschiebung, sich die Erde mehr oder weniger im Zentrum des Urknalls befinden muss.”

    Nein, auch wenn das Universum positive Krümmung hatte und direkt nach dem Urknall also wirklich so etwas wie fast ein Punkt gewesen ist, ist jeder Punkt im Universum gleichberechtigt.

    Um es mit der 3D-Kugelanalogie zu sagen, das Universum wäre die Oberfläche der Kugel, da ist kein Punkt besser als der andere. Der Mittelpunkt der Kugel ist gar kein Teil des Universums.

    Den Mittelpunkt muss es so auch gar nicht geben. Die Oberfläche einer 4-D-Kugel ist zwar ein 3-D Raum positiver Krümmung. Das ein 3-D Raum gekrümmt ist, setzt aber nicht die Existenz einer vierten Dimension heraus.

    (Die Zeit ist zwar eine vierte Dimension, aber in der Robertson-Walter Metrik ist es eben nicht so, dass wir eine Kugel in der Vierdimensionalen Raumzeit haben, sondern einen 3D unterraum und die Zeit, wobei der 3D Raum in sich gekrümmt ist. )

    Also würde ich eher die Modelle in Betracht ziehen, die es zulassen, das die selbe Beobachtung von jedem beliebigen Punkt im Universum gemacht werden kann.

    Das machen alle seriöusen kosmologischen Modelle, die Grundannahmen von denen man ausgeht sind in jedem Fall, das Universum ist homogen (an jedem Punkt gleich) und isotrop (keine Richtung ist ausgezeichnet). Das nennt man das schwache Kosmologische Prinzip. Früher ging man noch davon aus, dass das Universum zu jeder Zeit gleich war, aber dieses “starke kosmologische Prinzip” widersprach der Beobachtung und wurde deshalb aufgegeben.

    @Hans
    “Nebenbei, aber nur just for fun, da “esotherische Quelle“: Das Universum soll Eiförmig sein, also in etwa der Form eines Hühnereis entsprechen. Man hätte also immer noch eine positive Krümmung, wenn ich das mit der Krümmung richtig verstehe, nur eben keine Kugelform, sondern ein mathematisch schwieriger (umständlicher?) zu beschreibendes Oval bzw. Ovoid.”

    Nein, wenn das Universum Eiförmig wäre, wäre die Krümmung bzw. der Krümmungstensor nicht überall konstant, da dien Krümmung eine Funktion der Energie-Impuls dichte ist, wäre diese nicht überall im Unviersum gleich*. Wäre das Universum “Eiförmig” wäre das schwache kosmologische Prinzip falsch, die Datenlage spricht aber eher dafür, das es richtig ist.

    *mit überall gleich meine ich das man kleine Störungen, wie Galixien und den Raum dazischen, wirklich als klein und damit nur als kleine korrektur zu dem homogenen Universum, betrachten kann.

  77. #78 Hans
    23. Juli 2014

    @sax. #77:
    Hm… ja okay, dass die Krümmung nicht überall gleich ist, kann man auch an einem gewöhnlichem Ei feststellen. Aber das ist ein Detail, um dass es mir gar nicht geht. Die Frage ist doch, ob die Krümmung des Universums positiv oder negativ oder “Null” ist? – Und bei einer Eiform, die man auch als stark verbeulte Kugel betrachten kann, müsste immer noch eine positive Krümmung vorliegen, wenn ich es richtig verstanden habe. Denn wenn man auf dem Ei in einer bestimmten Richtung immer geradeaus wandert kommt man ja auch irgendwann wieder an der Stelle an, wo man los ging.

  78. #79 sax
    23. Juli 2014

    Ja, da hast du recht.

  79. #80 Toddy
    23. Juli 2014

    @Alderamin

    Das hier zwei Flächen parallel aufeinanderklatschen ist wahrscheinlich der Darstellung per se geschuldet.

    In meiner Phantasie ist das durchaus nicht so 😉

    Letztenendes versucht man ja vereinfacht, das “Aufeinandertreffen” zweier drei?(raum)dimensionaler Universen darzustellen.

    Nun denn…zwei vakuum-energetische Sphären “vereinen” sich in einem zeitlosen, ?-dimensionalem Irgendwo und initiieren den Großen Massenauswurf.

    Klingt wie Sci-Fi – könnte aber so passiert sein.

    Ich bin jetzt nicht der große Mathematiker, weiss daher nicht ob es in der Beschreibung von höherdimensionalen Räumen ein “parallel” gibt, bzw. geben muss.

    Klar kann man anführen, dass es zur Erklärung des Universums nicht mehr als drei Raumdimensionen bedarf.
    Andererseits – warum sollte es sie nicht geben?
    Passt für mich auch schön zum Modell, welches die relativ schwache Gravitation erklären kann.

  80. #81 Alderamin
    23. Juli 2014

    @Toddy

    Mein Punkt war eigentlich nur, dass die Kollision zweier Branes nicht unbedingt das Synchronitätsproblem löst. Dazu müssten sie eben und exakt parallel sein. Warum sollten sie aber?

    Parallelität gibt es selbstverständlich auch in höherern Dimensionen. Man kann ja einen Abstand Raum mit der höheren Dimension definieren, und der muss dann an jedem Punkt zwischen den Branes gleich groß sein.

  81. #82 Hering
    24. Juli 2014

    Spezifiziert mal genauer was es bedeutet: das Universum ist unendlich groß . Was davon ist unendlich groß? Die räumliche Ausdehnung? Die zeitliche Ausdehnung? Deuten unendliche Größen in der Physik nicht auf Probleme in der Theorie?

  82. #83 Alderamin
    24. Juli 2014

    @Hering

    Was davon ist unendlich groß? Die räumliche Ausdehnung?

    Ja.

    Die zeitliche Ausdehnung?

    Ja.

    Deuten unendliche Größen in der Physik nicht auf Probleme in der Theorie?

    Nein, wieso? Ein Problem in der Theorie hat man nur, wenn eine Größe gegen unendlich geht in Abhängigkeit von einem Parameter, der selbst nur gegen einen endlichen Wert läuft, etwa die Schwerkraft in Abhängigkeit vom Radius (gegen 0 laufend) bei einem schwarzen Loch. Aber nicht, wenn sie gegen unendlich geht, während es der Parameter ebenfalls tut, wie z.B. der Durchmesser des Universums in Abhängigkeit von der Zeit. Lineare oder einfache polynomielle Abhängigkeiten sind kein Problem.

  83. #84 swage
    26. Juli 2014

    Um es mit der 3D-Kugelanalogie zu sagen, das Universum wäre die Oberfläche der Kugel, da ist kein Punkt besser als der andere. Der Mittelpunkt der Kugel ist gar kein Teil des Universums.

    Hmm, ja, aber da wir normalerweise von 3 geometrischen Dimensionen sprechen kommst du nur zu 4 dimensionalen geometrischen Modellen nur wenn du eine Dimension “temporär wegfallen lässt”. Minkowski Raumzeit, wenn mich nicht alles täuscht.

    Das sind nicht-euklidische Koordinaten (und daher sehr richtig nicht nicht Teil des Universums). Ich habe so meine Probleme mit einem Urknall im… Hyperraum. Auch Stringtheorie macht mir aus dem selben Grund Schwierigkeiten. Zunächst mal hätte ich ganz gerne einen Nachweis das dieser existiert. Mathematisch ist das alles kein Problem, aber…

  84. #85 Stefan
    27. Juli 2014

    @Alderamin: “Lies Dir das mal durch, dann sollte klar sein, wie man aus unendlicher Zeit unendlichen Raum machen kann: https://edge.org/conversation/next-step-infinity

    String-Theorie ist dabei vorrausgesetzt. Und die ist noch immer nach Jahrzehnten nur eine mathematische Spielerei. Ergebnisse in Cern deuten eher auf eine Falsifizierung der String-Theorie hin. Der Bs-Mesonen-Terfall, in zwei Myonen (Bs-Mesonen zerfielen in zwei Myonen) bringt die Supersymmetrie ganz ordentlich ins wanken udn damit logsicherweise die String-Theorie. Soll mir recht sein, nach jahrzehntelangen Beschwören, dass die String-Theorie der Wunderwuzzi der Physik ist, wo viel Geld reingesteckt wurde, ohne nennenswerte Erkenntnise (im wissenschaftlichen Sinn), wirds Zeit die Theorie grundlegend zu hinterfragen.

    Und da gibts mit der loop quantum gravity einen mehr als guten Ersatz, die auch schon falsifizierbare Aussagen tätigt, die auch schon überprüft wurden. So Laufzeitunterschiede von Licht: https://arxiv.org/PS_cache/arxiv/pdf/0708/0708.2889v3.pdf und eine passende Erklärung warum uns Protonen mit über 10’9 ev ereichen. Mit der Loop Quantum Cosmology gibts auch eine Erklärung für Urknall, usw., den Big Bounce.

    Leider wird noch immer allzu sehr die String-Theorie gehyped und das obwohl sie – entgegen den Versprechungen der Vertreter – keine falsifizierbaren Aussagen tätig – zumindestens nicht solche, die in absehbarer Zeit nachprüfbar sind bzw. die eindeutig sind (also solche Aussagen, die nicht automatisch zu ad-hoc-Thesen führen, wie “gut, wenn keine Schwarzen Löcher im Cern gefunden werden, dann ist halt eines der anderen Modelle der String-Theorie richtig”).

    Sorry, fürs auskotzen, aber ich kann schon ncihts von der STring-Theorie hören. 😉

  85. #86 Alderamin
    27. Juli 2014

    @Stefan

    String-Theorie ist dabei vorrausgesetzt.

    Das ist falsch. Nur ewige Inflation wird vorausgesetzt. Die ist, laut Text, für die String-Theorie erforderlich. Das Gegenteil davon ist nicht, dass im Falle, dass die String-Theorie falsch ist, die ewige Inflation nicht stattfindet. Tatsächlich steht im Text:

    As a recent and impressive example, seven years of data from the Wilkinson Microwave Anisotropy Probe—which looks at the Big Bang’s omnipresent remnant radiation, known as the cosmic microwave background—has confirmed in great detail inflation’s predictions for the particular type of nonuniformities in the early universe.

    Dazu kommt, dass auch die BICEP2-Messungen auf ewige Inflation hindeuten.

    Du kannst Dir Deinen Rant gegen die String-Theorie also sparen. Ewige Inflation kann es mit oder ohne String-Theorie geben.

  86. #87 Niels
    27. Juli 2014

    @Stefan
    Nö, dabei wird nur Inflation vorausgesetzt. Die String-Theorie taucht nur als zusätzliches Argument für das wahrscheinliche Vorliegen von Eternal inflation auf.
    Der Teil, um den es Alderamin geht, hat jedenfalls mit String-Theorie überhaupt nichts zu tun.

  87. #88 StefanL
    28. Juli 2014

    @Stefan
    vom Fermi gamma ray telescope https://apps3.aps.org/aps/units/maspg/thompson.pdf pg 39ff

  88. #89 Stefan
    29. Juli 2014

    @Alderamin: Kannst du dir dagegen deine Aggressivtät sparen? Danke. Ich habe was gegen die Strign-Theorie geschrieben, nicht gegen dich und du wirst persönlich (“Ran”).

    “Ewige Inflation”, wie sie hier verstanden wird, ist Wesensbestandteil der String Theorie und ist nicht zu verwechseln mit Lindes Thesen dazu. Aguirre schreibt das auch basierend auf die Vorstellungen der STring Theorie. Aguirre leitet ebenso das “Foundational Questions program”, ein dick gefördertes Programm für die String Theorie. Und die BICEP-Daten weisen – mehr noch nicht (!), warten wir erstmal ab – ind ie Richtung von Linde. Das hängt aber alles nur bedingt mit Aguirres Thesen zusammen.

  89. #90 Alderamin
    29. Juli 2014

    @Stefan

    Sorry wenn’s zu aggressiv rüberkam, ich bin da zu sehr auf das “Auskotzen” angeprungen.

    Es mag ja sein, dass Aguirre ein glühender String-Theoretiker ist (ich bin übrigens keineswegs ein Anhänger dieser Theorie, die ist mir durchaus suspekt).

    Aber die ewige Inflation taucht schon bei Guth in seinem Buch “The Inflationary Universe” auf, das stammt aus den frühen 80ern, wenn ich mich recht erinnere. Darin ist von der String-Theorie keine Rede, sondern nur von der Beinahe-Unmöglichkeit, dass ein so schnell expandierender Raum überall gleichzeitig aufhört zu expandieren, wenn dabei das Inflaton-Feld abrupt auf einen niedrigeren Zustand tunnelt.

    Und ich hab Aguirre deswegen hier zitiert, weil er erklärt, wie ein unendliches Universum aus einer Inflationsphase hervorgehen kann. Das heißt ja nicht, dass es so passiert sein muss. Diese ganze Multiversums-Geschichte ist doch höchst hypothetisch, da gibt’s verschiedene Ideen und keine davon kann bisher bewiesen werden.

    Jedenfalls ist die Schlussfolgerung, dass aus einer möglicherweise nicht korrekten String-Theorie folgen würde, dass es keine ewige Inflation gibt, nicht zulässig. Die hängt vielmehr damit zusammen, auf welche Weise das Inflaton-Feld seinen hochenergetischen Zustand verlässt (abrupt oder “gracefully”). Und es ist ja noch gar nicht klar, wie die Vakuumenergie überhaupt funktioniert, sie liefert uns ja im Moment “the worst prediction of all physics”, wie Lawrence Krauss immer sagt.

  90. #91 Stefan
    31. Juli 2014

    Ich kann zu diesem Thema die Bücher von Brian Greene sehr empfehlen. “Verstehen” Ist bei Thema Unendlichkeit und bei 3 Dreidimensionalen Räumen und Milliarden von Lichtjahren sehr hoch gegriffen, aber nach der Lektüre hatte ich eine etwas besser Vorstellung. Florian, ich finde deine Erklärungsversuche sehr gut, aber diese Komplexe Thema ist schwierig verständlich ohne die Vorgeschichte von Urknall und (wahrscheinlicher) Inflationärer Theorie zu erläutern….

  91. #92 Alderamin
    1. August 2014

    @Stefan

    Ich kann zu diesem Thema die Bücher von Brian Greene sehr empfehlen

    Die habe ich alle gelesen. Schließe mich Deiner Empfehlung an. Außer vielleicht beim “Eleganten Universum”, da geht’s zu sehr ins Eingemachte, wird dann nachher auch etwas langatmig.

  92. […] an Florians Beitrag zur Frage, wie groß das Universum ist, gibt es heute Auskunft darüber, ob es unendlich groß ist. Soviel vorweg: Es ist flach. Wahrscheinlich. Und damit unendlich […]

  93. #94 bfluer99
    8. September 2014

    Das universum MUSS endlich sein. Wenn man denkt: beim Urknall hätte sich die Masse die dabei entstanden ist unendlich ausgebreitet! Dh. Dass diese ganze masse immernoch “unterwegs” ist, und dann wäre die zahl der Atome so klein dass kein feststoff entstehen kann

  94. #95 denker
    3. Dezember 2014

    Man könnte sich das alles so vorstellen: Wenn man zwei große Spiegel hätte und einen auf die linke und den anderen auf die rechte Seite vom Kopf halten würde, würde man, wenn man in eines der Spiegel schaut, sich unendlich wiederholend sehen. D.h. das Universum ist zwar nicht unendlich groß, aber man kann nie einen Rand erreichen, da man gegen Ende des leeren Raums aufgrund der starken Raumkrümmung wieder zu dem Punkt zurückkommen würde, von dem man angefangen hat… Also das Universum als in sich geschlossenes schwarzes Loch, aus dem kein Entkommen möglich ist. So stelle ich es mir vor….

  95. #96 rainer rupp
    München
    31. März 2015

    Man kann jeden der behauptet dasUniversum wäre nicht unendlich mit einer einfachen Kinderfrage ausbremsen.
    Nicht mit “Sind wir schon da?” Sondern “Was kommt dahinter?”
    Selbst Hochproffesoren können darauf keine Antwort geben die Leute, die was anderes behaupten, als dass das Weltall unendlich gross ist, reihen sich meiner Meinung nach in eine Reihe berühmter Wissentschaftler ein und zwar in eine der Wissenschaftler, die einst berechneten die Erde ist flach und die Sonne umkreist die Erde 😀

  96. #97 Florian Freistetter
    31. März 2015

    @rainer rupp: “Selbst Hochproffesoren können darauf keine Antwort geben die Leute, die was anderes behaupten, als dass das Weltall unendlich gross ist, reihen sich meiner Meinung nach in eine Reihe berühmter Wissentschaftler ein und zwar in eine der Wissenschaftler, die einst berechneten die Erde ist flach und die Sonne umkreist die Erde”

    Damit man eine Frage sinnvoll beantworten kann, muss die Frage selbst sinnvoll sein. Beantworte mir doch mal die Frage: Was liegt nördlich des Nordpols? Kannst du nicht? Dann musst du aber wohl genau so dumm sein wie die “Hochprofessoren” die du hier kritisierst…

  97. #98 PDP10
    31. März 2015

    Die gleiche Frage könnte man im Übrigen zu jedem Ort auf der Erde stellen: “Was kommt dahinter?” Der nächste Ort natürlich.

    Dummerweise ist die Erdoberfläche trotzdem nicht unendlich Groß.

    Und was bitte ist ein “Hochprofessor”? 🙂

  98. #99 Alderamin
    31. März 2015

    @PDP10

    Und was bitte ist ein “Hochprofessor”?

    Na, halt das, was hinter einem Professor kommt. Aber was kommt hinter einem “Hochprofessor”?

  99. #100 Spritkopf
    31. März 2015

    Und was bitte ist ein “Hochprofessor”? 🙂

    Das ist einer, der in Dortmunds Lagerhallen die oberen Regale befüllt. Noch nie gehört?

  100. #101 Spritkopf
    31. März 2015

    @PDP10
    Du hast das Wort übrigens falsch geschrieben. Korrekt ist es so, wie Herr Rupp es geschrieben hat, nämlich “Hochproffesor”. 😉

  101. #102 Kyllyeti
    31. März 2015

    @rainer rupp

    Kleine Nachfrage (im Auftrage meiner Kinder, die noch was lernen wollen):

    Welche Wissenschaftler waren das denn namentlich, die

    einst berechneten die Erde ist flach und die Sonne umkreist die Erde

    ?

  102. #103 PDP10
    31. März 2015

    @Spritkopf #100:

    Boah glaubste! Wat et nich allet geben tut! 🙂

  103. #104 Basilius
    Rokujōma no Shinryakusha!?
    31. März 2015

    @rainer rupp

    Da hat Kyllyeti eine interessante Frage gestellt. Mich würde auch interessieren wer denn genau in dieser Reihe der berühmten Wissenschaftler steht?
    0_0

  104. #105 Adent
    31. März 2015

    @Basilius
    Das waren natürlich allesamt berühmte Hochproffessoren, ich kenne da namentlich die Hr. Profs. Schwall, Gelaber und Rauch.
    Mal ehrlich, wer so einen orthographischen Bauchschuß wie Hochproffesor abläßt, der hat es nicht wirklich verdient einer Antwort gewürdigt zu werden 😉

  105. #106 Gebhard Greiter
    23. Mai 2015

    Wenn ich die Allgemeine Relativitätstheorie richtig verstehe, dann würde ein Raumfahrer, der in ein Schwarzes Loch gerät, dies gar nicht merken und das Zentrums des Lochs (seine singuläre Stelle) auch gar nicht erreichen, da der Weg nach dorthin — aus seiner Sicht — unendlich lang wäre. Mit anderen Worten: Seinem Erleben nach wäre das Innere des Ereignishorizonts eines Schwarzen Lochs eine unendlich große Welt.

    Astronomen aber, die das Schwarze Loch von außen beobachten, würden es als eine nur endlich große Region unseres Universums wahrnehmen.

    FRAGE an die Experten: Enthält diese Argumentation Fehler? Wenn ja, welche?

  106. #107 Andreas
    3. Juni 2015

    @ Gebhard Greiter

    Vielleicht findet sich noch jemand, der Deine Fragen ausführlich beantworten kann.

    Die Ereignisse beim Eintritt in ein Schwarzes Loch stellen sich entgegengesetzt dar als von Dir beschrieben.

    Für einen äüßeren Beobachter tritt mit zunehmender Annäherung des Raumfahrers an den Ereignishorizont der Effekt der Zeitdilatation ein. Der Raumfahrer bewegt sich scheinbar immer langsamer in Richtung Schwarzes Loch, bis es schließlich zum Stillstand kommt. Gleichzeitig kommt es zu einer Gravitationsrotverschiebung – bis das Raumfahrzeug praktisch unsichtbar wird. Das Eintauchen eines Objektes oder eines Raumfahrers ins Innere des Schwarzen Lochs bleibt damit prinzipiell unbeobachtbar.

    Für den Raumfahrer selbst laufen die Ereignisse in seiner gewohnten Eigenzeit ab. Er wird (allerdings nur theoretisch) das Zentrum des Schwarzen Loches in endlicher Zeit erreichen. Allerdings treten schon in der Nähe des Ereignishorizonts so starke Gezeitenkräfte auf, die ihn buchstäblich zerreißen würden. Nach heutigem Kenntnisstand ist die Passage eines Schwarzen Lochs mit anschließender Reise durch ein Wurmloch nur Science Fiction vorbehalten.

  107. #108 Gebhard Greiter
    5. Juni 2015

    @Andreas

    Was die Sicht des äußeren Beobachters B betrifft, gebe ich Dir völlig recht.

    Auch dein Argument über die Gezeitenkräfte, die das Raumschiff zerreißen würden, sehe ich ein.

    Ich will mich jetzt aber mal auf den (natürlich nur hypothetischen) Standpunkt stellen, dass der Raumfahrer A – den ich mir jetzt mal als eine aus nur einem einzigen Atom bestehende Uhr vorstelle – nach erfolgreicher Überquerung des Loch-Horizonts nicht gleich zerrissen wird. Dann will ich wissen, wie weit aus seiner Sicht der Weg bis hin zum Zentrum des Lochs denn nun sein wird.

    Stellen wir uns vor, dass weit vor ihm her schon ein anderer Raumfahrer AV exakt denselben Weg gekommen ist, sich jetzt also dem Zentrum des Lochs schon weit mehr genähert hat als A.

    Nachdem der Raum um AV herum deutlich stärker gekrümmt ist, als um A herum, müsste die Sicht von A auf AV rein qualitativ doch nun eigentlich gut vergleichbar sein mit der Sicht von B auf A bevor A durch B nicht mehr wahrnehmbar war.

    Aus Sicht von A erreicht AV das Zentrum des Loches nie, und so müsste doch auch für A der Weg nach dort unendlich lang sein.

    Wenn das nicht richtig sein sollte, muss es wohl so sein, dass die Einsteinsche Feldgleichung im Inneren des Loch-Horizonts gar nicht mehr anwendbar ist (bzw. dort anders auszusehen hat).

    FRAGE also: Endet ihre Gültigkeit tatsächlich am Loch-Horizont?

    Oder endet sie erst an der Singularität (dem Zentrum des Lochs)?

    Wie ich mir die Feldgleichung und ihre Interpretation gemerkt habe, findet sich beschrieben in Notiz https://greiterweb.de/zfo/Relativ.htm#msgnr0-115 (die auf Dieter Lüst zurückgeht).

  108. #109 Krypto
    5. Juni 2015

    @Gebhard:
    Der Raumfahrer erreicht die vermutete Singularität in endlicher Zeit; insofern liegt hier ein weit verbreiteter Denkfehler vor.
    Da außerdem Raum und Zeit innerhalb des SL gewissermaßen die Koordinaten tauschen, wird Dein Gedankenexperiment erheblich komplexer.
    Schau doch mal bei Andreas Müller vorbei:
    https://www.wissenschaft-online.de/astrowissen/astro_sl_kosmo.html#kosmo

  109. #110 Alderamin
    5. Juni 2015

    @Gebhard Greiter

    Nachdem der Raum um AV herum deutlich stärker gekrümmt ist, als um A herum, müsste die Sicht von A auf AV rein qualitativ doch nun eigentlich gut vergleichbar sein mit der Sicht von B auf A bevor A durch B nicht mehr wahrnehmbar war.

    Nicht ganz. B ruht ja. A fällt Av hinterher. Was wichtig ist: für einen bewegten Beobachter ist der Ereignishorizont ein anderer als für einen ruhenden. Wenn etwa A gerade am Ereignishorizont von B ist, und Av schon dahinter, dann kann A möglicherweise Av trotzdem noch sehen.

    Ich finde das Wasserfallmodell ganz anschaulich (auch wenn es Schwächen hat): die Raumzeit fließt gewissermaßen wie ein Wasserfall in das Schwarze Loch hinein (und überhaupt auf jede Masse zu). Man kann sich relativ zu diesem Fluss nur mit Lichtgeschwindigkeit bewegen. Am Ereignishorizont relativ zu einem weit entfernt ruhenden Beobachter fließt die Raumzeit mit Lichtgeschwindigkeit ins Schwarze Loch und deswegen kann man diesem nicht mehr entkommen, weil man nicht mehr als c gegen den Strom schwimmen kann.

    Aber aus Sicht von A kann er durchaus weiter schauen und es kann ihn noch Licht gegen den Strom von weiter innen erreichen, weil der Raum sich dort mit weniger als c relativ zu ihm bewegt. Das heißt aber auch, dass A den Ereignishorizont stets vor sich hinwegweichen und zusammenschrumpfen sieht, bis zur Singularität, A erreicht ihn aus seiner Sicht nicht. Also selbst wenn für ihn Av da am Horizont einfriert, weicht dieser zurück in Richtung Singularität.

    Wie lange dauert es bis zum Zentrum? Der Fluss wird immer schneller, schon am Ereignishorizont ist man mit c unterwegs. Bei einem normalen Schwarzen Loch mit ein paar km Radius dauert es nur Sekundenbruchteile, bei einem supermassiven mit ein paar AU dauert es bestenfalls Minuten aus der Sicht des fallenden Beobachters. Eigentlich müsste er sogar die Strecke längenverkürzt sehen, er bewegt sich ja relativ zum Zentrum mit hoher Geschwindigkeit.

    Nur ein externer, ruhender Beobachter hat den Eindruck, dass ein Objekt am Ereignishorizont einfriert, weil dessen Licht dem einfließenden Raumzeitstrom entgegenlaufen muss, was zunehmend schwieriger fällt, bis es am Ereignishorizont gar nicht mehr gelingt. Das Licht bleibt da gewissermaßen stehen, die Frequenz sinkt auf 0 und alle zeitbehafteten Vorgänge scheinen stehen zu bleiben (bzw. hat, wie gesagt, ein fallender Beobachter A einen weiter innen liegenden Ereignishorizont, wo die Raumzeit relativ zu seiner Position sich mit Lichtgeschwindigkeit nach innen bewegt, und dort sieht A Objekte einfrieren).

    Für die Anschauung ist das Wasserfallmodell ganz gut, aber siehe auch die Diskussion unter dem verlinkten Artikel von Martin Bäker.

  110. #111 Andreas
    7. Juni 2015

    Alderamin hat ja nun schon sehr ausführlich und anschaulich diese Problematik dargestellt.

    Ja, mich würde auch interessieren, an welchem Zeitpunkt gerade hier in diesem speziellen Fall die Gesetze der Relativitätstheorie nicht mehr gelten. Ich würde meinen, dass dies schon am Ereignishorizont der Fall ist, weil bereits zu diesem Zeitpunkt der zeitliche und räumliche Verlauf der “Reise” ins Schwarze Loch nicht mehr sinnvoll beschrieben werden kann.

    @ Alderamin

    Weißt Du darüber etwas Konkretes?

  111. #112 Alderamin
    7. Juni 2015

    @Andreas

    M.E. versagt die RT erst bei der Singularität, weil die Quantenphysik und die RT da widersprüchliche Ergebnisse liefern. Der Ereignishorizont wurde doch aufgrund der ART erst vorhergesagt, eben weil die mögliche Fluchtgeschwindigkeit (auch für Licht) eine obere Grenze hat.

    Die heute diskutierten verheinheitlichenden Theorien wie Schleifen-Quantengravitation und String-Theorie vermeiden Singularitäten, weil sie kleinste gequantelte Volumina oder kleinste Objektausdehnungen annehmen, es gibt in ihnen keine Punktmassen. Und dann passt es mit der RT.

  112. […] Als ich nun nach Antworten suchend durch das Netz surfte, bin ich auf eine recht interessante Krücke gestoßen, die sich wohl in den Köpfen einiger Wissenschaftler bildete. Ich zitiere aus dem “scienceblog“: […]

  113. #114 Alderamin
    12. Juni 2015

    @#113

    Ich hab’ da mal einen Kommentar hinterlassen und den Unsinn höflich und sachlich richtig gestellt. Landete in der Mod. Wurde dann kommentarlos gelöscht. Der Unsinn steht noch da. Danke.

  114. #115 Niels
    12. Juni 2015

    @Alderamin
    Jetzt kann man ihn lesen, er wurde also doch nicht gelöscht.

  115. #116 Andreas
    12. Juni 2015

    Hab mal einen Blick in den o. g. Blog (# 113) riskiert und Alderamins Kommentar gelesen. Und ich muss in diesem Falle sagen: Bravo, so funktioniert Vermittlung von Wissen(schaft). Und Markus (Verfasser des Posts) hat diesen sachlich und dankend angenommen.

    Kleine Anmerkung: Die Ausführungen von Markus mögen fachlich nicht korrekt sein – aber als “Unsinn” würde ich sie nicht abtun. Er hat nur als unbedarfter Laie seine Gedanken geäußert, weit davon, jemanden eine “neue” Theorie aufzuzwingen. Und was ich für wichtig erachte: Dieser Markus macht sich Gedanken über das Universum und ist auch bereit (das zeigt sein Kommentar) kosmologische Fakten als Erkenntnisgewinn zu betrachten. Das allein unterscheidet ihn von der großen Zahl unserer Mitmenschen, für die “solche Sachen” in unserer schnelllebigen und oberflächlichen Alltagswelt keinen Platz haben.

  116. #117 Alderamin
    13. Juni 2015

    @Niels

    Ok, dann will ich nichts gesagt haben. Die Vorschau war jedenfalls verschwunden (ich war aber auch zwischendurch vom Rechner ausgeloggt, vielleicht hat WordPress mich nicht mehr erkannt).

    @Andreas

    Danke. Das Wort “Unsinn” fiel dann auch mehr aus Verärgerung, dass eine dreiviertel Stunde Tipparbeit plötzlich weg zu sein schienen. Tut mir nun leid, wo er freundlich drauf geantwortet hat. 🙁

  117. #118 Gebhard Greiter
    24. Juni 2015

    @Andreas #111:

    Teilergebnisse der Stringtheorie (die mit dem holographischen Prinzip zusammenhängen) scheinen darauf hinzudeuten, dass der Raum unseres Universums tatsächlich schon am Ereignishorizont Schwarzer Löcher endet, also nicht erst an der singulären Stelle im Zentrum des Lochs.

    Sie hierzu Dieter Lüst: Quantenfische, dtv 2014, Seite 307-308 (beginnend in der drittletzten Zeile auf Seite 307).

  118. #119 Andreas
    24. Juni 2015

    @ Gebhard Greiter,

    das ist wiederum eine andere Auffassung und sehr interessant. Ich selbst bin auch davon ausgegangen (siehe # 111). Es dürfte sicher auch schwer beschreibbar sein, an welchem Punkt die Singularität “beginnt”. Und was passiert, sagen wir mal 10 Meter “vor” der Singularität, mit einem einfallenden Objekt. Ein weiterer Aspekt: Strenggenommen gibt es diese Singularität physikalisch gar nicht (so jedenfalls die bisherigen Ergebnisse der Quantenphysik), vielmehr stellt sie ein mathematisches Hilfskonstrukt dar.

  119. #120 Martin
    27. Juli 2015

    Hallo @Florian Freistetter,

    Harald Lesch sagt aber in seiner Sendung “Alpha Centauri”, dass das Universum garnicht unendlich groß sein kann, da es in einem unendlich großen Universum unendlich viele Sterne geben müsste. Wer hat jetzt recht???

  120. #121 Martin
    27. Juli 2015

    … und er sagt, dass die Nacht in einem unendlich großen Universum hell sein müsste.

  121. #123 Martin
    28. Juli 2015

    Vielen Dank, @Florian, das klingt plausibel!

  122. #124 buechse
    28. Juli 2015

    Beim lesen diverser Kommentare (bei weitem nicht alle) unter diesem Thema fiel mir eine Frage ein.

    Seit dem Urknall expandiert das Universum und seine Bestandteile streben auseinender, am besten beobachtbar bei den Galaxien.
    Wie ausgeprägt sind dabei die Richtungen, in die Galaxien “fliegen”? Sind das bei unseren Nachbarn wie Andromeda große oder geringe Unterschiede? Könnte man anhand der unterschiedlichen Richtungen, in die sich Galaxien bewegen, nicht theoretisch einen Mittelpunkt ausmachen, der den Ursprung darstellt, oder sitze ich da einem Denkfehler auf?

  123. #125 Franz
    28. Juli 2015

    @buechse
    Jede Galaxie sieht die anderen von sich wegfliegen, somit denken alle im Universum sie wären der Mittelpunkt 🙂

  124. #126 Andreas
    29. Juli 2015

    Zur Ergänzung von Franz:

    Das Ganze ist nicht nur als optischer Effekt zu verstehen, sondern hat seinen Ursprung in der Entstehung des Universums im Urknall nach dem kosmologischen Standardmodell.

    Es war ja nicht so, dass die Materie in einen vorhandenen Raum explodiert ist, sondern der Raum selbst ist im Urknall erst entstanden und dann inflationär – so will es jedenfalls das Standardmodell – expandiert. Jedes Raumgebiet im Universum und damit auch der Raum, wo Du und ich uns aufhalten, ist daher immer noch Teil dieses “Mittelpunkts”. Schwer vorstellbar, aber der Ort des Urknalls und damit das “Zentrum” des Universums ist überall.

  125. #127 Holger
    Hamburg
    28. September 2015

    Die Urknalltheorie ist nur eine Hypothese, und keineswegs bewiesen. Diese Hypothese ist die Modernisierung des frühmittelalterlichen geozentrischen Weltbildes. Das Moderne des heutigen, und daher des spätmittelalterlichen ‘Weltbildes ist – was die Kirche nicht mehr leugnen kann – die Anerkennung, dass die Erde nicht im Mittelpunkt steht (die Sonne auch nicht). Aber sonst haben wir nach der Urknallhypothese ein Blasenuniversum, mit soundsoviel Milliarden Lichtjahren Durchmesser.
    Ebenso dogmatisieren viele “Wissenschaftler” immernoch mit der Lichtkonstante herum, das heißt: Es wird ernsthaft behauptet, dass sich das Licht im interstellaren Raum mit konstanter Geschwindigkeit ausbreitet – daher also Photonen (Lichtteilchen) niemals langsamer werden und an Energie verlieren. Damit haben wir das nächste spätmittelalterliche aber modernisierte Phänomen staatlich bezahltter Professoren, das moderne Perpeduum Mobile.
    Wischen wir einfach die Präsentationen des Spätmittelalters beiseite so kommt man auf eine ganz einfache Lösung:
    Materie existiert unendlich in unendlichem Raum und unendlicher Zeit. Verwendet man anstatt des Begriffes “Universum” das Wort Unendlichkeit, so erhält man einen Hauch von Philosophie.

  126. #128 Dietmar
    28. September 2015

    @Holger: (Der Holger, der für die Zeitung mit den vier großen Buchstaben kleine Texte herunterschmieren darf? Passen würd´s …)

    Die Urknalltheorie ist nur eine Hypothese, und keineswegs bewiesen.

    Dann erkläre mal die kosmische Hintergrundstrahlung und die Temperaturunterschiede in ihr.

    Diese Hypothese ist die Modernisierung des frühmittelalterlichen geozentrischen Weltbildes.

    Nein, ganz im Gegenteil: Genauso wenig, wie Du der Mittelpunkt auf einer Erdscheibe bist, obwohl der Horizont von Dir aus überall gleich weit entfernt und kreisrund ist, wenn der Blick nicht verstellt ist, ist die Erde das Zentrum des Universums. Die Hintergrundstrahlung ist der kosmische Horizont, der sich von einer anderen Perspektive aus ebenso zeigen würde, als wäre man dort im Zentrum.

    Wenn Du so etwas Simples nicht verstehst, meinst Du, Du kannst Physik philosophisch widerlegen? Das ist lächerlich.

    Es wird ernsthaft behauptet, dass sich das Licht im interstellaren Raum mit konstanter Geschwindigkeit ausbreitet – daher also Photonen (Lichtteilchen) niemals langsamer werden und an Energie verlieren.

    Wo soll denn die Energie hin? Woran sollen die Photonen denn im leeren Raum ihre Energie abgeben?

    Damit haben wir das nächste spätmittelalterliche aber modernisierte Phänomen staatlich bezahltter Professoren, das moderne Perpeduum Mobile.

    Das perpetuum mobile ist aber die Idee einer Maschine, die immer in Bewegung bleibt. Etwas Künstliches. Photonen wurden von niemandem gebaut. Können also kein perpetuum mobile sein. Du hast aufgeschnappt, dass es kein perpetuum mobile geben kann. Das ist ja schon mal schön. Jetzt denkst Du, das wäre Philosophie, wenn man einen Begriff falsch anwendet und damit glaubt, Physik zu widerlegen?

    Materie existiert unendlich in unendlichem Raum und unendlicher Zeit.

    Alle Materie? Also ist die Kernfusion in den Sternen, die die Elemente bildet, aus denen Dein Körper besteht, nur ein Märchen?

    Verwendet man anstatt des Begriffes “Universum” das Wort Unendlichkeit, so erhält man einen Hauch von Philosophie.

    Und verwendet man statt des Begriffs “schokoladeummantelter Keks” die Worte “längste Praline der Welt” erhält man: einen schokoladenummantelten Keks. Begriffe zu jonglieren und umzudeuten ist keine Philosophie sondern Rhetorik. Google mal die Begriffe, die Du so vollmundig meinst anwenden zu können!

  127. #129 Dietmar
    28. September 2015
  128. #131 Alderamin
    28. September 2015

    @Florian

    Ach, Florian, Du bist doch “Wissenschaftler”, warum sollte ein “Kommentator” wie “Holger” Deine Artikel mit Wohlwollen “lesen”… 😉

    @Holger

    Frage an den “Experten”: wenn in Sternen Wasserstoff zur Energieerzeugung verbraucht wird, wo kommt in einem ewigen Universum dann der frische Brennstoff her?

  129. #132 Holger
    Hamburg
    28. September 2015

    1940 wurde der Priester und Theologe Georges Edouard Lemaître aufgrund seiner wissenschaftlichen Leistungen in die Päpstliche Akademie der Wissenschaften berufen. Seine Leistungen waren für die Kirche von unschätzbarem Wert. Er fand einen Weg für die Vereinbarkeit katholischer Schöpfungslehre und Wissenschaft. Kam doch die alte Schöpfungslehre mit dem Fortschreiten der Wissenschaft mehr und mehr in Bedrängnis. Die aufkommende Frage
    „Ist das Universum unendlich groß?“ ließ Gotteszweifel wachsen.
    Eine Lösung musste gefunden werden.

    Lemaitre gilt als Begründer der Urknalltheorie. Seine Arbeiten beziehen sich im Kern auf die „Expansion des Weltalls“. Denn etwas Expandierendes muss in frühestem Stadium von einem zentralen Punkt aus begonnen haben. Das eröffnet die Möglichkeit des Glaubens an einen Schöpfer, der den Knall ausgelöst hat.

    1951 nahm die Päpstliche Akademie der Wissenschaften Lemaîtres Theorie an. Papst Pius der XII. trug in einer abschließenden Rede, am Tag der Annahme der Urknalltheorie vor, dass der Urknall der zeitlich festgelegte Anfang der Welt-, ein göttlicher Schöpfungsakt sei.

    Erst kurz vor seinem Tod erfuhr Lemaitre von der kosmischen Mikrowellenstrahlung, welche als Erhärtung seiner Theorie angesehen wurde.

    Das soll nur ein kurzer Ausflug in die Historie der Urknall-Hypothese sein, um zu verstehen, mit wessen Theorie wir es zu tun haben und warum diese Theorie entstanden ist.

  130. #133 noch'n Flo
    Schoggiland
    28. September 2015

    @ Holger:

    Du vergisst zu erwähnen, dass Lemaître auch ein studierter und promovierter Astrophysiker war, dem es vor allem um wissenschaftliche Erkenntnis ging. Dass seine Urknalltheorie von der Kirche so gut umgedeutet werden konnte, dass sie in den Schöpfungsmythos passte, war eher eine Art Nebenprodukt – wenn auch nicht unwillkommen.

  131. #134 Alderamin
    28. September 2015

    @Holger

    Und Edwin Hubble hatte mit der Kirche nie was zu tun, fand dieselbe Beobachtung unabhängig, veröffentlichte sie vor Lemaitre auf Englisch, der daraufhin davon absah, auf seine nur auf französisch veröffentlichten Ergebnisse zu pochen. Tatsächlich war es allerdings Alexander Friedmann, der 1922 als erster eine Lösung der Allgemeinen Relativitätstheorie für das Universum fand, die auf eine Expansion (oder alternativ einen Kollaps) desselben führte, und Vesto Slipher fand als erster, dass ferne Galaxien eine entsprechende Rotverschiebung zeigten.

    Seitdem haben unzählige Astronomen, die ebenfalls nichts mit der Kirche am Hut haben, viele davon gar Atheisten, die Beobachtungen nachvollzogen, bestätigt, verfeinert und durch weitere Beobachtungen untermauert. Der Himmel lässt sich nicht vertuschen. Wenn irgendeiner einen Unsinn über den Himmel behauptet, wird der Unsinn von einem anderen schnell entlarvt werden. Ein Grundpfeiler der Wissenschaft ist das Nachprüfen von Behauptungen, mit dem Ziel, die Behauptung zu Fall zu bringen. Das ist in bald 90 Jahren bei der Urknalltheorie niemandem gelungen. Und das liegt nicht daran, dass alle anderen Leute weniger clever als Du sind.

    Zurück zu meiner Frage: wie und wo entsteht in einem ewigen Universum der Brennstoff für die Sterne?

  132. #135 Andreas
    28. September 2015

    Der Reihe nach:

    @ Florian

    Bei allem Respekt, aber die Urknalltheorie als bewiesen anzusehen, ist doch etwas übertrieben. Es gibt eben viele Wissenschaftler (absolut gesehen, nicht relativ), die sich mit Konstrukten wie Inflatonfeld, Dunkler Energie … nicht abfinden können. Ja, das Thema hatten wir schon. Du stehst hinter der Urknalltheorie – okay, aber wenn Du zur Begründung für dieses Weltmodell auf Deine eigenen Beiträge im Block verweist, ist das für Jemanden, den Du überzeugen willst, wenig glaubhaft.

    @ Holger

    An den obigen Ausführungen erkennst DU, dass auch ich kritisch der Urknalltheorie gegenüberstehe, aber aus ganz anderen Gründen.
    Was die religiöse Auslegung der Urknallkritiker betrifft, da kann ich überhaupt nicht zustimmen. Ich könnte sogar das Gegenteil behaupten, weil – und jetzt kommt`s:

    Ich hatte meine Schulbildung in der DDR und da gab es in der 10. Klasse auch Astronomieunterricht. Mit Religion hatte das Schulwesen aber herzlich wenig am Hut. Die Enstehung der Welt aus dem Urknall wurde als physikalische Theorie interpretiert, die einen Schöpfer (Gott) überflüssig macht – also genau das Gegenteil von dem, was Du behauptest.

    Grüße Andreas

  133. #136 Florian Freistetter
    28. September 2015

    @Andreas: “aber wenn Du zur Begründung für dieses Weltmodell auf Deine eigenen Beiträge im Block verweist, ist das für Jemanden, den Du überzeugen willst, wenig glaubhaft. “

    Es ist also nur glaubhaft, wenn jemand anderes als ich erklärt, welche Beobachtungsdaten das Urknallmodell stützen? Kann ich jetzt irgendwie nicht ganz nachvollziehen. Die Realität ist die Realität, egal ob sie von mir oder jemandem anderen erklärt wird…

  134. #137 Adent
    28. September 2015

    @Andreas
    Ähm, wenn man aber liest, was Florian in den Artikeln schreibt, dann sind das ja nicht seine Erfindungen oder Behauptungen, sondern das was andere an Belegen erforscht und herausgefunden haben, wo ist also das Problem?

  135. #138 Holger
    Hamburg
    29. September 2015

    Andreas hat Recht, wenn er schreibt, dass es wohl kein Problem ist, von wem Wahrheit aufgeschrieben wird. Wahr ist wahr, ob selbst aufgeschrieben oder von Anderen.
    Jedoch denke ich, dass man auf dieser Plattform mit kurzen Beiträgen Erklärungen für seine Überzeugungen geben kann. Für eine Doktorarbeit ist hier nicht genug Platz. Und auf Doktorarbeiten oder gar Bücher zu verweisen, ist ja wohl ein bisschen zu einfach.
    „Lies erst mal das und das… „
    Kann ich auch machen:
    „Lies erstmal Newton… „ oder „Lies erstmal Einstein…“ (wäre vielleicht gar nicht so schlecht, vielleicht wurde ihm was angedichtet, was gar nicht von ihm kommt.“
    Ich habe ein Problem damit, wenn Hypothesen (oder Theorien) nicht logisch schlüssig sind. Das ist also das Mindeste. Die Sache muss aufgehen, muss mir die Welt erklären, auch mit einfachen Worten. Einfach nur Kinderfragen mit einfachen Worten beantworten.
    Zum Beispiel: „Warum ist der Nachthimmel schwarz?“

  136. #139 Franz
    29. September 2015

    @Holger
    Was ist an der Urknalltheorie nicht logisch schlüssig ?

  137. #140 Andreas
    29. September 2015

    @ Florian

    Wenn Du schreibst: “Die Realität ist die Realität, egal ob sie von mir oder jemandem anderen erklärt wird…” – dann ist dies erst einmal richtig. Das Problem sehe ich darin, dass der Urknall eben keine Realität darstellt. Es ist und bleibt eine Theorie oder ein Modell, die, wie man immer so schön sagt, den Anfang des Universums aufgrund unserer Beobachtungen und unserem Wissensstand am Besten beschreibt. Da habe ich gar nichts dagegen einzuwenden. Deshalb fand ich die Überschrift des Artikels ” Den Urknall gab es wirklich” etwas vermessen.

    Was die Beobachtungen, die Du anführst betrifft: Da kommt es darauf an, welche man einbezieht und welche man lieber weglässt. Der Objektivität halber sollten auch solche Phänomene erklärt werden, wie die unterschiedliche Rotverschiebung in Größenordnungen (und damit Entfernung) von solchen Objekten wie NGC 4319 und Quasar Markarian 205.
    Diese beiden Objekte sind über eine Materiebrücke miteiander verbunden, obwohl das aufgrund der Entfernung gar nicht geht. Es gibt noch eine Vielzahl weiter Objekte, die dieses Phänomen aufweisen.
    Die Erklärung der Urknallkritiker: Die Rotverschiebung ist nicht mit der Entfernung der Objekte korrelliert – logisch, wo soll sonst diese Materiebrücke herkommen?
    Die Vertreter der Urknallhypothese: Es handelt sich um eine (zufällige!!!) Erscheinung im Hintergrund.
    Es möge sich jeder ein Bild machen, wie “zufällig” dies sein kann. Zudem zeigen sich Ausbuchtungen an beiden Objekten, die auf einen Materieabfluss hindeuten. Wie ich weiß, gibt es noch keine eindeutige(!) Erklärung, ich lasse mich jedoch gern eines Besseren belehren.
    Das war nur ein Beispiel, mit welchen Interpretationsschwierigkeiten die Urknalltheorie zu kämpfen hat – da gibt es noch sehr viele.
    Das bedeutet aber noch nicht, dass das kosmologische Standardmodell falsch ist. Aber es zeigen sich einige Unzulänglichkeiten, die zuwenig kritisch hinterfragt werden.
    Ich glaube nicht, dass durch die Einführung immer neuer, bizarrer Urknallmodelle, wie sie in den letzten Jahrzehnten wie Pilze aus dem Boden schossen, der Wissenschaft ein echter Dienst erwiesen wird. Hier wäre eine kontroverse Diskussion hilfreich. Aber wenn prominente Wissenschaftler wie Halton Arp davon ausgeschlossen wurden (Streichung der Beobachtungszeit, Nichtteilnahme an seinen Vorträgen) ist das kein gutes Zeichen. Wäre nicht da die Gelegenheit gewesen, den Kritiker der Urknalltheorie zu widerlegen. Es kam aber keiner – warum wohl.

    Zum Abschluss ein Zitat von Rudolf Kippenhahn, ehemaliger Leiter des Max-Planck Instituts für Astrophysik über Alton Harp und (meiner Meinung nach) passend zum Blog “Ist Wissenschaft dogmatisch”:

    „Wir brauchen Leute wie ihn, sonst besteht die Gefahr, dass sich in der Wissenschaft Cliquen bilden, die keine Kritik von außen zulassen.“

    Zu spät, die sog. Elite der Wissenschaft hat sich längst etabliert …

  138. #141 andy f
    29. September 2015

    @Holger: Es tut mir wirklich leid, dir das sagen zu müssen. Aber es gibt Dinge, die eben NICHT mit einfachen Worten als Antworten auf Kinderfragen zu erfassen sind. Und das gilt schon für deutlich alltäglicheres als die Frage nach dem Urknall.

    Wobei, wenn du kindgerechte Antworten haben möchtest bist du in der Kirche ja ganz gut aufgehoben.

  139. #142 Captain E.
    30. September 2015

    @Andreas & Holger:

    In der Wissenschaft sind Theorien immer logisch schlüssig. Wären sie es nicht, würde man sie noch “Hypothese” nennen.

    Und die Theorie von der Entstehung des Universums per Urknall kennt viele Belege und erklärt manches, und durch Gegenbeweise erschüttert werden konnte sie nicht. Mit anderen Worten: Sie scheint brauchbar zu sein.

  140. #143 gaius
    30. September 2015

    @Holger: “Die Sache muss aufgehen, muss mir die Welt erklären, auch mit einfachen Worten.”

    Wieso gehst du davon aus, dass die Struktur des Universums an die äusserst begrenzten Wahrnehmungs- und Denkfähigkeiten des Menschen angepasst sein sollte?

    Die Evolution hat den Menschen für ganz andere Aufgaben gemacht. Es ist ein Wunder, dass wir überhaupt etwas vom Universum begreifen können. Da können wir nicht noch verlangen, dass es uns einfach vorkommt …

  141. #144 Captain E.
    30. September 2015

    Mich durchzucken gerade zwei Sketch-Szenarien: 😉

    1
    Petrus: Herr, mach doch bitte die Welt nicht dermaßen kompliziert. Kein Mensch versteht sie wirklich.
    Gott: Unsinn! Ich mache sie so kompliziert wie ich will, und jetzt stör mich nicht mehr.

    2
    Petrus: Herr, die Menschen beginnen, die Welt zu verstehen.
    Gott: Oh nein, das darf nicht geschehen! Schnell, hol mir meinen Werkzeugkoffer! Ich muss der Welt ganz dringend mehr Komplexität hinzufügen.

  142. #145 nur Consuela
    30. September 2015

    @Holger
    Nur der Vollständigkeit halber: Genau wegen des Urknalls ist der Nachthimmel schwarz, schön erklärt von FF hier:
    https://scienceblogs.de/astrodicticum-simplex/2009/02/10/warum-ist-es-nachts-dunkel/

  143. #146 bikerdet
    30. September 2015

    @ nur Consuela :

    Ne, so geht das nicht. Wenn jemand hier in Florians Blog Fragen stellt, darf Florian diese natürlich nicht beantworten. Und schon gar nicht darf er auf eigene Artikel im Blog verweisen. Das wäre ja unglaubwürdig….

    P.S. Nur zur Sicherheit : Das war Sarkasmus !

  144. #147 nur Consuela
    30. September 2015

    @ bikerdet: Keine Angst, ich verstehe dich schon richtig.

    Bis zu einem gewissen Grad kann ich den Einwand ja sogar verstehen – auf diversen Pseudowissenschafts-Seiten wird ja auch mit Vorliebe auf sich selbst zurückverlinkt. Im Unterschied dazu könnte Florian hier aber zweifellos auf weiterführende Papers verlinken – die muss man dann halt aber auch wieder lesen, was mit den gewünschten “einfachen Worten” dann wieder nur schwer zusammengeht …

  145. #148 Andreas
    30. September 2015

    Ich gebe Captain E. (#142) mit seiner Aussage im ersten Absatz vollkommen recht.
    Was die Urknalltheorie betrifft, so ist sie meiner Meinung nach durchaus “brauchbar”, wie es Captain E. so schön ausdrückt. In den 70´und 80´Jahren war ich davon sogar überzeugt. Aber dann kamen Inflation, Inflatonfeld, Skalarfeld, Vakuumfluktationen des Vakuums (ohne Vakuum, da Raum und Zeit doch erst mit dem Urknall entstanden sein sollen), Dunkle Energie und weitere zusätzlich eingefügte Annahmen, für die es keine Belege gibt – nur damit das Modell wieder stimmt – der Urknall wird vorausgesetzt und die Urknalltheorie beweist sich selbst. Ich habe mal gelesen und es sollte nachdenklich stimmen (Sinngemäß):

    In keinem anderen Bereich der Wissenschaft würde das ständige Einführen neuer,nicht belegbarer zusätzlicher Annahmen als Indiz für die Stichhaltigkeit einer Theorie akzeptiert werden.

  146. #149 Alderamin
    1. Oktober 2015

    @Andreas

    Aber dann kamen Inflation, Inflatonfeld, Skalarfeld,

    Die Urknalltheorie funktioniert auch ohne Inflation. Allerdings ist die Inflation das plausiblere Szenario, da es auch die beobachteten Strukturen in der Hintergrundstrahlung liefert.

    Wie erklären alternative Theorien diese? Soviel ich weiß gar nicht.

    Vakuumfluktationen des Vakuums (ohne Vakuum, da Raum und Zeit doch erst mit dem Urknall entstanden sein sollen)

    Der Ursprung der Expansion ist reine Hypothese und (noch) kein fester Bestandteil der Urknalltheorie. Solange es keine Theorie der Quantengravitation gibt, kann über diese Phase nur spekuliert werden. Das gilt im Prinzip auch noch für die Inflationsphase, aber die macht immerhin schon überpürfbare Vorhersagen (Feinstruktur und Polarisierung der Hintergrundstrahlung). Vakuumfluktuationen an sich sind aber schon belegt und werden von der Heisenbergschen Unschärferelation ja auch verlangt, der Energiegehalt eines sehr kleinen Volumens kann nicht für beliebig kurze Zeiten exakt 0 sein. Was durch den Casimir-Effekt ja auch experimentell belegt werden kann.

    Dunkle Energieund weitere zusätzlich eingefügte Annahmen, für die es keine Belege gibt

    … ist bisher nur der Name für eine Beobachtung, dass nämlich die Expansion des Universums sich beschleunigt hat (bzw. die Rotverschiebung mit der Entfernung überproportional zunimmt, und für große Entfernungen dann wieder unterproportional). Ein Mechanismus ähnlich der Inflation, beruhend auf Vakuumenergie wäre möglicherweise eine Erklärung, aber bisher weiß niemand genau, was sie ist. Allerdings hat sie genau den richtigen Betrag, um die beobachtete Flachheit des Universums zu erklären (kritische Dichte) und ihr Wert führt auch dazu, dass die ältesten Sterne nicht älter als das aus dem Urknall ermittelte Alter des Unviersums sind. Das passt zu gut zusammen, um irgendein Zufall zu sein.

    nur damit das Modell wieder stimmt – der Urknall wird vorausgesetzt und die Urknalltheorie beweist sich selbst.

    Das ist definitiv falsch. Die Dunkle Energie kam völlig unerwartet. Auch wenn man sich nicht erklären konnte, wie die kritische Dichte zusammen kam (im Prinzip hätte es auch noch mehr dunkle Materie sein können, zwischen den Galaxien, und im Prinzip löst die Inflation das Flachheitsproblem ja auch) und es war in den 80ern/90ern jedem Kosmologen völlig klar, dass das Weltall seine Expansion verlangsamen müsse – zum Big Crunch oder zu endloser, aber sich verlangsamender Expansion. Auch die Inflation wurde nicht erfunden, um das Flachheits- und Horizontproblem zu lösen, sondern der Alan Guth suchte eine Lösung für das Problem magnetischer Monopole. An der Urknalltheorie ändert sich für die Zeit nach 10^-33 s gar nichts, wenn man die Inflation weglässt, man hat dann halt nur zwei, drei ungelöste Probleme mehr.

    Alternative Theorien erklären überhaupt nur Einzelbeobachtungen und haben für den Nachschub an frischem Gas nur hypothetische “weiße Löcher” und für die Rotverschiebung eine bisher auch noch nie auf unabhängige Weise nachgewiesene oder aus irgendeiner bestätigten Theorie folgende “Lichtermüdung” anzubieten. Das sind doch mindestens genau so abgedrehte Postulate. Und die meisten Beobachtungen erklären sie gar nicht.

    Die Sache mit Markarian 205 ist übrigens schon seit 1992 geklärt. Das Licht des Quasars durchläuft den Halo der Galaxie und entsprechende Linien wurden nachgewiesen, er ist also weiter weg. Und die vorgebliche Brücke, die man auf dem Foto im Wiki-Artikel gar nicht erkennen kann, kann ja einfach ein Sternenstrom einer verschluckten Galaxie sein, so was hat die Milchstraße auch. Wo der Quasar halt zufällig dahinter liegt. Solche Einzelobjekte, die auch im Rahmen der Urknalltheorie erklärbar sind, bringen diese nicht zu Fall, dafür passt anderswo zu viel zusammen, das ansonsten unerklärt bliebe.

    In keinem anderen Bereich der Wissenschaft würde das ständige Einführen neuer,nicht belegbarer zusätzlicher Annahmen als Indiz für die Stichhaltigkeit einer Theorie akzeptiert werden.

    Ist das von Unzicker? Wie soeben begründet versucht man Beobachtungen durch schlüssige Hypothesen zu erklären und sucht dann auch nach Belegen für diese – etwa nach den Teilchen der Dunklen Materie oder der Polarisierung der Hintergrundstrahlung durch die Inflation. Eine Zusatzhypothese ist zunächst stets unbelegt.
    Aber auch ohne Zusatzhypothesen ändert sich nichts an dem groben Bild des Urknallmodells, das durch viele unabhängige Beobachtungen gestützt wird.

    Die alternativen Theorien greifen aber auch tief in die Trickkiste, schon alleine um Einzelerklärungen hin zu bekommen. Wenn etwa die Steady-State-Theorie die Feinstruktur der Hintergrundstrahlung oder das Verhältnis der Elemente im primordialen Gas erklären soll, dann müsste sie sich auch eine Menge einfallen lassen. Bisher gibt’s dazu aber nur ein Schulterzucken von ihr.

  147. #150 Captain E.
    1. Oktober 2015

    @Andreas:

    Das war ja mal ein hübscher Versuch von “Verbaljudo”. Nur wie willst du begründen, dass die Urknalltheorie einerseits brauchbar und andererseits falsch ist? Denn genau das behauptest du doch, oder? Die Wissenschaftler hätten sich etwas ausgedacht, was vorne und hinten nicht hinhaut, und bevor sie sich von ihrer hübschen Idee verabschieden, erfinden sie sich alle möglichen “Sonderlocken” hinzu.

    Aber wie Alderamin schon erwähnt hatte, ist das Ganze das Resultat jahrzehntelanger Forschungen und ist in sich konsistent. Zusatzhypothesen werden natürlich angenommen, um die Ergebnisse in bestimmten Fällen besser zu machen. Wo diese schon zuvor gut gewesen sind, sollen diese Hypothesen dann keine Veränderungen bewirken, zumindest nicht zum schlechteren.

  148. #151 Andreas
    1. Oktober 2015

    @ Captain E.

    Ich habe nie gesagt, dass die Urknalltheorie unbedingt falsch ist, aber die in regelmäßigen Abständen erfolgende Annahme zusätzlicher Faktoren ist für mich nicht zufriedenstellend. Es wäre doch hilfreich, wenn auch nach anderen Erklärungen bzw. Interpretationen der Beobachtungen gesucht würde, vorurteilsfrei, ohne das Standardmodell gleich zu verteufeln. Das geschieht jedoch nicht – welche “seriöse” Forschungseinrichtung gäbe wohl die finanziellen Mittel, die Technik und ausreichend Beobachtungszeit für derartig “abwegige” Untersuchungen?
    Doch wenn man nicht sucht, wird man auch nichts finden.

  149. #152 Alderamin
    1. Oktober 2015

    @Andreas

    aber die in regelmäßigen Abständen erfolgende Annahme zusätzlicher Faktoren ist für mich nicht zufriedenstellend.

    So funktioniert aber doch Wissenschaft generell. Man macht eine neue Beobachtung, man stellt eine, oder zwei, drei Hypothesen auf und sucht nach Bestägigungen oder Widersprüchen. Solange man keine zweifelsfreien Widersprüche findet, gilt die Hypothese und wird zur Theorie, wenn sie weitere Beobachtungen erklären kann. Und viel mehr wird dann auch nicht daraus, es kommt niemals der Zeitpunkt, wo eine Theorie “endgültig richtig” wird. Man kann nur mit immer mehr Beobachtungen den Bereich, wo die Theorie funktioniert, besser auszuloten versuchen.

    Bei der Kosmologie hat man naturgemäß das Problem, dass man wenig Experimente machen kann und sich auf Beobachtungen der Ergebnisse vergangener Prozesse beschränken muss. Das schafft grundsätzlich mehr Unsicherheit als bei der Laborphysik. Wir können z.B. keine Inflation im Labor ausprobieren, und unsere Beschleuniger kratzen ja nur ein wenig an Teilchenenergien, die kurz nach der Inflation herrschten. Wir können auch keine Sonde ein paar Milliarden Lichtjahre in den Weltraum hinaus schießen, sie gegenüber der Hintergrundstrahlung in einen Ruhezustand versetzen und dann beobachten, wie sich ihre Entfernung mit der Zeit entwickelt. Einen experimentell nicht zugänglichen Bereich auszuloten, ist schwierig.

    Das gilt aber für alle anderen alternativen Modelle genau so.

    Die Annahmen von Inflation, Dunkler Materie und Dunkler Energie liefern jedenfalls ein schlüssiges Bild, das in Simulationen wie Illustris ziemlich gut das beobachtete Weltall reproduziert, und das ist eine bestätigende Beobachtung. Dass das Weltalter aus der Urknalltheorie zu den ältesten Sternen passt, ist auch eine bestätigende Beobachtung. Dass das Verhältnis der Elemente im ursprünglichen Gas zu der Menge an Dunkler Materie passt, die zur Erklärung der Rotationskurven von Galaxien und zum Zusammenhalt von Galaxienhaufen nötig ist, und die man im Spektrum der Hintergrundstrahlung findet, ist eine weitere bestätigende Beobachtung, usw. Da gibt es viele Zusammenhänge.

    Es ist nicht einfach so, dass man für jede neue Beobachtung eine neue Zusatzannahme braucht, die dann nur die neue Beobachtung erklärt, das wäre ein schlechtes Modell (etwa, dass man Lichtermüdung postuliert, die nur die Rotverschiebung erklärt und sonst nichts). Wer die Urknalltheorie nur flüchtig aus populärwissenschaftlichen Quellen kennt, der mag diesen Eindruck bekommen, aber er stimmt nicht. Manche dieser Zusammenhänge sind so komplex, dass sie dem Laien nicht einfach verständlich gemacht werden können, etwa wie man aus der Signatur baryonischer akustischer Oszillationen auf die Dichte der Dunklen Energie schließen kann. Es ist daher schwierig, dieses verwobene Netz aus sich bestätigenden Annahmen und Beobachtungen feinsäuberlich und in kleinen für Laien verdaulichen Häppchen aufzudröseln.

  150. #153 Andreas
    1. Oktober 2015

    @ Alderamin

    Vielen Dank für Deine und weiter oben auch sehr ausführliche Darstellung des gegenwärtigen Kenntnisstandes.

    Um es vorweg zu nehmen, ich habe mich seit meiner Jugend für Astronomie und in ihrer Erweiterung auch mit Kosmologie beschäftigt. Leider – und beim Lesen mancher Originalmanuskripte bedaure ich dies – habe ich nicht studiert und mein Wissen aus seriösen populärwissenschaftlichen Quellen bezogen. Die Schwierigkeit besteht darin, die Argumente der beiden Seiten (pro/kontra Urknall) mit dem dafür nötigen Wissensstand beurteilen zu können. Es fehlt der Einblick und das Verständnis, sowohl die Details als auch die Beobachtungen, die eine Theorie stützen rechnerisch – das ist ganz wichtig – nachzuprüfen. Das gebe ich unumwunden zu, betrifft aber nur mich persönlich.

    Was aber ist mit den (zugegebenermaßen relativ gesehen wenigen) seriösen (!) Wissenschaftlern, die dem kosmologischen Standardmodell weiterhin kritisch gegenüberstehen? Es hat doch sicherlich seinen Grund, warum diese die gemachten Beobachtungen und Annahmen nicht akzeptieren und auf eine Forschung in alternativer Richtung suchen. Alles Egozentriker wie Arp, auf der Suche nach Selbstdarstellung, verrannt in eine fixe Idee oder mangelndes Wissen (wie ich) ? Nein, das kann ich mir beim besten Willen nicht vorstellen. Warum werden ihre Forschungsergebnisse und Publikationen (sofern diese in anerkannten Fachzeitschriften veröffentlicht werden) mit Ignoranz behandelt?

    Es geht mir gar nicht darum, zu beurteilen, ob das CDM richtig oder falsch ist. Um es salopp zu sagen, ob Entstehung aus dem Nichts (Urknalltheorie) oder ewiges Universum (Steady-State-Theorie ???) – ich ziehe keines der beiden Weltmodelle aus religiösen oder philosphischen Erwägungen vor.

    Wenn die Urknallkritiker unter exakt (!) denselben Bedingungen forschen könnten, wie sie den etablierten Wissenschaftlern zur Verfügung stehen, wären wir in ein, zwei Jahrzehnten viel weiter. Dann würde die Gegenüberstelllung und Diskussion zeigen, in welche Richtung unser Verständnis zur Entstehung des Universums geht. Und es ist völlig egal, welches Modell sich durchsetzt (vielleicht ein neues). Jeder der beiden “Parteien” hätte auf seine Weise einen Beitrag geleistet. Doch dies wird wohl nur eine Vision bleiben…

  151. #154 Alderamin
    1. Oktober 2015

    @Andreas

    Die Schwierigkeit besteht darin, die Argumente der beiden Seiten (pro/kontra Urknall) mit dem dafür nötigen Wissensstand beurteilen zu können.

    Dann mach’s wie anderswo: guck’ Dir die Leute an, die eine Meinung vertreten und gewichte die Anzahl der pro- und contra-Stimmen.

    Zu den Leuten habe ich hier was gefunden (leider etwas älter, viele Links sind tot, aber dieses Cosmology Statement, wo Du den Satz, den ich Unzicker zuschrieb, anscheinend her hast, ist nun auch schon 11 Jahre alt und Arp ist auch schon zwei Jahre tot).

    Was aber ist mit den (zugegebenermaßen relativ gesehen wenigen) seriösen (!) Wissenschaftlern, die dem kosmologischen Standardmodell weiterhin kritisch gegenüberstehen?

    Kreationisten, Cranks, Leute, die sich in ihrer Ehre verletzt fühlen, die ewig Gestrigen, Egozentriker, Leute, die grundsätzlich immer opponieren, sehr viele Fachfremde und Ingenieure und auch solche Menschen, die einzelne Ideen ablehnen und ihre eigenen gerne veröffentlicht sähen. Es gibt viele Gründe, warum Menschen an irgendwelchen Meinungen festhalten. Das Hauptziel des Cosmology Statement scheint jedenfalls zu sein, sich den Reviews der bösen Kollegen zu entziehen. Abgesehen davon und der Ablehnung der Urknalltheorie eint diese Leute nichts, nicht einmal eine alternative Theorie, da glaubt jeder was anderes.

    Warum werden ihre Forschungsergebnisse und Publikationen (sofern diese in anerkannten Fachzeitschriften veröffentlicht werden) mit Ignoranz behandelt?

    Weil sie keine überzeugenden Daten oder Argumente liefern? Warum wurde denn das Perlmutter-Papier zu beschleunigten Expansion zugelassen, was doch auch der geltenden Forschungsmeinung widersprach? Weil es überzeugte. Es ist der Job derjenigen Wissenschaftler (oder solcher, die sich dafür halten), die Fachwelt von ihren Ideen zu überzeugen. Wenn das nicht gelingt, sollten sie sich Gedanken machen, wie sie ihre Behauptungen besser belegen können, statt sich in die Schmollecke zurück zu ziehen.

    Wenn die Urknallkritiker unter exakt (!) denselben Bedingungen forschen könnten, wie sie den etablierten Wissenschaftlern zur Verfügung stehen, wären wir in ein, zwei Jahrzehnten viel weiter.

    Auch schön abgeschrieben. Auch wenn man 10 Jahre lang ein totes Pferd reitet, wird man damit den CHIO nicht gewinnen. Wenn die Vertreter alternativer Ideen ein Argument hätten, bekämen sie auch Teleskopzeit. Wenn man ihre Argumente allerdings widerlegt hat (wie das Markarian-Beispiel), dann gehen andere Leute auch mal vor mit ihren Anträgen auf Beobachtungszeit. Denn die ist limitiert.

    Übrigens gibt es mittlerweile ganz ordentliche Sternwarten, wo man für kleines Geld per Internet automatische Aufnahmen machen lassen kann, mit Filtern und Pipapo unter besten Beobachtungsbedingungen (Damian Peach schrieb in einer der letzten Sky & Telescopes darüber, dass er da auch Aufnahmen anfertigt, da ist man mit $30 die Stunde dabei, wobei nur die echte Belichtungszeit abgerechnet wird). Und die Beobachtungen anderer Forscher sind doch auch meist allgemein zugänglich. Selbst die vom Hubble-Teleskop. Wer hindert sie daran, diese Daten zu nutzen?

    Dann würde die Gegenüberstelllung und Diskussion zeigen, in welche Richtung unser Verständnis zur Entstehung des Universums geht.

    Die Diskussion findet doch längst statt. Und siehe da, die Autoren nutzten Daten des Sloan Digital Sky Survey, die allen Wissenschaftlern zur Verfügung stehen, die haben nicht gejammert, dass sie keine Teleskopzeit bekamen. So geht Wissenschaft.

  152. #155 Adent
    2. Oktober 2015

    @Andreas

    Nein, das kann ich mir beim besten Willen nicht vorstellen. Warum werden ihre Forschungsergebnisse und Publikationen (sofern diese in anerkannten Fachzeitschriften veröffentlicht werden) mit Ignoranz behandelt?

    Meines Erachtens nach begehst du genau hier deinen Denkfehler. Du sagst, Urknallmodell schön und gut, ist ja wohl nicht ganz falsch, aber warum gibt es (seriöse) Wissenschaftler die dem widersprechen und lieferst dan neigentlich gleich die richtige Erklärung dazu, “glaubst” sie aber nicht, warum?
    Es gibt genügend Beispiele für hochintelligente Leute, die sich zu irgendeinem Zeitpunkt “verlaufen”, als beste Beispiele dienen da immer die Nobelpreiscranks, die haben auf ihrem Gebiet hervorragende Forschugn gemacht und denken dann, ich muß auch richtig liegen, wenn ich zum Beispiel an etwas völlig unbelegtes glaube. Dem ist aber nicht so, jeder kann irren und das zu erkennen ist mit so einem Ego wahrscheinlich noch viel schwerer als für unsereiner.
    Wo die Forschung an solchen Themen unterdrückt oder ignoriert wird würde ich ja gern mal sehen, das erscheint mir wie eine gängige Verschwörungstheorie und ich habe es noch nie belegt gesehen, kannst du das zeigen?
    Natürlich wird Forschung nicht von öffentlichen Geldern unterstützt, wenn sie in sich schon inkonsistent ist, egal ob sie dem aktuell gängigen Modell widerspricht oder nicht und da liegt eher der Hund begraben.

  153. #156 Franz
    2. Oktober 2015

    @Andreas
    Warum werden ihre Forschungsergebnisse und Publikationen (sofern diese in anerkannten Fachzeitschriften veröffentlicht werden) mit Ignoranz behandelt?
    Vielleicht weil sich viele der Leser schon mal mit der Thematik auseinandergesetzt haben und gleich am Anfang ein paar gravierende Fehler finden, oder die Ergebnisse anderen Studien widerspricht, die schon mal als sehr zutreffend bewertet wurden. Andere Lösungen werden dann immer unwahrscheinlicher und da die Mittel für Forschung begrenzt sind, muss man sich auf bereits gewonnene Erkenntnisse stützen und kann nicht in allen Richtungen arbeiten.

    Andererseits, was spricht dagegen sich die Daten von Raumsonden, Observatorien und Beobachtungsanlagen zu holen und an einer alternativen Theorie zu arbeiten, quasi in der ‘Freizeit’, so wie Einstein.

    Übrigens, hat Einstein eigentlich jemand mal gerügt, warum er während der Arbeitszeit an seinen Theorien gearbeitet hat (Achtung Ironie)

  154. #157 Franz
    2. Oktober 2015

    @Captain E.
    Es besteht auch die Frage: Was bedeutet es wenn man von einem ‘zerstörten’ Universum spricht. Ist IMO gar nicht so einfach zu definieren.

  155. #158 Captain E.
    2. Oktober 2015

    Ich schätze, diese hypothetischen “Alternativtheorie-Artikel” tauchen wegen des Peer-Review-Prinzips nicht in den Journalen auf. Gute Peer-Reviewer werden einen alternativen Ansatz nicht in Bausch und Bogen verwerfen, aber den wissenschaftlichen Standards muss er natürlich schon genügen. Wenn es daran hapert, dann gibt es eben keine Veröffentlichung in einer seriösen Fachzeitschrift.

  156. #159 Captain E.
    2. Oktober 2015

    @Franz:

    Wohl wahr! Ich hatte jetzt einmal eine implizite Definition des Universums nach Alderamin vorausgesetzt.

  157. #160 Florian Freistetter
    2. Oktober 2015

    Über Alternative Kosmologie wird mehr als genug publiziert… https://scienceblogs.de/astrodicticum-simplex/2012/11/09/ist-wissenschaft-dogmatisch/

  158. #161 Captain E.
    2. Oktober 2015

    Ich bezog mich oben übrigens auf den Wettbewerbs-Gastbeitrag, in dem Alderamin so schön geschildert hat, wie sich das Universum entwickeln könnte über Zeiträume, die das bisherige Alter um ein Vielfaches übersteigen.

    @Andreas:

    Mach dir bitte einmal folgendes klar: Physiker und Astronomen stellen keine unveränderlichen Wahrheiten auf, sondern Modelle. Jetzt wird dich natürlich der Gedanke durchzucken, dass ein Modell niemals die Wirklichkeit ist. Recht hättest du zumindest. Niemand kann sich ein 1:1-Modell der Wirklichkeit zurecht zimmern. So gesehen müssen alle Modelle falsch sein, und das sind sie natürlich auch.

    Der Trick besteht nun darin, die Modelle so gut zu machen, dass man trotzdem daraus brauchbare Berechnungen und damit sogar Vorhersagen machen kann. Wenn man etwas erst berechnet und danach beobachtet und beides gut übereinstimmt, dann hat man ein gutes Modell verwendet. Wenn es gravierende Abweichungen gibt, war das Modell für diesen Fall nicht besonders gut, und man muss sich Gedanken darüber machen, was daran zu verbessern wäre. Wenn so ein verbessertes Modell für einen bekannten Einzelfall plötzliche schlechtere Resultate liefert, dann war die vorgenommene Änderung mit an Sicherheit grenzender Wahrscheinlichkeit keine gute, und man muss das Ganze noch einmal überdenken.

    Das kosmologische Modell hat etwas von einem schlecht geschriebenen Computerprogramm, dem sogenannten “Sphagetti-Code”. Ändert man an der einen Stelle etwas, wackelt es womöglich an einer ganz anderen. Die Wissenschaftler schauen sich dieses Knäuel aber immer sehr genau an und sind ziemlich gut darin, ein “Wackeln” zu bemerken.

  159. #162 Gerrit
    2. Oktober 2015

    @Franz

    Andere Lösungen werden dann immer unwahrscheinlicher und da die Mittel für Forschung begrenzt sind, muss man sich auf bereits gewonnene Erkenntnisse stützen und kann nicht in allen Richtungen arbeiten. Andererseits, was spricht dagegen sich die Daten von Raumsonden, Observatorien und Beobachtungsanlagen zu holen und an einer alternativen Theorie zu arbeiten…

    Raumsonden, Observatorien und Beobachtungsanlagen werden gebaut um “alternativen Theorie” zu testen. Man gibt doch kein Geld für Wissenschaft aus, wenn man nicht erwarten kann etwas Neues zu lernen.

  160. #163 Andreas
    3. Oktober 2015

    @ all

    Da habe ich ja eine ganz schöne Diskussion losgetreten…
    Ich danke allen, sich daran beteiligt haben, besonders Alderamin für seine ausführlichen und informativen Posts.
    Aus Zeitmangel bitte um Verständnis, dass ich nicht auf alle Beiträge antworten kann.

    Eine Sache noch, die ich nicht ganz verstehe:
    – Vakuumfluktuation (Casimir-Effekt): Wie Alderamin schreibt, kann der Energiegehalt in eines hinreichend kleinen Vakuums auf Dauer nicht = 0 sein. Diese Vakuumfluktationen finden zwar in einem idealen Vakuum statt, dieses Vakuum aber befindet sich doch in einem bereits vorhandenen Raum. Ist das nicht ein gravierender Unterschied zwischen heutigen experimentellen Messungen und einer Ära, wo die Vakuumfluktuationen den Raum erst schaffen mussten? Anders ausgedrückt: Kann es ein Vakuum in einem (noch) nicht existierenden Raum geben?
    Andererseits kann auch der Casimir Effekt ohne Quantenfluktuation durch quantentheoretische Störungsrechnungen hergeleitet werden (Wikipedia – Van-der-Waals-Kräfte). Allerdings habe ich davon noch nie was gehört und gebe offen zu, diesen Sachverhalt (in diesem Falle Null Ahnung!) beurteilen zu können.

    Zur Inflation: Danke an Alderamin für seine Darstellung. Ich hatte es bisher so verstanden, dass das Universum ohne Inflation weder seine heutige Größe in der seit dem Urknall zur Verfügung stehenden Zeit erreichen konnte, noch bestimmte (komme jetzt nicht auf Einzelheiten) grundlegende Eigenschaften des Universums ohne ihn erklärbar wären.

    Nochmals zum Abschluss: Ich bin kein Gegner der Urknalltheorie, aber werde sie weiterhin kritisch hinterfragen, solange keine stichhaltigen Beobachtungen für sie sprechen. (z.B. Zweifelsfrei nachgewiesene Temperaturschwankungen der kosmischen Hintergrundstrahlung und natürlich der Nachweis der in der Inflationsphase entstandenen Gravitationswellen).

  161. #164 Alderamin
    3. Oktober 2015

    @Andreas

    Kann es ein Vakuum in einem (noch) nicht existierenden Raum geben?

    Das weiß man nicht, nicht ohne Theorie der Quantengravitation. Eine Möglichkeit (die z.B. in Lawrence Krauss’ Buch angesprochen wird) wäre, dass auch die Raumzeit selbst fluktuiert, also Raumzeit in Dimensionen der Planck-Länge und Planck-Zeit plötzlich größer und kleiner wird oder aus Nichts entsteht und sogleich wieder verschwindet. Wenn das enthatltene Vakuum dann einen zufälligen Energiegehalt hat, der prinzipiell unbeschränkt ist, muss der Prozess nur hinreichend oft passieren, bis irgendwann mal zufällig eine so hohe Energiedichte entsteht, dass eine inflationäre Ausdehnung losgetreten wird, und das kann durchaus ein wenig dauern (unten, vorletzter Absatz). So jedenfalls die Hypothese.

    Andererseits kann auch der Casimir Effekt ohne Quantenfluktuation durch quantentheoretische Störungsrechnungen hergeleitet werden (Wikipedia – Van-der-Waals-Kräfte). Allerdings habe ich davon noch nie was gehört und gebe offen zu, diesen Sachverhalt (in diesem Falle Null Ahnung!) beurteilen zu können.

    Martin Bäker hat dazu mal was geschrieben.

    Ich hatte es bisher so verstanden, dass das Universum ohne Inflation weder seine heutige Größe in der seit dem Urknall zur Verfügung stehenden Zeit erreichen konnte, noch bestimmte (komme jetzt nicht auf Einzelheiten) grundlegende Eigenschaften des Universums ohne ihn erklärbar wären.

    Im wesentlichen löst die Inflation das Flachheits- und das Horizontproblem. In der Hintergrundstrahlung sind Gebiete auf der gleichen Temperatur, die so weit voneinander entfernt sind, dass das Licht diese Strecke zum damaligen Weltalter noch nicht hätte zurücklegen können. Das ist das Horizontproblem: wie gelang der Temperaturausgleich?

    Bei der Inflationstheorie war das Universum zunächst klein genug, um einen Temperaturausgleich mit Lichtgeschwindigkeit zu bewerkstelligen und wuchs dann sprunghaft so stark, dass enthaltene Gebiete kausal entkoppelt wurden, aber da sie mit der gleichen Temperatur starteten, behielten sie diese auch über die Inflation hinaus.

    Das Flachheitsproblem sagt aus, dass das Universum bei minimaler Abweichung von der kritischen Dichte für geometrische Flachheit (Dreiecke haben 180° Winkelsumme, Kreise haben 2*’Pi*r Umfang; nach der ART krümmen Masse und Energie ja den Raum, und dann gelten diese Euklidischen Beziehungen nicht mehr) entweder sehr schnell dichter oder sehr schneller weniger dicht werden müsste, so dass eine positive oder negative Krümmung resultieren müsste. Die Inflation hätte den Raum aber gewissermaßen flachgezogen, wie die Oberfläche eines großen Ballons, so dass die Krümmung vernachlässigbar wurde.

    Ohne Inflation könnten z.B. zwei unendlich große kollidierende Branes der M-Theorie in einer höheren Dimension kollidiert sein, wie zwei Platten, die sich berühren. Wenn diese jeweils im Temperaturgleichgewicht waren, dann wäre es auch das resultierende Universum, uind die Branes könnten auch beide flach sein; eine davon wäre dann unser Universum.

    Dann gibt’s noch die Möglichkeit eines vorherigen Big Crunch eines Vorgängeruniversums, in welchem die Temperatur schon ausgeglichen war (weiß nicht genau, ob dies auch das Flachheitsproblem lösen würde).

    Oder es war ganz anders. Jedenfalls bleibt der Rest der Urknalltheorie damit erhalten. Das Schöne an der Inflation ist, dass sie auch die Feinstruktur der Hintergrundstrahlung bzw. der Galaxienverteilung erklärt, denn sie hat Quantenfluktuationen in makroskopische Dichteunterschiede vergrößert, aus denen dann durch ihre höhere Anziehungskraft Filamente von Galaxienhaufen wuchsen. Mit der Dunklen Energie hat sie eine kleine Schwester, die beobachtbar ist. Und sie erlaubt dem Universum, in Summe eine Energie von Null zu haben, denn für die Arbeit, die sie zur Verdünnung der Dichte der Materie gegen deren Schwerkraft aufbrachte, konnte sie sich endlosen Kredit bei der Gravitation holen, die die Schulden als negativ zu wertende potenzielle Energie anschrieb (die Materie könnte die Arbeit wieder freigeben, wenn man sie auf ihren Schwerpunkt zurück fallen ließe; wie einen Ball, den man aufhebt, und der die aufgebrachte Energie zum Hochheben wieder freigibt, wenn man ihn fallen lässt). Das macht die Inflation attraktiv. Es geht aber auch ohne.

    Zweifelsfrei nachgewiesene Temperaturschwankungen der kosmischen Hintergrundstrahlung

    Zweifelsfrei hat die doch schon COBE nachgewiesen, WMAP genauer vermessen, und PLANCK dies mit noch höherer Genauigkeit wiederholt. Die bunten Flecken in diesen Bildern sind die Temperaturschwankungen, im Millionstel Kelvin-Bereich, aber ohne Zweifel nachgewiesen, und das wiederholt.

    natürlich der Nachweis der in der Inflationsphase entstandenen Gravitationswellen

    Der Nachweis durch bestimmte Polsarisationmoden in der Hintergrundstrahlung wäre ein guter Beleg für die Inflation, der Nicht-Nachweis aber keine Falsifizierung, denn die Gravitationswellen müssen nicht unvermeidlicherweise eine Signatur in der Hintergrundstrahlung hinterlassen haben, die wir nachweisen können. Natürlich wäre es schön, wenn diese B-Moden bald nachgewiesen werden. Die Beobachtungen und Auswertungen laufen.

  162. #165 Andreas
    5. Oktober 2015

    @ Alderamin

    Der Beitrag von Martin Bäker (Casimir-Effekt) ist schon interessant und beantwortet auch meine Fragen, die so im Raum standen.

    “Zweifelsfrei nachgewiesene Temperaturschwankungen der kosmischen Hintergrundstrahlung”:

    Grundsätzlich sicher richtig – aber lt. Wkipedia (Stand Sept. 2015) gibt es doch noch einige Abweichungen, die erklärt werden müssen…
    “Trotz der generell ausgezeichneten Übereinstimmung der gemessenen Eigenschaften des kosmischen Mikrowellenhintergrunds mit den theoretischen Vorhersagen gibt es einige Aspekte in den Daten, die nicht vollständig verstanden sind und zu anhaltenden Diskussionen führten…”
    Es bleibt also weiterhin spannend…

    In “Die Zukunft des Universums” – das ist genau der von dir verlinkte Artikel schreibt Florian zuletzt:

    “Die andere Möglichkeit wäre, dass irgendwo im immensen zukünftigen Universum durch eine Quantenfluktuation wieder spontan ein kleines, falsches Vakuum entstünde und somit ein neues Universum, das per Inflation seinen eigenen Raum generieren würde. Das wäre ein extrem unwahrscheinlicher Vorgang, aber nach 10 hoch10 hoch 56 Jahren könnte man in etwa damit rechnen…
    Vermutlich gebiert unser Universum also noch unzählige weitere, und das Spiel wiederholt sich auf ewig in einem endlosen Raum.”

    Betrifft das nur unser Universum? Könnten nicht auch parallel zu unserem Universum existierende Universen dieselbe Entwicklung nehmen? Wenn dies immer wieder und wieder passiert (s. o. ), dann auch “an allen Orten”, würde ein fiktiver außenstehender Beobachter nicht den Eindruck haben, dass das Universum, wohin man auch blickt, immer gleich aussieht? Er würde mehrere Universen “sehen”, in unterschiedlichen Entwicklungsstufen, aber wohin er auch blickt, das für ihn sichtbare Universum wäre für ihn in allen Richtungen und zu jeder Zeit im Durchschnit gleich. Ein seit ewigen Zeiten bestehendes Universum… Kaum vorstellbar, aber auch die Entstehung aus dem “Nichts” ist für mich genauso unbefriedigend.

    Ist dieses Szenario wirklich so weit von der Steady-State-Theorie entfernt? Im astrophysikalischem Sinn sicher, da fehlt es noch an Erklärungen wie die ständige Erzeugung neuer Materie. Aber ansonsten würden sich Urknalltheorie und die Steady-State-Theorie in ihrer Gesamtheit betrachtet gar nicht ausschließen, da die eine nur einen “kleinen” Ausschnitt (Urknall), die andere hingegen die Gesamtheit des Universums betrachtet (nicht erklärt; das kann sie nicht).

    @ Alderamin

    Der Beitrag von Martin Bäker (Casimir-Effekt) ist schon interessant und beantwortet auch meine Fragen, die so im Raum standen.

    “Zweifelsfrei nachgewiesene Temperaturschwankungen der kosmischen Hintergrundstrahlung”:
    Grundsätzlich sicher richtig – aber lt. Wkipedia (Stand Sept. 2015) gibt es doch noch einige Abweichungen, die erklärt werden müssen…
    “Trotz der generell ausgezeichneten Übereinstimmung der gemessenen Eigenschaften des kosmischen Mikrowellenhintergrunds mit den theoretischen Vorhersagen gibt es einige Aspekte in den Daten, die nicht vollständig verstanden sind und zu anhaltenden Diskussionen führten…”

    Es bleibt also weiterhin spannend…

    In “Die Zukunft des Universums” – das ist genau der von dir verlinkte Artikel schreibt Florian zuletzt:
    “Die andere Möglichkeit wäre, dass irgendwo im immensen zukünftigen Universum durch eine Quantenfluktuation wieder spontan ein kleines, falsches Vakuum entstünde und somit ein neues Universum, das per Inflation seinen eigenen Raum generieren würde. Das wäre ein extrem unwahrscheinlicher Vorgang, aber nach 10 hoch10 hoch 56 Jahren könnte man in etwa damit rechnen…

    Vermutlich gebiert unser Universum also noch unzählige weitere, und das Spiel wiederholt sich auf ewig in einem endlosen Raum.”

    Betrifft das nur unser Universum? Könnten nicht auch parallel zu unserem Universum existierende Universen dieselbe Entwicklung nehmen? Wenn dies immer wieder und wieder passiert (s. o. ), dann auch “an allen Orten”, würde ein fiktiver außenstehender Beobachter nicht den Eindruck haben, dass das Universum, wohin man auch blickt, immer gleich aussieht? Er würde mehrere Universen “sehen”, in unterschiedlichen Entwicklungsstufen, aber wohin er auch blickt, das für ihn sichtbare Universum wäre für ihn in allen Richtungen und zu jeder Zeit im Durchschnit gleich. Ein seit ewigen Zeiten bestehendes Universum… Kaum vorstellbar, aber auch die Entstehung aus dem “Nichts” ist für mich genauso unbefriedigend.

    Ist dieses Szenario wirklich so weit von der Steady-State-Theorie entfernt? Im astrophysikalischem Sinn sicher, da fehlt es noch an Erklärungen wie die ständige Erzeugung neuer Materie. Aber ansonsten würden sich Urknalltheorie und die Steady-State-Theorie in ihrer Gesamtheit betrachtet gar nicht ausschließen, da die eine nur einen “kleinen” Ausschnitt (Urknall), die andere hingegen die Gesamtheit des Universums betrachtet (nicht erklärt; das kann sie nicht).

  163. #166 gaius
    5. Oktober 2015

    @Andreas: Interessante Diskussion!

    Hinweis am Rande: Der Artikel “Die Zukunft des Universums” ist von Alderamin selbst (im Rahmen des Schreibwettbewerbs).

  164. #167 Alderamin
    5. Oktober 2015

    @Andreas

    “Trotz der generell ausgezeichneten Übereinstimmung der gemessenen Eigenschaften des kosmischen Mikrowellenhintergrunds mit den theoretischen Vorhersagen gibt es einige Aspekte in den Daten, die nicht vollständig verstanden sind und zu anhaltenden Diskussionen führten…”

    Das dürfte sich auf den Wert des von PLANCK ermittelten Hubble-Parameters beziehen, der nämlich mit seinen Fehlerbalken außerhalb des Bereichs liegt, der von WMAP und dem Supernova-Projekt gemessen wurde. Aber nicht auf die Temperaturschwankungen, da stimmen PLANCK und WMAP überein (wobei die Messungen von PLANCK die bessere Winkelauflösung haben).

    Betrifft das nur unser Universum? Könnten nicht auch parallel zu unserem Universum existierende Universen dieselbe Entwicklung nehmen?

    Durchaus, aber darüber können wir bisher nur spekulieren.

    Wenn dies immer wieder und wieder passiert (s. o. ), dann auch “an allen Orten”, würde ein fiktiver außenstehender Beobachter nicht den Eindruck haben, dass das Universum, wohin man auch blickt, immer gleich aussieht?

    Es ist die Frage, was ein außenstehender Beobachter davon “sehen” könnte und ob er in die entstehenden Universen (mit ihrem eigenen Raum) überhaupt hinein sehen könnte. Hypothetisch (mangels Quantengravitation) könnte sogar in einem Schwarzen Loch ein Universum entstehen, das seinen eigenen Raum produziert – laut Alan Guth braucht es nur eine hinreichend hohe Materie-/Energiedichte, um eine Inflation anzustoßen, und laut dem, was die gesicherte Physik hergibt, müsste die Dichte im Zentrum eines Schwarzen Lochs unendlich sein (wobei die Auffassung ist, dass an dieser Physik noch nachgebessert werden muss, weil sie nicht zur Quantentheorie passt). Jedenfalls würden wir als Außenstehende nichts davon bemerken, was innerhalb des Ereignishorizonts passiert, wie auch immer.

    Kaum vorstellbar, aber auch die Entstehung aus dem “Nichts” ist für mich genauso unbefriedigend.

    Es gibt halt nur zwei Möglichkeiten: ein ultimativer Beginn (wohlmöglich in fernster Vergangenheit) oder ein ewiger Vorgang (ggf. könnte ein Blockuniversum als dritte Möglichkeit zählen, oder auchals eine Variante der ersten Möglichkeit betrachtet werden). Vermutlich werden wir das nie in Erfahrung bringen. Das mag unbefriedigend sein, aber wir haben halt keinen Rechtsanspruch darauf, die Welt in ihrer Gänze erfassen zu können. Es sei an die Zeit erinnert, in der in ein paar Billionen Jahren die Hintergrundstrahlung und die ferneren Galaxien nicht mehr zu sehen sein werden (siehe den von Dir zitierten Artikel). Dann wird man auch nicht mehr in Erfahrung bringen können, dass es einen primordialen Feuerball gab. Lawrence Krauss weist in seinen Vorträgen immer gerne auf diese “miserable Zukunft” hin.

    Ist dieses Szenario wirklich so weit von der Steady-State-Theorie entfernt? Im astrophysikalischem Sinn sicher, da fehlt es noch an Erklärungen wie die ständige Erzeugung neuer Materie.

    Ja, denn die Steady-State-Theorie befasst sich mit diesem unseren beobachtbaren Universum und streitet dessen Entstehung und Evolution ab. Aussagen über Multiversen und die Entstehung neuer Universen sind erst in jüngster Zeit populär geworden und werden von einigen Kosmologen gar als unwissenschaftlich abgetan, da sie teilweise grundsätzlich nicht falsifizierbar sind (je nachdem, welche Version der Inflation nachgewiesen werden kann, aber evtl. implizit daraus folgen: eine rasch lokal endende Inflation kann praktisch nicht überall gleichzeitig enden und würde folglich ewig andauern und forwährend Universen produzieren müssen).

    Die Steady-State-Theorie hat außerdem noch mehr Probleme als nur die Materieerzeugung (und das Verhältnis von Wasserstoff zu Helium und Lithium, das eindeutig auf einen bestimmten Fusionprozess hinweist): sie hat keine Erklärung für die Existenz der Hintergrundstrahlung und keine für deren Struktur. Sie hat keine Erklärung für das beschränkte Alter der Sterne und keine für die Abwesenheit von Quasaren in der jüngeren Vergangenheit. Sie kann auch nicht erklären, warum Galaxien früher kleiner waren und eine höhere Sternentstehungsrate hatten, wie man dies in der Ferne beobachtet. Sie hat keinen Mechanismus dafür, das Universum am Kollaps zu hindern. Sie kann auch nicht den “Zoom-Effekt” erklären, der sich bei fernen Galaxien zeigt: da das Universum früher kleiner war, dieses aber auf die gesamte Himmelskugel projiziert erscheint, erscheinen ferne Objekte so groß, als ob sie viel näher wären (siehe Kurve “Angular diameter“: Objekte bei z=10 erscheinen genau so groß wie bei z=0,3, obwohl sie 10-mal weiter entfernt sind, das muss z.B. auch bei der Vermessung von Winkelabständen in der Hintergrundstrahlung berücksichtigt werden, 400000 Lichtjahre dort erscheinen unter einen Sehwinkel von 1°, was ansonsten bei 23 Millionen Lichtjahren Entfernung der Fall ist). Die Steady-State-Theorie widerspricht somit den Beobachtungen in vielfältiger Weise. Dieses Pferd ist tot.

  165. #168 Andreas
    5. Oktober 2015

    @ gaius

    Danke für den Hinweis!

    @ Alderamin

    Sorry, dass ich den Hinweis auf den Autor überlesen habe.

    Ein Super Artikel! Ein spannendes Thema – ausführlich und fesselnd geschrieben. Besonders das mögliche Endszenario habe ich so noch nicht gelesen.

    Vielen Dank!

  166. #169 Alderamin
    5. Oktober 2015

    @Andreas

    Danke.

    Oben ist übrigens noch was in der Moderation, schau’ später nochmal hier rein, wenn Florian Gelegenheit hatte, es freizuschalten.

  167. #170 Andreas
    7. Oktober 2015

    @ Alderamin

    Vielen Dank für Deinen Post # 167.
    Es ist wirklich beeindruckend, wie Du jeden Beitrag, jede Frage so ausführlich und individuell beantwortest. Das ist sicherlich aufwendiger, als ein bereits vorhandenes Konzept in Form eines eigenständigen Beitrags umzusetzen.

    Ja, okay – Steady-State, auch in ihren Modifikationen scheint tatsächlich tot zu sein. Um alle von Dir genannten Aspekte, die zu einer Bestätigung der Urknalltheorie führen, nachvollziehen zu können, werde ich wohl noch einige Zeit brauchen – Kosmologie ist nun einmal sehr komplex ;.)

    Ich bin bestimmten ad hoc Annahmen bzgl. des Standardmodells trotzdem noch etwas kritisch eingestellt. Aber ich sehe die Sache nach Deinen Ausführungen jetzt etwas differenzierter.

    Und: Dein Beitrag “Die Zukunft des Universums” ist Spitze und mein Favorit des Schreibwettbewerbes.

    Grüße Andreas

  168. #171 Alderamin
    7. Oktober 2015

    @Andreas

    Danke, freut mich, wenn die Antworten zum Verständnis beigetragen haben und der Artikel gefallen hat.

    Ich bin bestimmten ad hoc Annahmen bzgl. des Standardmodells trotzdem noch etwas kritisch eingestellt.

    Ich bin ein bisschen hinterher mit dem Lesen meiner Sky & Telescopes, aber in der August-Ausgabe (oder war’s schon September? Hier der Artikel online) habe ich am Sonntag gelesen, dass man bei zwei Projekten die gravitationsbedingte Verzerrung der Hintergrundgalaxien um nähere Galaxien gemessen hat, um die Halos aus Dunkler Materie zu vermessen, die man um alle Galaxien vermutet, und wurde auch fündig. Das spricht zum Beispiel sehr stark dafür, dass es diese Materie tatsächlich gibt und kein modifiziertes Gravitationsgesetz im Spiel ist. Wie soll ein modifiziertes Gravitationsgesetz etwa um eine abgeflachte Spiralgalaxie einen kugelförmigen Halo mimikrieren?

    Eine kugelförmige Verteilung passt hingegen zu Teilchen, die nicht elektromagnetisch wechselwirken können, denn die Reibung und inelastischen Stöße, die zur Abflachung einer Gaswolke führen, aus der eine Galaxie hervorgeht, beruhen auf der Abstoßung der Elektronen. Ohne die würdest Du auf der Stelle durch Deinen Stuhl und den Erdboden unter Dir Richtung Erdmittelpunkt fallen. Neutrinos und eben auch die vermuteten Teilchen der Dunklen Materie tun genau das. Neutrinos sausen mit fast Lichtgeschwindigkeit durch die Erde hindurch und entschwinden dem Sonnensystem und der Milchstraße. Die langsamen DM-Teilchen schwirren hingegen um den Schwerpunkt der Milchstraße herum und entkommen ihr nicht. So die Theorie und dafür sprechen die Beobachtungen.

  169. […] Unendlichkeit ist ein abstraktes Konzept, welches wir uns schwer vorstellen können. Etwas das sehr, sehr groß ist (zum Beispiel unser Universum), kommt uns unendlich groß vor. Aber wir wissen noch nicht einmal, ob das Universum unendlich groß ist oder nicht. […]

  170. #173 Patrick Bruns
    Berlin
    3. August 2016

    Ich gehe davon aus, dass es keinen Urknall gegeben haben kann. Jedenfalls nicht von der Gesamtheit aller Dinge. Vieleicht gab es soetwas wie einen “Urknall” bezogen auf eine makroskopische Form, größer sogar als Supercluster, die wir noch nicht erkennen, welche vieleicht sogar unser beobachtbares Universum überschreitet und wovon es mehrere gibt, genau wie es mehrere Sonnensysteme, Galaxien und Galaxienhaufen gibt und diese entstehen. So eine Form kann irgendwann mal aus irgendetwas entstanden sein und vieleicht sogar durch eine andere Diemension hervorgekommen sein, doch die Gesamtheit aller Dinge kann schon rein logisch keinen Anfang gehabt haben. Man assoziiert die Geburt eines Individuums, z. B. eines Menschen, insbesondere des eigenen Bewusstseins, Egos, damit , dass alles, selbst die Gesamtheit von allem, irgendwann eine Geburt ,einen Beginn ,gehabt haben muss. Das ist eine begrenzte, egozentrische Sichtweise. Unsere Wahrnehmung hat Grenzen, das Universum hat keine. Aus meiner Überzeugung heraus, dass es nicht nichts geben kann, gehe ich davon aus, dass es unendliche verschiedene und viele Dimensionen, Möglichkeiten, gibt, denn alles ist irgendwann und irgendwo möglich. Das Universum ist unendlich komplex, also auch unendlich weit, hatte nie einen Anfang und erlebt keinen endgültigen Tot. Alle Dimensionen gehen lückenlos ineinander über, ohne Stück für Stück aneinander gereiht zu sein und für die jeweils innewohnenden Individuen sind lediglich die eigene und niedere ,,Ebenen” zugänglich, außer jemand oder etwas nimmt sie mit in höhere. Eigenständig kann kein Individuum aufhören zu existieren und den Horizont seiner Möglichkeiten überschreiten. Mit dem Begriff Dimensionen, meine ich nicht mehrere Universen, da es nur eine Gesamtheit aller Dinge geben kann. Das Universum besteht also sozusagen aus mehreren Dimensionen. Ich unterscheide höhere und niedrigere Dimensionen darin, dass die ,,Höheren” die Eigenschaften der ,,Niedrigeren” mit in sich einschließen und auf diese aufbauen, ohne sich von ihnen abzukapseln. Vielmehr durchdringen sie einander, haben also eine identische Lokalität. Wir sind also von unendlichen Dimensionen durchdrungen, doch wir haben von unserer dreidimensionalen Physis aus keinen Einfluss auf höhere Dimensionen. Als existierende Individuen können wir nicht die wahren Eigenschaften höherer Dimensionen erfahren oder beschreiben. Wir können nur sagen, dass sie da sind – mehr nicht. Man kann es praktisch nicht überprüfen. Es ist von unserem Standpunkt aus nicht zu beweisen. Höhere, als die uns zugänglichen Dimensionen, bleiben uns absolut verborgen, solange wir in unseren Ursprünglichen existieren. Ist das Universum unendlich, dann nicht nur im Bezug auf die Weite (Größe),sondern auch aus entgegengesetzter Perspektive im Bezug auf die Nähe (Feinheit). Damit meine ich die absolute Negation des Kleinen – die Unendlichkeit des Kleinen. Die Größe nährt sich unendlich nah dem ,,Nullpunkt” aber erreicht ihn nicht. Das Große spiegelt sich im Kleinen. Doch auch dies ist bloß eine Metapher, in diesem Fall ein mathematisches Beispiel. Es gibt keine Maßeinheit für Unendlichkeit, erst recht, weil Unendlichkeit auch unendliche Möglichkeiten beinhält. Durch bloße Mathematik kommt man hier zu keinem schlüssigen Ergebnis. Mathematik wird durch unsere begrenzte Wahrnehmung definiert. Sie zeigt nur einen bestimmten Ausschnitt. Also ist die Annahme es gäbe eine erste Dimension auch falsch, denn es gibt unendlich. Es ist richtig, dass wir 3, bzw. 4 Dimensionen, mit der Zeit einbezogen, für uns unterteilen, jedoch sind es nicht die Erste bis Vierte, sondern bloß vier innerhalb unendlich vieler. Wer meint es gäbe lediglich das, was man mit seinen körpereigenen Sinnen oder selbst konstruierten Maschinen wahrnehmen kann, sowie praktisch zu beweisen ist, sieht den Wald vor lauter Bäumen nicht. Nimmt man nämlich an, dass Universum sei unendlich, müsste jemand, der nur nach einzelnen praktisch nachweisbaren Puzzlestücken sucht, unendlich lange, unendlich viel suchen und finden, um es letztendlich zum vollkommenen Muster zusammenzufügen. So funktioniert das nicht. Wenn man die Theorie der uns unzugänglichen Dimensionen mit einbezieht, bedeutet dies sogar, dass wir ab einem bestimmten Punkt der Suche nichts mehr finden würden und fälschlicherweise annehmen würden, wir hätten alles erkannt und ausfindig gemacht. Wissenschaftler, die die Urknalltheorie vertreten verlieren sich im Detail. Ich nehme Unendlichkeit einfach an, denn es zählt für mich zum unumstoßbaren Grundverständnis, seid ich denken kann. Etwas, was sich nicht auf dieses stützt, entzieht sich meiner Meinung nach sämtlicher Rationalität.
    – Der bekannte Physiker, Stephen Hawking, meint folgendes:
    „Wäre die Grenze des Universums nur ein normaler Punkt in Raum und Zeit, könnten wir über ihn hinausgehen und das dahinter gelegene Gebiet zu einem Teil des Universums erklären. Wäre dagegen der Rand des Universums eine Art Riss, eine Region, in der die Raumzeit bis zur Unkenntlichkeit zerstaucht und die Dichte unendlich wäre, hätten wir große Schwierigkeiten, sinnvolle Randbedingungen zu definieren“, schrieb er später. Hawking nahm kurzerhand an, dass das Universum gar keinen Rand und keine Grenze besitzt. Dieses „No-Boundary Proposal“ überwindet gleichsam die in der Physik übliche Unterscheidung von Anfangs- oder Randbedingungen einerseits und Naturgesetzen andererseits. Es macht die Randbedingungen zu einem Teil der Gesetze. Denn eine Quantentheorie der Gravitation eröffnet die Möglichkeit, dass die Raumzeit keine Grenze hat. „Es wäre also gar nicht notwendig, das Verhalten an der Grenze anzugeben“, erklärt Hawking diesen schwierigen Gedanken. „Es gäbe keine Singularitäten, an denen die Naturgesetze ihre Gültigkeit einbüßten, und keinen Raumzeitrand, an dem man sich auf Gott oder irgendein neues Gesetz berufen müsste, um die Grenzbedingungen der Raumzeit festzulegen. Man könnte einfach sagen: Die Grenzbedingung des Universums ist, dass es keine Grenze hat. Das Universum wäre völlig in sich abgeschlossen und keinerlei äußeren Einflüssen unterworfen. Es wäre weder erschaffen noch zerstörbar. Es würde einfach SEIN.

  171. #174 Captain E.
    3. August 2016

    Dir ist aber schon klar, dass die Physik sehr gute Gründe hatte, einen “Urknall” anzunehmen? Ich nenne da nur einmal Temperatur, Hintergrundstrahlung, Expansion und Entropie. Da greift eins ins andere und ergibt ein recht stimmiges Gesamtbild.

  172. #176 Bullet
    3. August 2016

    Das ist aber eine starke Ansage …

    Aus meiner Überzeugung heraus, dass es nicht nichts geben kann

    … die das Universum natürlich brennend interessiert, folgt,

    dass es keinen Urknall gegeben haben kann

    Auch hier hilft “Geschichte der Naturwissenschaft”:
    damals, im alten Griechenland, gab es einen berühmten Philosophen, der es für völlig einsichtig hielt, daß Materie kontinuierlich ist und im Gegenzug absurd, daß Materie aus kleinsten Teilchen bestehen könnte, zwischen denen dann ja leerer Raum sein müsse. Nach seiner Auffassung (gestützt durch den Stand der damaligen Meßtechnik) konnte man einen Wassertropfen durchaus fünfzigmal halbieren, ohne daß sich etwas am Konzept “Wassertropfen” ändere.
    Doof nur, daß dieser berühmte Philosoph Aristoteles hieß und mit seiner Auffassung den durchaus korrekten atomistischen Ansatz des zu seiner Zeit offenbar durchaus bekannten Philosophen Demokrit qua Autorität ins Abseits schob, wo er erst um 1800 wieder herausgeholt wurde.

    Merke: Überzeugungen sind in allen Belangen naturwissenschaftlicher Art nicht nur nutzlos, sondern hinderlich.

  173. #177 Alderamin
    3. August 2016

    @Patrick Bruns

    Zumindest für unser beobachtbares Universum hat es ganz sicher einen Urknall gegeben, bei dem der Wasserstoff entstand, den die Sterne zu Helium umwandeln (sowie das Lithium, das in Sternen zerstört wird). Ansonsten wäre irgendwann mal aller verfügbarer Wasserstoff verbraucht und die Sterne würden verlöschen (was ganz sicher irgendwann einmal in unserem Universum der Fall sein wird). Die Galaxien entfernen sich auch alle voneinander, und wenn man das zurückrechnet, endet man zwangsläufig in einem Punkt. Vorher in einem Feuerball. Den kann man als kosmische Hintergrundstrahlung in großer Ferne heute noch beobachten.

    Es ist aber nicht ausgeschlossen, dass unser Universum nicht das einzige ist, ja, das wird von einigen Theorien fast zwingend verlangt. Das lässt sich aber leider kaum nachweisen, daher kann man darüber nur spekulieren, was die meisten Physiker nur ungerne tun. Hier gibt’s einen netten Artikel, der ein paar Implikationen erklärt. Ansonsten hat Florian ein Buch rezensiert, das ich zu dem Thema empfehlen kann.

    Nichtsdestotrotz hat in unserem beobachtbaren Universum ein Urknall stattgefunden, aus dem die Materie und Energie und wohlmöglich auch die Naturgesetze des beobachtbaren Universums hervor gingen. Das schließt in keinster Weise aus, dass es davor und jenseits davon nicht noch mehr gab oder gibt.

  174. #178 Adent
    3. August 2016

    @Patrick Bruns

    Wir sind also von unendlichen Dimensionen durchdrungen, doch wir haben von unserer dreidimensionalen Physis aus keinen Einfluss auf höhere Dimensionen. Als existierende Individuen können wir nicht die wahren Eigenschaften höherer Dimensionen erfahren oder beschreiben. Wir können nur sagen, dass sie da sind – mehr nicht. Man kann es praktisch nicht überprüfen. Es ist von unserem Standpunkt aus nicht zu beweisen. Höhere, als die uns zugänglichen Dimensionen, bleiben uns absolut verborgen, solange wir in unseren Ursprünglichen existieren.

    Hmmmm, woran erinnert mich diese äußerst “praktische” Argumentation nur…

  175. #179 Spritkopf
    3. August 2016

    @Patrick Bruns

    Ich gehe davon aus, dass es keinen Urknall gegeben haben kann. Jedenfalls nicht von der Gesamtheit aller Dinge.

    Eine Privatphysik der Sorte “Ich kann mir XYZ einfach nicht vorstellen” mag vor 2000 oder auch 500 Jahren noch funktioniert haben, als man weder das Vorwissen noch die technischen Mittel hatte, um die Hypothese XYZ zu überprüfen. Heute nicht mehr.

    die Gesamtheit aller Dinge kann schon rein logisch keinen Anfang gehabt haben.

    Warum in Dreiteufelsnamen greifen eigentlich die Privatphysiker immer so gern auf Logik (oder das, was sie dafür halten) zurück? Warum fragen sie sich nicht erst einmal, was denn bitte logisch daran sein soll, einen bestimmten Sachverhalt zu bestreiten, wenn es für ihn haufenweise Indizien und Belege gibt?

    Mit dem Begriff Dimensionen, meine ich nicht mehrere Universen, da es nur eine Gesamtheit aller Dinge geben kann. Das Universum besteht also sozusagen aus mehreren Dimensionen. Ich unterscheide höhere und niedrigere Dimensionen darin, dass die ,,Höheren” die Eigenschaften der ,,Niedrigeren” mit in sich einschließen und auf diese aufbauen, ohne sich von ihnen abzukapseln. Vielmehr durchdringen sie einander, haben also eine identische Lokalität. Wir sind also von unendlichen Dimensionen durchdrungen, doch wir haben von unserer dreidimensionalen Physis aus keinen Einfluss auf höhere Dimensionen.

    Und hier stellst du eine Menge Behauptungen auf, die alle nur eine einzige Basis haben: Du hast sie dir ausgedacht und findest, dass sie plausibel klingen.

    Leider interessiert das niemanden.

  176. #180 Krypto
    3. August 2016

    @Spritkopf:
    Doch, interessant fand ich die Textwand schon.
    Auch wenn Patrick inhaltlich und logisch arg daneben liegt, finde ich sein Interesse an der Kosmologie gut!
    @Patrick:
    Ich wünsche Dir, dass Du völlig ergebnisoffen Dein Wissen vertiefst und die hier aufgezeigten Argumente beherzigst. Es ist ein höchst spannendes Thema; also schiebe Deine Überzeugung mal weit beiseite und viel Spaß beim Schmökern und Fragen stellen 😉

  177. #181 Rene Böhme
    Leipzig
    28. August 2016

    Ich merke schon, hier wird viel diskutiert.
    Auch wenn ich nicht alles komplett durchgelesen habe, lese ich hier rein gar nix von der Unendlichkeit der Zeit und des Raumes.
    Ich bin mir zu 100% sicher, dass beides so sein muss und ich kann es auch einfach erklären…

    Was war vor dem Urknall und was ist hinter unserem sichtbaren Universum? Mehr Raum und Zeit, ob mit oder ohne Materie, denn irgendwas ist immer irgendwann irgendwo.

    So kann man sich auch leicht den Sinn der Menscheit erklären. Es gibt keinen, weil wir nix sind im Zusammenhang mit der Unendlichkeit des Raumes und der Zeit.

    Der menschliche Verstand ist nicht in der Lage sich Unendlichkeit vorzustellen.
    Ich hoffe ich konnte trotzdem ein wenig Licht ins Dunkle bringen.

    Lg Rene

  178. #182 Captain E.
    28. August 2016

    Nein, das konntest du nicht. Deine Idee taugt nicht als Erklärung, weil sie die Erkenntnisse der heutigen Physik leider völlig ignoriert.

    Nach allem, was wir wissen, gab es vor dem Urknall nichts – keine Materie, keinen Raum, nicht einmal Zeit.

  179. #183 Rene´
    28. August 2016

    Ich finde man sollte auch nicht immer alles glauben, was einem gesagt wird und sich selbst länger Gedanken darüber machen. Früher haben selbst Gelehrte geglaubt, dass die Erde eine Scheibe ist, bis dann mal das Gegenteil bewiesen wurde.
    Es wird doch einfach nur einen Anfangspunkt gesucht, damit man es sich vorstellen kann. Oder kannst du mir beweisen, dass es vorher keinen Raum ohne Materie und keine Zeit gab?

  180. #184 Alderamin
    28. August 2016

    @Captain E.

    allem, was wir wissen, gab es vor dem Urknall nichts – keine Materie, keinen Raum, nicht einmal Zeit.

    Über die Zeit vor dem Urknall wissen wir eigentlich gar nichts und können demgemäß auch nicht ausschließen, dass es da schon irgendetwas gab. Und das Standardmodell der Kosmologie ist ein unendlich großes Universum, das jenseits des kosmologischen Horizonts immer weiter geht.

    Was nicht bedeutet, dass Renés 100%ige Sicherheit irgendwas an unserem Wissensstand ändern würde. Am Ende zählen keine Gefühle, nur Messungen.

  181. #185 Alderamin
    28. August 2016

    @René

    Ich finde man sollte auch nicht immer alles glauben, was einem gesagt wird und sich selbst länger Gedanken darüber machen. Früher haben selbst Gelehrte geglaubt, dass die Erde eine Scheibe ist, bis dann mal das Gegenteil bewiesen wurde.

    Kritisches Denken ist grundsätzlich nicht verkehrt, je nachdem aus wlecher Quelle es kommt, aber der Wissenschaft kann man schon vertrauen, den letzten Stand des Wissens zu repräsentieren, und zum Kritisieren muss man schon ein paar Jahre studiert haben und die Originalarbeiten verstehen.

    Und was die Kugellform der Erde betrifft, spätestens seit den alten Griechen (Erathostenes bestimmte als erster ziemlich korrekt ihren Umfang) war dies bekannt. Es waren nicht die Gelehrten, die das dachten, sondern das ungebildete Volk. Sogar die mittelalterlichen Kaiser hielten als Reichsinsignie den Reichsapfel in der Hand, eine Kugel mit Kreuz, die die Welt symbolisierte.

  182. #186 Rene´
    28. August 2016

    Eigentlich kann ja auch jeder das glauben was er will oder? Das besagt ja auch der Konstruktivismus. Es müssen ja nicht alle die gleichen Wahrheiten haben.

  183. #187 Alderamin
    28. August 2016

    @René

    Jeder darf glauben was er will, aber eine echte Wahrheit gibt’s (makroskopisch, jedenfalls) eigentlich nur eine; alles andere sind nur Scheinwahrheiten, die jemand für richtig hält, weil er’s nicht besser weiß. Und die Wissenschaft ist eine Methode, um der echten Wahrheit möglichst nahe zu kommen. Dass dies im Allgemeinen recht gut gelingt, beweist das ganze Zeugs, das wir seit dem 17. Jahrhundert so entwickelt haben, z.B. das Gerät, auf dem Du herumtippst. Die Grundlagen dafür hat die gleiche Methodik gesetzt, mit der wir das All beobachten und die Beobachtungen bewerten.

    Wobei natürlich die Luft an der vorderen Front der Forschung dünner wird. Von der erfährt man am ehesten durch die Presse, deswegen scheint Forschung immer so vorläufig und unsicher zu sein. Von den Beobachtungen, die das Bild festigen, hört man meistens nicht viel.

  184. #188 Captain E.
    28. August 2016

    In der Mathematik ist das Unendliche gang und gäbe. Abzählbar und überabzählbar sind dabei Kriterien, wie unendlich etwas ist. Es gibt also unendlich viele natürliche Zahlen, bei den ganzen Zahlen kommen die ebenfalls unendlich vielen negativen hinzu, und dann kann man noch die rationalen Zahlen hinzunehmen, die sich als Brüche aus ganzen Zahlen beschreiben lassen. Die reellen Zahlen füllen diese Lücke, und davon gibt es nicht nur (überabzählbar) unendlich viele, viele haben auch unendlich viele Nachkommastellen wie etwa Florians Lieblingszahl Pi oder das enorm wichtige e des natürlichen Logarithmus.

    Die Physik scheint aber eher dahin zu tendieren, dass es Unendlichkeiten gar nicht gibt, gibt es doch so etwas wie die kleinste Zeit oder die kleinste Länge. Das reale Universum ist also womöglich völlig gequantelt und alles andere als unendlich.

  185. #189 PDP10
    28. August 2016

    @Rene:

    Eigentlich kann ja auch jeder das glauben was er will oder? Das besagt ja auch der Konstruktivismus

    Nein. Das besagt der Konstruktivismus nicht.

    Das ist bloß die Ausrede derer die zu faul zum denken sind.

  186. #190 Alderamin
    28. August 2016

    @Captain E.

    Zeitlich Richtung Zukunft ist Unendlichkeit mit der sich beschleunigenden Expansion schon mal wahrscheinlich. Je nachdem, wie man den Zeitbegriff auslegt, kann daraus auch räumliche Unendlichkeit folgen.

    Ansonsten gilt: https://www.astro.ucla.edu/~wright/cosmology_faq.html#RB

  187. #191 Captain E.
    29. August 2016

    @Alderamin:

    Mag sein – man bewegt sich da in beiden Richtugnen auf dünnem Eis. Man könnte ja genausogut sagen, dass in dem (von dir?) vor einiger Zeit skizzierten “Lebens” unseres Universums spätestens wenn das letzte Proton zerfallen sein wird, der Begriff “Zeit” nicht mehr relevant sein wird. Das mag zwar noch Billiarden oder Trilliarden von Jahren dauern, aber unendlich wäre so etwas nicht.

  188. #192 JF
    29. Dezember 2016

    Können wir uns als Menschen vorstellen, dass eine “Oberfläche” 3D ist?

  189. #193 Atilla
    Universum
    2. März 2017

    Ich denke, der Punkt entstand im unendlichem Nichts und da im Nichts die Bedingung dafür vorhanden war, müssen viele Punkte gleichzeitig entstanden sein.

  190. #194 Ralf Schneider
    Deutschland
    8. Juni 2017

    … wenn ich über die Erde wandere, komme ich irgendwann wieder an meinen Startpunkt. Die Entfernung / Zeit kann ich messen. Es ist endlich, nicht unendlich. Die Erde wird von einem Raum umgeben. Wenn dieser endet ??? was kommt danach und danach Unendlichkeit ??? Es gibt nie ein Ende und das ist etwas, was mich seit Kindheit beschäftigt. Aber noch nie hat es jemand erklären können. Schade.

  191. #195 Krypto
    8. Juni 2017

    @Ralf:
    Ich denke, Du hast es nur noch nie verstehen können 😉

  192. #196 Bullet
    8. Juni 2017

    @Ralf: “unendlich” ≠ “endlos”.

  193. […] Heute ist mein Geburtstag. Ich gehe wie jeden Tag zur Arbeit. Am Abend werde ich ein Glas Rotwein trinken – auf mich und das Leben, das mir geschenkt wurde. Einfach so. Für nix. Ein zutiefst verstörender Gedanke, der mich – wenn ich mich wirklich auf ihn einließe – an den Rand des Wahnsinns führen würde. Etwa so wie der Versuch, mir das Universum vorzustellen, diese positiv oder negativ oder gar nicht gekrümmte {Un-}Endlichkeit. […]

  194. #198 Hanz Wuhast
    9. Februar 2018

    Jaja, und die Erde ist eine Scheibe. Blablabla….

  195. #199 Bullet
    10. Februar 2018

    Sollte das ein ernsthafter Kommentar sein oder hast du nur gerülpst?

  196. #200 Andreas
    10. Februar 2018

    Wird schon ernst gemeint sein. Wenn der IQ dem einer Amöbe entspricht, kommt eben so etwas dabei raus …

  197. #201 Bullet
    10. Februar 2018

    Manchmal posten Kommentatoren etwas im “falschen” Artikel. Sowas kann dann beliebig seltsam aussehen.
    Hier allerdings … nuja, ich neige dazu, dir zuzustimmen.

  198. #202 Jens Hafner
    Zürich
    3. März 2018

    Manchmal sinniere ich über mein Aquarium dort drüben in der Ecke, sehe die Fische darin und stelle mir vor, der ein oder andere mache sich Gedanken über die Welt ausserhalb, sieht mich da am Computer, sieht den hellen grossen Fleck neben mir (Fenster), wie er heller und dunker wird über die Stunden hinweg, und kommt zu dem Schluss, es gibt eine Welt ausserhalb des Universums, eine andere Dimension. Und ein Gott (der Futter gibt!) lebt auch dort.
    Doch egal was der Fisch so denkt, er hat keine Ahnung vom Garten dort unten, der Strasse dort hinten, geschweige denn vom Seeufer um die Ecke oder gar der Küste von Frankreich, weit im Westen hinter dem Horizont. Auch wenn er heimlich aus einem Schneckenhaus ein Fernrohr bauen sollte, weder den Kirchturm würde er finden und auch nicht die wahre Sonne, die den hellen Fleck namens Fenster verursacht, den er ja bestenfalls für die Sonne hält. Auch das in der Welt draussen kein Wasser ist, ich weiss nicht, ob ihm das klar ist.

    Und wir? Sind wir nicht genau so verloren im Denken? Vielleicht sehen wir das Ganze namens Universum in viel zu kleinen, zu “fischigen” Massstäben. Ist das alles, was es gibt, in einem Tesserakt zuhause? Aus Laniakea-Pixeln gefertigt? Und viele Tesserakte liegen wie Atome in einer zeitlos harrenden sechsten Dimension nebeneinander und bilden ein kleines Korn am Strand eines siebendimensionalen Planeten in einem achtdimensionalen Universum, wo dann endlich auch Gott zu finden wäre …

    Meine Fische …. ich kann nur hoffen, dass ihr nicht über mich hier draussen nachdenkt. Ihr würdet euch in allem irren. Denn nicht einmal ein Gott bin ich, auch wenn ich euch gleich Futter bringe.

  199. #203 Dampier
    3. März 2018

    @Jens Hafner

    Sind wir nicht genau so verloren im Denken?

    Bin mir nicht sicher, ob der Vergleich so passend ist, aber dein Text gefällt mir :]

  200. #204 Aleister Lavey
    25. Juni 2018

    Warum verschweigt man hier, daß die widerlegte Steady State Theory (SST) einen Nachfolger hat?? Gemeint ist natürlich die Quasi Steady State Cosmology (QSSC)! Erneut war Fred Hoyle einer der Mitbegründer. Die QSSC: Das Universum ist nicht entstanden, sondern die Raumzeit hat schon immer existiert und *wird* immer existieren. Das Universum ist unendlich groß. Das *sichtbare* Universum entstand durch einen lokalen “Mini Bang”. Mini Bangs finden immer wieder irgendwo statt. Sie passieren nicht außerhalb von Raum und Zeit (im Gegensatz zum Urknall in der Big Bang Cosmology), sondern innerhalb der unendlichen Raumzeit. Ein anderer Mini Bang könnte z.B. in 10 Stunden 50 Trilliarden LJ von uns entfernt stattfinden. Logischerweise würden wir das nie registrieren. “Unser” Mini Bang ereignete sich vor ca. 13,7 Milliarden Jahren und wird als “der Urknall” (der das gesamte Universum erschaffen haben soll) fehlinterpretiert! Das komplette Universum ist von einem sog. “C-Feld” erfüllt, das grob gesagt das Gegenstück zur “Dunklen Energie” (bekannt aus der Big Bang Cosmology) ist. Hat sich an einem Ort sehr viel Masse auf kleinem Raum zusammengefunden, wird die lokale Raumzeit instabil und das C-Feld bringt in einem explosivem Prozess neue Materie (zwischen 10 hoch 6 und 10 hoch 16 Sonnenmassen) hervor. Die lokale Raumzeit expandiert dann. In Phasen mit *vielen* Mini Bangs expandiert auch das *gesamte* Universum. In Phasen mit *wenigen* Mini Bangs findet eine Kontraktion statt. Das Universum oszilliert also.

  201. #205 Aleister Lavey
    25. Juni 2018

    Ich kann sie schon kommen hören, die Zweifel der “Urknaller” und die Proteste der Christen (die QSSC hat nichtmal *ansatzweise* etwas mit einem Schöpfergott zu tun)! Doch warum wird die QSSC in den Old-School-Medien von Professor Lesch (berichtet doch sonst über nahezu alles) und Co. totgeschwiegen?? Warum verschweigt man etwas? Antwort: Weil dieses Etwas einem schaden könnte, falls es publik wird! Wäre die QSSC irgendwie fehlerhaft, hätten Lesch und die anderen “Urknaller” sie mit Freuden im TV in epischer Breite zerpflückt! Doch der Urknall als göttlicher Schöpfungsakt muß ja unangetastet bleiben! Das haben die Päpste Johannes Paul II und besonders Pius XII ja auch gefordert und deshalb Kosmologen und Institutionen beeinflußt (ist alles belegbar!)! Während sich Hawking jedes Jahr mit dem jeweils aktuellen Papst traf, machten Anti-Urknall-Kosmologen in einem Statement klar, daß sie und ihre Forschungen benachteiligt werden. Wer denkt, daß diese Diskriminierung *nicht* vom Vatikan ausgeht (der in punkto Astronomie ja stets gelogen hat), dem ist nicht mehr zu helfen! Da auch Kosmologen Familien haben, die es zu ernähren gilt, sind natürlich die meisten Anti-Urknall-Kosmologen notgedrungen ins Urknall-Lager gewechselt!

  202. #206 PDP10
    25. Juni 2018

    @Aleister Lavey:

    Wenn Sie den ganzen Quatsch mit den Verschwörungstheorien im zweiten Post weggelassen hätten, könnte man über den ersten Post möglicherweise ja diskutieren – obwohl er hier an der falschen Stelle ist, weil es im Artikel oben gar nicht um Urknall vs. kein Urknall geht.

    Aber so … nö danke.

  203. #207 Andreas
    25. Juni 2018

    Um es vorweg zu sagen, auch ich bin nicht vom Big Bang nach gängiger Lehrmeinung überzeugt. Zu viele Postulate, für die es keine Stütze durch tatsächliche Beobachtungen gibt, haften ihm an. Und doch gibt es derzeit kein besseres Modell, welches tatsächlich funktioniert.
    Die QSSC mag interessant klingen, aber auch sie bedient sich unbewiesener Annahmen, wie zum Beispiel dem hypothetischen C-Feld. Um die Mini Big Bangs zu initialisieren, braucht es Weiße Löcher, die über Wurmlöcher von einem Schwarzen Loch gespeist werden. Und mit Ausnahme der letzteren, gibt es keinerlei Hinweise oder gar Belege (geschweige denn Beweise), dass diese tatsächlich existieren.

    Zum Thema Big Bang und Vatikan:
    Ich bin in der ehemaligen DDR aufgewachsen. Dort wurde ein materialistischen Weltbild vermittelt. Zu diesem gehörte auch die Urknalltheorie, aber als Kontrahent (!!!) zur biblischen Schöpfungslehre.
    Ich habe auch recherchiert und kann bestätigen, dass eine gewisse Einflussnahme seitens des Vatikan tatsächlich vorhanden war (ist?). Dennoch gibt es viele in “Kosmologiekreisen” bekannte namhafte Wissenschaftler, die “unbehelligt” vom Vatikan auch alternative Ideen in Fachzeitschriften veröffentlichen.
    Und wäre Georges Edouard Lemaître kein Priester und Theologe gewesen, hätte es diese unglückliche Verbindung Urknall = Bestätigung des Schöpfungsaktes vielleicht gar nicht gegeben.

  204. #208 PDP10
    25. Juni 2018

    Und wäre Georges Edouard Lemaître kein Priester und Theologe gewesen,

    *oherrimhimmel*

    Wie oft noch?

    Lemaitre war promovierter Theoretischer Physiker und Theologe – und hat als Priester nie praktiziert …

  205. #209 PDP10
    26. Juni 2018

    Nachtrag:

    Glaubt irgendjemand, dass sich irgendein ernsthafter Wissenschaftler / eine Wissenschaftlerin die sich mit Kosmologie oder Elementarteilchenphysik beschäftigen dafür interessieren, was der Vatikan dazu zu sagen hat?

    Gibts vom Papst Papers in der Physical Review D?

    I dare doubt it.

  206. #210 Andreas
    26. Juni 2018

    @PDP10
    “Georges Edouard Lemaître (* 17. Juli 1894 in Charleroi, Belgien; † 20. Juni 1966 in Löwen, Belgien) war ein belgischer Theologe, katholischer Priester und Astrophysiker”. – Originaltext Wikipedia-.
    Natürlich war er Priester, wenn er dieses Amt auch nie tatsächlich ausgeübt hat. Wo ist jetzt das Problem?

  207. #211 PDP10
    26. Juni 2018

    @Andreas:

    Wo ist jetzt das Problem?

    Das das aus dem Zusammenhang gerissen ist und damit gelogen durch Auslassung.

    Leider hat der Filter meine Antwort gefressen, in dem ich die ersten beiden Absätze des Wikipedia Artikels unter “Leben und Werk” zitiert habe.

    Wer des lesens mächtig ist, kann selbiges aber tun.

    Hat der Papst jetzt Papers in der PhysRev D oder nicht?

  208. #212 Spritkopf
    26. Juni 2018

    @Andreas

    Ich habe auch recherchiert und kann bestätigen, dass eine gewisse Einflussnahme seitens des Vatikan tatsächlich vorhanden war (ist?).

    OK, Johannes Paul II hat anläßlich eines Besuchs von Stephen Hawking diesen gebeten, nicht am Urknall selbst zu forschen, da dieser als göttlicher Akt sakrosankt sei. Ich wüsste aber nicht, dass Hawking oder sonst irgendein Kosmologe auch nur einen abgegriffenen Groschen auf die päpstliche Bitte gegeben hätte.

    Dennoch gibt es viele in “Kosmologiekreisen” bekannte namhafte Wissenschaftler, die “unbehelligt” vom Vatikan auch alternative Ideen in Fachzeitschriften veröffentlichen.

    Wie soll denn eine solche “Behelligung” überhaupt aussehen? Exkommunikation? Eine Bannbulle aus Rom? Oder gar ein Inquisitionsgericht mit anschließender Lebendfeuerbestattung?

  209. #213 Captain E.
    26. Juni 2018

    Ob Lemaître nun jemals als Priester gewirkt hat oder nicht, ist aber auch völlig irrelevant.

    Die steile These, der Vatikan und die Päpste hätten den Urknall propagiert, dürfte kaum zu halten sein. War es nicht im Gegenteil so, dass die katholische Kirche in der damals jungen Urknalltheorie eine vermeintlich passende wissenschaftliche Version ihrer biblischen Genesis gesehen hat? Um nicht die eigenen Felle davon schwimmen zu sein, hat man dann kackfrech behauptet, die Wissenschaft hätte soeben die biblische Schöpfungsgeschichte bewiesen. Lemaître soll entsetzt gewesen sein über die Vereinnahmung seiner Ideen.

    Ein Kollege hat neulich ins gleiche Horn getutet. Angeblich wäre es der Wissenschaft noch niemals gelungen, irgendeine Aussage aus der Bibel zu widerlegen. Wenn man in so einer Diskussion in die Details einsteigen will, dann dreht und windet sich die Bibelfraktion und verdreht immer alles so, dass es zu ihrer Denkweise zu passen scheint.

  210. #214 Florian Freistetter
    26. Juni 2018

    @Aleister Lavey: “Warum verschweigt man hier”

    Weil ich vom Vatikan bezahlt werde, natürlich! Ich krieg von denen jede Woche ne Kiste Messwein, einen Stapel All-Purpose-Ablassbriefe und einen Koffer voll Geld!

  211. #215 Alderamin
    26. Juni 2018

    @Spritkopf

    Wie soll denn eine solche “Behelligung” überhaupt aussehen? Exkommunikation? Eine Bannbulle aus Rom? Oder gar ein Inquisitionsgericht mit anschließender Lebendfeuerbestattung?

    Es kommt noch schlimmer! Es gibt eine Resolution (B4), bei der kommenden Versammlung der astronomischen Union in Wien das Hubble-Gesetz in Hubble-Lemaitre-Gesetz umzubenennen! Und Florian hat angekündigt, bei der Versammlung dabei zu sein !!!!!!!!!!!!

    !-!-!-!-!-! V-E-R-S-C-H-W-Ö-Ö-Ö-R-U-N-G !-!-!-!-!-!

  212. #216 Spritkopf
    26. Juni 2018

    Ich krieg von denen jede Woche ne Kiste Messwein

    Und das Zeug ist teuer. Nix Aldiplörre, Marke “Saurer November” aus dem Tetrapak.

  213. #217 Andreas
    Düsseldorf
    8. Dezember 2018

    Hallo,
    ich verstehe nicht wirklich alles, aber ich habe mal (ich glaube es war Lesch im TV), der gesagt hat dass das Universum nicht unendlich sein könnte, weil es sonst auch unendlich viele Sterne geben würde und es heller wäre. Das habe ich immer geglaubt, kann mir jemand mal sagen ob ich es falsch sehe? LG Andreas

  214. #219 Peter Michalicka
    Altmünster/Traunsee/OÖ
    12. Dezember 2018

    Ein Universum nur aus kosmischer Hintergrundstrahlung hat einen Radius von 1967 Milliarden Jahen.

  215. #220 Andreas
    13. Dezember 2018

    @ Peter Michalicka: ???

    Ja, klar, und bis Weihnachten sind es noch 11 Kilometer … Im Ernst – was soll der Unsinn?

  216. #221 Artfulman
    Hamburg
    3. Januar 2019

    Ich meine, die Existenz von schwarzen Löchern deutet auf eine Endlichkeit des Universums hin.

  217. #222 Leviathan
    Lüneburg
    4. März 2020

    Viele interessante Gedanken hier. Ich würde auch nicht mehr annehmen, das das Universum so statisch und geregelt funktioniert wie in der alten Urknalltheorie.

    Ich habe eine Frage an Herr Freistetter, vielleicht mag er mir antworten, auch wenn schon 2020 ist. Oder jemand anderes, der sich gut genug auskennt.

    Vor ein paar Tagen bin ich zum ersten Mal auf den Satz gestoßen: Das Universum ist flach. Ich habe das zunächst für eine Idiotie gehalten, so wie “die Erde ist flach”.

    Und ich muss kritisieren, auch wenn flach hier vermutlich nicht flach im Sinne einer zweidimensionalen Fläche gemeint ist, das klingt ja gerade ebenso unsinnig. Ich glaube, das ist die beste Art falsche Ideen/Vorstellungen zu implizieren.

    Ich will also gern gegenprüfen, ob ich es nun richtig verstanden habe, nachdem ich ein paar Dinge dazu gelesen habe.

    Da wird ja auch wieder das Beispiel vom Gummituch benutzt, das fand ich schon beim Thema Schwarzes Loch irreführend. Wir spechen schließlich immer von Raum, nicht von Fläche, da entstehen ganz falsche Vorstellungen.

    Was also heißt, das Universum ist “flach”? Ich hab ein sehr gutes räumliches Vorstellungsvermögen, und will versuchen zu beschreiben was ich denke:

    Für mich hat “flach” nur Sinn, wenn wir nicht von Fläche im Sinne eines Blatt Papiers sprechen. Genauer gesagt – nicht “flach”, sondern “ohne Krümmung”.

    Zum Beispiel wie ein Linsenglas, das aber keine Krümmung hat, also einen “flachen Schliff”, und damit keine Linsenwirkung hat, sondern lediglich ein Stück Glascheibe bleibt. Und mein Fensterglas verzerrt nichts, also hat es keine Krümmungen.

    Also wenn wir das flache Universum nun mit einem belibigen Körper aus Glas gleichsetzen, so ist es flach in dem Sinne, das es an keinem Punkt eine Verzerrung aufweist. Man kann direkt “hindurchsehen”, ohne Krümmungen der Optik.
    (Damit meine ich nicht örtliche Krümmungen durch Massen)

    Auch dieser Vergleich hinkt, denn wir schauen nicht hindurch, wir sind ja darin. Aber die Struktur des Raums (womit auch immer er gefüllt ist) hat selbst keine Krümmung.
    Es wird also nichts verzerrt.

    Schließt dies nun eine Kugelform oder ähnliches aus? Denn eine eckige Form kann es ja nicht sein, das Universum hat sicher keine Ecken und Kanten, so wie ein Glaswürfel. (Im Grunde kann man dann ja nur annehmen, es habe keine Form, keine Begrenzung, es ist unendlich im wortwörtlichen Sinne.)

    Das Universum ist flach klingt nach Pseudowissenschaft. Vielleicht sollte man besser sagen, es hat keine (Raum-)Krümmung?

    Liege ich richtig?

  218. #223 Captain E.
    5. März 2020

    @Leviathan:

    […]

    Das Universum ist flach klingt nach Pseudowissenschaft. Vielleicht sollte man besser sagen, es hat keine (Raum-)Krümmung?

    […]

    Physiker machen sich das Leben zuweilen auch etwas leichter. Aber ja, genau das ist meines Wissens gemeint. Auf kosmische Distanzen hinweg ist praktisch keine Raumkrümmung festzustellen.

    Analog sei die Erde betrachtet. Die Erdkrümmung kann man meist nicht wirklich wahrnehmen, aber messbar ist sie natürlich. Das Universum ist deutlich “flacher” als die Erdoberfläche.

    Und klar, die Erdoberfläche ist im Grunde zweidimensional und das Universum dreidimensional. Das ist natürlich nicht ganz dasselbe.

  219. #224 Unbenannt
    21. Oktober 2020

    Woher wissen Sie, dass es einen Urknall gab? Gibt es Beweise dafür?

  220. #225 Spritkopf
    21. Oktober 2020

    @Unbenannt

    Eine sehr ausführliche Zusammenfassung des heutigen Wissensstands zum Urknall gibt es drüben bei Alpha Cephei. Ist viel zu lesen, aber auch für Laien verständlich.

  221. #227 Dok
    Neukirchen-Vluyn
    27. April 2021

    Man muss unendlich einfach hinnehmen. Ein Urknall wird in Zeit und Raum nichts besonderes sein,sondern immer und zu jeder Zeit stattfinden. Ich denke das der Urknall an sich zu regional gesehen wird. In Zeit und Raum wird der Urknall nichts besonderes sein. Wir denken einfach nicht unendlich,so sind wir stets bemüht alles zu begrenzen.